Intro Part A & B

¡Supera tus tareas y exámenes ahora con Quizwiz!

A patient who stopped smoking 3 years ago has not resumed smoking and actively encourages other family members to stop smoking. This patient is demonstrating which step in the process of valuing? a) treasuring b) acting c) prizing d) choosing

b) acting This patient is demonstrating the consistency and regularity on the value that is consistent with the acting phase of the process of valuing. Choosing involves freely selecting from alternatives after careful consideration of the consequences of each alternative. Prizing and treasuring involve pride, happiness, and public affirmation.

Which type of quantitative research is conducted to directly influence or improve clinical practice? a) basic research b) applied research c) experimental research d) descriptive research

b) applied research Applied research, a type of quantitative research, is designed to directly influence or improve clinical practice.

A client arrives at the emergency department after experiencing several black, tarry stools. The nurse will develop a cause and effect by: a) asking the client to provide a stool specimen for guaiac testing. b) asking the client if he or she has recently taken ferrous sulfate (iron) or bismuth subsalicylate. c) insisting the client not eat or drink anything until further instructed. d) determining if the client has any food or drug allergies.

b) asking the client if he or she has recently taken ferrous sulfate (iron) or bismuth subsalicylate.

Which guideline should a nurse use when choosing a position (location) in relation to a client during a verbal interaction? a) Assess the client's culture during the initial meeting or assessment. b) Take note of the client's cues when choosing a position and act on these cues. c) The nurse should ask the client where he would like the nurse to position herself and move accordingly. d) Choose a position that is no closer than 2 feet, but no farther than 4 feet.

b) Take note of the client's cues when choosing a position and act on these cues.

A client who was admitted to the acute care unit with angina pectoris is no longer having chest pain. Based on this assessment, which of the following does the nurse decide to do with the plan of care for chest pain? a) Establish a new plan of care b) Terminate the plan of care c) Modify the plan of care d) Continue the current plan of care

b) Terminate the plan of care Terminate the plan of care for chest pain as the expected outcome has been achieved. The plan of care for chest pain does not need to be continued or modified. A new plan of care is not indicated at this time.

A client has requested a translator so that she can understand the questions that the nurse is asking her during the client interview. The nurse knows what is important when working with a client translator? a) That talking loudly helps the translator and the client understand the information better b) That translators may need additional explanations of medical terms c) That it is always okay to not use a translator if a family member can do it d) That talking directly to the translator facilitates the transfer of information

b) That translators may need additional explanations of medical terms It is true that even professional translators don't understand all medical terms and may need some clarification at times

Which client outcome is an example of a psychomotor outcome? a) The client reports testing blood sugar before meals. b) The client demonstrates stair climbing using a quad cane. c) The client identifies signs and symptoms of digoxin toxicity. d) The client reports grilling or baking meats rather than frying.

b) The client demonstrates stair climbing using a quad cane.

At the beginning of prenatal care, the goal for the client was to gain 25 pounds by the end of the pregnancy. At 30 weeks of pregnancy, the client has only gained 1 pound. Which statement(s) would help the nurse most appropriately interpret this data? a) The client is progressing toward achieving the goal. The plan should be continued. b) The client is not achieving the goal. The nurse should determine the reasons the client has not been gaining weight. c) The client has partially achieved the determined goal. The nurse should revise the goal to reflect a more realistic outcome. d) It is too early to evaluate if the goal has been achieved. The client has 10 more weeks of pregnancy.

b) The client is not achieving the goal. The nurse should determine the reasons the client has not been gaining weight.

A unit-based infection control task force was developed in an attempt to reduce catheter-acquired infections. The group consists of 10 team members. During the past three meetings, one person dominated the meeting and did not allow other members ample time to speak. The best way to address the team dysfunction is to: a) have group members issue a written warning to the dominant member. b) have group members confront the dominant member to promote the needed team work. c) plan a meeting where the dominant person cannot attend. d) pick a team leader who is not the dominant member.

b) have group members confront the dominant member to promote the needed team work.

When reviewing information about the growth of healthcare services, which are would the nurse expect to find as the most rapidly growing area? a) hospitals b) home care c) nursing homes d) parish nursing

b) home care Home care services are the most rapidly growing area of healthcare.

A pediatric nurse provides teaching to numerous patients in her care. Which group of children benefits most from being involved in the teaching-learning process? a) toddlers b) school-aged children c) infants d) preschoolers

b) school-aged children Teaching related to infants, toddlers, and preschool children should be directed at the parents. School-aged children are capable of logical reasoning and should be included in the teaching-learning process whenever possible.

An RN working on a busy hospital unit delegates patient care to unlicensed assistive personnel (UAPs). Which patient care could the nurse most likely delegate to a UAP safely? Select all that apply. a) Performing the initial patient assessments b) Making patient beds c) Giving patients bed baths d) Administering patient medications e) Ambulating patients f) Assisting patients with meals

b, c, e, f. Performing the initial patient assessment and administering medications are the responsibility of the registered nurse. In most cases, patient hygiene, bed-making, ambulating patients, and helping to feed patients can be delegated to a UAP.

Which statement regarding FOCUS charting is most accurate? a) Problem, Intervention, Evaluation (PIE) charting is used with focused charting. b) Each note should include each section of the Data, Action, Response (DAR) format of charting. c) The charting focuses on client strengths, problems, or needs. d) The charting focuses on the injury or illness only.

c) The charting focuses on client strengths, problems, or needs. FOCUS charting focuses on client strengths, problems, or needs.

A client diagnosed with advanced lung cancer has a nursing diagnosis of Ineffective Coping. What assessment data would provide evidence to the nurse for this diagnosis? a) The client asks about hospice services. b) The client states, "I hope that I am able to attend my daughter's wedding." c) The client states, "I am sure the doctors have misdiagnosed me." d) The client makes funeral plans.

c) The client states, "I am sure the doctors have misdiagnosed me."

The nurse considers which client aspect as nonverbal communication? a) The client's religious practices b) The client's values and beliefs c) The client's tone of voice d) The client's accent

c) The client's tone of voice

Process an ethical dilemma

1. Ask if this is an ethical dilemma. 2. Gather all relevant information. 3. Clarify values. 4. Verbalize problem. 5. Identify possible courses of action. 6. Negotiate a plan. 7. Evaluate plan.

ANA code of ethics

-The nurse, in all professional relationships, practices with compassion and respect for the inherent dignity, worth, and uniqueness of every individual, unrestricted by considerations of social or economic status, personal attributes, or the nature of health problems. -Responsible for articulating nursing values, for maintaining integrity of the profession and its practice, for shaping social policy.

Question: In order to improve quality care for clients there are four steps that the nurse recognizes as being crucial for the process. Place them in the correct order. 1 Plan a strategy using indicators 2 Implement a change 3 Assess a change 4 Discover a problem

4, 1, 2, 3 Discover a problem Plan a strategy using indicators Implement a change Assess a change

When charting the assessment of a client, the nurse writes, "Client is depressed." This documentation is an example of: a) factual statement. b) relevant data. c) interpretation of data. d) important information.

c) interpretation of data. It is always best to describe behavior rather than to interpret behavior. Recording the client's behavior factually allows other professionals to explore causes of the behavior with the client. Stating that "Client is depressed" is an interpretation of the client's behavior and not a factual statement.

A parent of a 17-year-old high school student is allowing the child to make the decision on the college he will attend. When the child requests direction from the parent in making this decision, the parent responds by informing him that he will need to make this decision on his own. This is an example of which type of value transmission? a) modeling b) responsible choice c) laissez-faire d) moralizing

c) laissez-faire This situation demonstrates laissez-faire value transmission, which is characterized by allowing the adolescent to explore values on his own and the development of a personal value system. The laissez-faire approach involves little or no guidance and can lead to confusion and conflict. Through modeling, children learn which is of high or low value observing parents, peers, and significant others. The moralizing mode of value transmission teaches a complete value system and allows little opportunity for the weighing of different values. Responsible choice encourages children to explore competing values and to weigh their consequences while support and guidance are offered.

A nurse is the discharge planner at a large metropolitan hospital. What would the nurse do to ensure continuity of care as patients move from acute care to home care? a) perform an admission health assessment b) participate in the transfer of patients to the ICU c) make referrals to appropriate agencies d) maintain records of patient satisfaction with services

c) make referrals to appropriate agencies Making appropriate referrals for patients as they move from acute care to home care is an essential component of discharge planning for continuity of care.

During an interaction with a critically ill patient's family, the nurse uses the communication technique of silence. This technique assists the family to: a) communicate with the patient b) plan for discharge c) organize their thoughts d) decrease anxiety

c) organize their thoughts Silence allows the family to organize their thoughts and develop any questions specific to understanding the patient's care.

What components are located at the center of a concept map? a) nursing diagnoses and assessment data b) nursing interventions and patient responses c) patient's current and past medical histories d) short-term and long-term goals

c) patient's current and past medical histories The patient's current ad past medical histories are located at the center of a concept map. The nursing diagnoses are developed and prioritized around the patient's history and reasons for hospitalization. Assessment data are categorized under the nursing diagnosis. Nursing interventions, patient responses, and goals are developed based upon the prioritized nursing diagnoses on a separate page.

It is important for the nurse to distinguish a patient's readiness to learn and ability to learn. An example of a patient's ability to learn includes: a) culture. b) emotional health. c) physical condition. d) social and economic stability.

c) physical condition. The readiness to learn focuses on a patient's willingness to engage in the teaching-learning process. Readiness to learn includes emotional readiness and experiential readiness such as social and economic stability, culture, attitudes toward learning, and past experiences with learning. The ability to learn focuses on physical condition, acuity of senses, cognitive abilities, literacy, level of education, and communication skills.

One of the best ways to affirm the efforts of patients who master new knowledge, attitudes, or skills is through: a) dialogue and discussion b) a grading scale c) positive reinforcement d) encouraging the family to learn with the patient

c) positive reinforcement People feel encouraged and supported when their efforts are acknowledged by another person, especially when they trust and value that other person.

A nurse instructs a group of parents about how to make their home safe for their toddlers. This is an example of teaching aimed toward which of the following? a) restoring health b) facilitating coping c) preventing illness d) promoting health

c) preventing illness Thorough assessment of the patients' learning needs may reveal the need for vehicular safety, home safety, domestic-violence recognition, recreational safety, occupational safety and health education. This type of patient education is aimed at enhancing health-protecting behaviors.

A nurse has volunteered to give influenza injections at a local clinic. What level of care is he demonstrating? a) tertiary b) secondary c) primary d) promotive

c) primary Giving influenza infections is an example of primary health promotion and illness prevention.

Your newly employed friend is a part of the company's HMO. You interpret that to mean the friend a) can have healthcare services from a provider of his choice. b) will be unable to have emergency care. c) receives all healthcare from providers within the organization. d) must pay an additional monthly premium.

c) receives all healthcare from providers within the organization. In most HMOs, the patient does not have a choice of healthcare providers and receives all services from providers within the HMO.

When taking a telephone order from a physician, the nurse verifies that she understands the order by: a) confirming the order with the nurse manager. b) asking the physician to summarize the orders given. c) repeating the order back to the physician. d) faxing the written order to the physician's office.

c) repeating the order back to the physician.

When using facts from the patient's medical record as part of the necessary information to assess learning needs, the nurse is using which type of data source? a) primary b) historic c) secondary d) hospital-owned

c) secondary Past and current patient medical records are considered secondary sources of information and can provide a history of medical problems, as well as documentation of nursing assessments, diagnoses, and interventions.

The mother of a toddler with asthma seeks supports from the parents of other children with asthma. The nurse recognizes that seeking and utilizing support systems is an example of which human dimension? a) physical dimension b) environmental dimension c) sociocultural dimension d) intellectual and spiritual dimension

c) sociocultural dimension Communicating with others and the use of support systems relate to the sociocultural dimension. Also related to the sociocultural dimension is an individual's relationship with others, being connected to a community, and feeling accepted and loved by others.

A nurse is attempting to calm an infant in the nursery. The nurse responds to the highest developed sense by: a) staring into the neonate's eyes and smiling. b) softly humming a song near the neonate. c) swaddling the child and gently stroking its head. d) offering the neonate infant formula.

c) swaddling the child and gently stroking its head.

A nurse is writing an evaluative statement for a patient who is trying to lower her cholesterol through diet and exercise. Which evaluative statement is written correctly? a) "Outcome not met." b) "1/21/15—Patient reports no change in diet." c) "Outcome not met. Patient reports no change in diet or activity level." d) "1/21/15—Outcome not met. Patient reports no change in diet or activity level."

d) "1/21/15—Outcome not met. Patient reports no change in diet or activity level." The evaluative statement must contain a date; the words "outcome met," "outcome partially met," or "outcome not met"; and the patient data or behaviors that support this decision.

A nurse is caring for a client who is newly diagnosed with terminal cancer. The nurse enters the client's room and finds the client sitting in the dark crying. Which of the following statements conveys empathy on part of the nurse? a) "I am so sorry you are going through this. Can we talk?" b) "Can you please tell me why you are crying?" c) "Sitting in the dark is not going to cure your cancer. Let's open the curtains." d) "I know this is hard for you. Is there any way I can help?"

d) "I know this is hard for you. Is there any way I can help?"

A patient who decides to leave the hospital against medical advice (AMA) must sign a form. What is the purpose of this form? a) To indicate the patient's wishes. b) To use in the event of readmission. c) To release the physician and hospital from legal responsibility for the patient's health status. d) To ethically illustrate that the patient has control of his or her own care and treatment.

c) To release the physician and hospital from legal responsibility for the patient's health status. Patients who leave the hospital AMA sign a form releasing the physician and hospital from legal responsibility for their health status. This signed form becomes part of the medical record.

Of the following clinic patients, which one is most likely to have annual breast examinations and mammograms based on the physical human dimension? a) Jane, because her best friend had a benign breast lump removed. b) Sarah, who lives in a low-income neighborhood. c) Tricia, who has a family history of breast cancer. d) Nancy, because her family encourages regular physical examinations.

c) Tricia, who has a family history of breast cancer. A family history of breast cancer is a major risk factor.

Quality of life

Central to discussions about end of life care, cancer therapy, physician assisted suicide, DNR.

Deontology

Defines actions as right or wrong, based on their "right making" characteristics such as fidelity to promises, truthfulness, and justice.

Moral Distress

Describes the anguish experienced when a person feels unable to act according to closely held core values. Ex. nurses who work on oncology, pediatric trauma, ICU.

Feminist Ethics

It looks to the nature of relationships to guide participants in making difficult decisions, especially relationships in which power is unequal or in which a point of view has become ignored or invisible.

A caregiver asks a nurse to explain respite care. How would the nurse respond? a) "A service that allows time away for caregivers" b) "A special service for the terminally ill and their family" c) "Direct care provided to individuals in nursing homes" d) "Living units for people without regular shelter"

a) "A service that allows time away for caregivers" Respite care is provided to enable a primary caregiver time away from the day-to-day responsibilities of homebound patients.

When the initial nursing assessment revealed that a patient had not had a bowel movement for 2 days, the student nurse wrote the diagnostic label "constipation." Which of the following comments is the nurse most likely to hear from the instructor? a) "Hold on a minute . . . Nursing diagnoses should always be derived from clusters of significant data rather than from a single cue." b) "Job well done . . . you've identified this problem early and we can manage it before it becomes more acute." c) "Is this an actual or a possible diagnosis?" d) "This is a medical, not a nursing problem."

a) "Hold on a minute . . . Nursing diagnoses should always be derived from clusters of significant data rather than from a single cue." A data cluster is a grouping of patient data or cues that points to the existence of a patient health problem. Nursing diagnoses should always be derived from clusters of significant data rather than from a single cue. There may be a reason for the lack of a bowel movement for 2 days, or it might be this individual's normal pattern.

The nurse has arranged to start an IV line for a client with pancreatitis. The nurse notes that the client appears anxious about the procedure. What is the most appropriate response by the nurse to decrease the client's anxiety? a) "I will start an IV that will add fluids directly to the blood stream." b) "I will start an IV with the number 18 catheters." c) "I will start an IV, which should not cause you too much pain." d) "I will start an IV, which should not take much time."

a) "I will start an IV that will add fluids directly to the blood stream."

A nurse manager is talking with a new nurse. The nurse manager determines that the new nurse is thinking critically based on which statement?

a) "If I give this medication, the client probably will be sleepy." b) "I don't know if the client understands." c) "If my client gets short of breath, I'm unclear about why." d) "I'm not sure what to do here?" CORRECT ANSWER: A

A nurse is requesting to receive change of shift report at the bedside of each client. The nurse giving the report asks about the purpose of giving report at the bedside. Which response by the nurse receiving report is most appropriate? a) "It will allow for us to see the client and possibly increase client participation in care." b) "It will give me a better sense of what my workload will be today." c) "It will let me see everything that has been done and things that need to be done." d) "It makes our client feel like we care, especially if we start the day off with a clean room."

a) "It will allow for us to see the client and possibly increase client participation in care."

A nurse is calling a physician to communicate a change in the client's condition. According to the I-SBAR-R format for hand-off communication among healthcare personnel, which is the most appropriate way to begin the conversation? a) "My name is Sue Smith, RN and I am calling regarding Mrs. Jones in room 356 at Jefferson hospital." b) "My name is Sue and I am calling about Mrs. Jones, a client of yours at Jefferson hospital." c) "Good morning, I am calling about Mrs. Jones who is a client of yours." d) "I have a client of yours at Jefferson hospital experiencing a change in her condition and needs seen immediately!"

a) "My name is Sue Smith, RN and I am calling regarding Mrs. Jones in room 356 at Jefferson hospital."

The public heath nurse is leaving the home of a young mother who has a special needs baby. The neighbor states, "How is she doing, since the baby's father is no help?" What is the nurse's best response to the neighbor? a) "New mothers need support." b) "The lack of a father is difficult." c) "How are you today?" d) "It is a very sad situation."

a) "New mothers need support." It is important that the nurse maintain confidentiality when providing care. The statement of "New mothers need support" is a general statement that all new parents need help. The statement is not judgmental of the family's roles.

A public health nurse is leaving the home of a young mother who has a special needs baby. The neighbor states, "How is she doing, since the baby's father is no help?" What is the nurse's best response to the neighbor? a) "New mothers need support." b) "The lack of a father is difficult." c) "How are you today?" d) "It is a very sad situation."

a) "New mothers need support." The nurse must maintain confidentiality when providing care. The statement "New mothers need support" is a general statement that all new parents need help. The statement is not judgmental of the family's roles.

When an older nurse complains that nurses just aren't ethical anymore, which reply reflects the best understanding of moral development? a) "The ability to behave ethically must be carefully cultivated; maybe we don't value this sufficiently to pay it the attention it deserves." b) "I don't agree that nurses were more ethical in the past. It's a new age and the ethics are new!" c) "Ethics is genetically determined ... it's like having blue or brown eyes. Maybe we're evolving out of the ethical sense you and your generation had." d) "No kidding! Who could be ethical in a practice setting like this!"

a) "The ability to behave ethically must be carefully cultivated; maybe we don't value this sufficiently to pay it the attention it deserves." The ability to be ethical, to make decisions, and to act in an ethically justified manner, begins in childhood and develops gradually.

The nurse is caring for a client who states that he hears voices in his head that tells him to do bad things. When the nurse enters the client's room, he is talking out loud to someone but there is nobody in the room. How should the nurse record this assessment?

a) Do not document this assessment because he could be using his Bluetooth to talk to his family. b) Document that the client is talking back to the voices in his head. c) Document this assessment based on the client's behaviors. d) Do not document this assessment because it is subjective. CORRECT ANSWER: C

Which of the following actions should the nurse take during the evaluation phase of the nursing process? a) Document improved pain after pain medication administered b) Discontinue indwelling urinary catheter per provider's order c) Provide client with follow-up appointment after discharge d) Have client give input into plan of care upon admission

a) Document improved pain after pain medication administered

A family has lost a member who was treated for leukemia at a nursing unit. The nurse provides emotional support to the family and counsels them to cope with their loss. Which quality should the nurse use in this situation? a) Empathy b) Indifference c) Pity d) Sympathy

a) Empathy

The student nurse understands that the evaluation phase of the nursing process includes which of the following? Select all that apply.

a) Evaluations should be documented daily in the client record. b) Evaluation does not involve nursing assessment. c) The evaluation is used to determine decisions about terminating, continuing, or modifying the plan of care. d) Evaluation is the last part of the nursing process. e) Only factors that positively affect the outcome should be identified during evaluation. CORRECT ANSWERS: A, C and D

A nurse is reading a journal article about providing individualized care. Which aspect would the nurse most likely read about as the almost universally accepted method for providing nursing care?

a) Experience b) Reflection c) Nursing Process d) Clinical reasoning CORRECT ANSWER: C

A theory of nursing will primarily inform which of the following aspects of a nursing student's activities and interactions at the bedside? a) Framing and giving meaning to student-patient interactions. b) Prioritizing nursing activities according to an established criteria. c) Assessing and addressing patients' illnesses. d) Choosing interventions based on evidence.

a) Framing and giving meaning to student-patient interactions. A theory of nursing provides broad, macro-level meaning to the practice of nursing by describing and explaining the nature of nurse-person interactions. Nursing theories do not typically address or inform the more micro-level aspects of nursing such as choosing interventions, acting on assessments, or prioritizing patient needs.

During the orientation to the hospital, the staff development educator discusses unit and institutional-based policies. What is the source of the practice rules that result in unit and institutional-based policies? a) Healthcare institution b) Federal legislation c) State legislation d) Board of nursing

a) Healthcare institution The healthcare institution determines the unit and institutional policies. These policies may vary from institution to institution. Such policies may include clinical procedures, policies specific to the institution, and personnel and employment policies.

A nurse is caring for a toddler who has been treated on two different occasions for lacerations and contusions due to the parents' negligence in providing a safe environment. What is an appropriate nursing diagnosis for this client? a) High Risk for Injury related to unsafe home environment b) High Risk for Injury related to abusive parents c) High Risk for Injury related to impaired home management d) Child Abuse related to unsafe home environment

a) High Risk for Injury related to unsafe home environment

Which of the following family structures is presently most common in the United States? a) Households with two parents, each of whom works outside the home b) Single-parent households headed by a woman c) Traditional nuclear families with one wage earner d) Blended families in which one or both partners bring children to the relationship

a) Households with two parents, each of whom works outside the home The family is an institution that has undergone significant change in recent decades. Currently, two-career families, in which both parents work outside the home, are the norm.

Which nursing organization was the first international organization of professional women? a) ICN b) ANA c) NLN d) NSNA

a) ICN The ICN, founded in 1899, was the first international organization of professional women.

It is important for the nurse to empathize with the client to develop a positive, therapeutic relationship. Which of the following is the characteristic of empathy? a) Identifying with the client's feelings b) Caring for the client without negative judgment c) Conveying genuine care to the client d) Experiencing feelings similar to that of the client

a) Identifying with the client's feelings

Which statement is not true regarding a medication administration record (MAR)? a) If the client refuses the dose you don't have to document this on the MAR. b) The MAR identifies routine times for medication administration. c) When using an electronic MAR, the nurse has to log off so that the next person using the computer does not sign off a medication under her name by mistake. d) The MAR distinguishes between routine and "as needed" medications

a) If the client refuses the dose you don't have to document this on the MAR.

A nurse who is caring for a client admitted to the nursing unit with acute abdominal pain formulates the care plan for the client. Which of the following nursing diagnoses is the highest priority for this client? a) Impaired comfort b) Disturbed sleep pattern c) Activity intolerance d) Disturbed body image

a) Impaired comfort

A nurse has completed a client assessment and is preparing to identify appropriate nursing diagnoses. Which area would the nurse most likely address in the diagnosis? Select all that apply.

a) Impaired mobility b) Imbalanced nutrition c) Ineffective coping d) Pneunomia e) Heart failure CORRECT ANSWERS: A, B, and C

A nurse is examining alternatives and judging the worth of evidence as part of preparing the plan of care for a client. The nurse would most likely be involved in which phase of the nursing process?

a) Implementation b) Diagnosis c) Planning d) Evaluation CORRECT ANSWER: C

Which of the following factors is the primary criterion for admission to a long-term care facility? a) Inability to provide self-care b) Advanced age c) Absence of family members locally d) Coexistence of multiple chronic health problems

a) Inability to provide self-care Long-term care facilities exist to provide care for individuals who are mentally or physically unable to care for themselves independently. These individuals are frequently older, with multiple health problems and many lack family caregivers, but these factors in and of themselves are not criteria for admission to long-term care.

Identifying the kind and amount of nursing services required is a possible solution for: a) Inadequate staffing. b) Nurses who are bored. c) Clients who fail to communicate their needs. d) Nurses frustrated with substandard care.

a) Inadequate staffing.

A client is being admitted from the emergency room reporting shortness of breath, wheezing, and coughing. What would the nurse formulate as an appropriate nursing diagnosis? a) Ineffective Airway Clearance b) Acute Dyspnea c) Bronchial Pneumonia d) Asthma Attack

a) Ineffective Airway Clearance

Which of the following flow sheets provides the health care provider with information on an ongoing record of fluid loss? a) Intake and output graphic sheet b) Health assessment flow sheet c) Vital signs graohic sheet d) Critical care flow sheet

a) Intake and output graphic sheet

Two nurses have disagreed about the role of intuition in nursing practice, with one nurse characterizing it as "hocus-pocus" and the other nurse advocating it as a superior problem-solving strategy. Which of the following statements best conveys the role of intuition in nurses' problem solving?

a) Intuition can be a clinically useful adjunct to logical problem solving. b) In experienced nurses, intuition can be a valid replacement for scientific problem solving. c) Intuition is reliable when those nurses implementing it have a special "gift." d) Intuition is an unreliable mode of thinking that should be avoided. CORRECT ANSWER: A

The nursing process is based upon the process of problem solving. What type of problem solving is the nurse using if she attempts to obtain a blood pressure on the client's right arm, the left arm, the left leg, and then finally the right leg, where a blood pressure is finally obtained?

a) Intuitive thinking b) Trial-and-error problem solving c) Scientific problem solving d) Critical thinking CORRECT ANSWER: B

On a typical day shift, 7 am to 7 pm, the nurse-client ratio on a busy floor is higher than usual because a member of the health care team called in sick for the day. Which example shows a nurse practicing with a good sense of legal competence?

a) It is so busy that some tasks just can not be done and therefore the nurse will just leave those tasks for the night shift, including the 4 pm labs. b) The nurse follows the chain of command and requests help for the tasks that she can not complete and that are important to the client care on the floor that day. c) Because the nurse is so busy there is no time to look up the safe dose for a medication, instead she asks a coworker that has been a nurse on the floor for 10 years about the medication. d) Instead of documenting every 2 hours per hospital protocol, the nurse documents a detailed shift assessment and an end of shift note to cover what has happened during the shift. CORRECT ANSWER: B

Which of the following is a true statement regarding critical thinking in nursing?

a) It supplies validation for reimbursement. b) It makes judgment based on conjecture. c) It shows trends and patterns in client status. d) It is a systemic way of thinking. CORRECT ANSWER: D

A patient who has been admitted to the hospital for the treatment of a gastrointestinal bleed requires a transfusion of packed red blood cells. Which of the following aspects of the nurse's execution of this order demonstrates technical skill? a) Starting a new large-gauge intravenous site on the patient and priming the infusion tubing. b) Understanding the Rh system that underlies the patient's blood type. c) Explaining the process that will be involved in preparing and administering the transfusion. d) Ensuring that informed consent has been obtained and properly filed in the patient's chart.

a) Starting a new large-gauge intravenous site on the patient and priming the infusion tubing. Performing tasks that require manual dexterity is a manifestation of technical skills. Explaining the transfusion process clearly is largely dependent on interpersonal skills, while understanding the theory behind blood types is indicative of cognitive skills. Informed consent lies within the domain of legal/ethical skills.

A new chemical plant is being built in the community. The nurse is concerned about the possibility of environmental pollution adversely affecting the health of the residents. What nursing diagnosis would the nurse use to address this concern? a) Risk for Community Contamination related to possible environmental pollution b) Knowledge Deficit related to effects of chemical plant pollution c) Deficient Community Health related to chemical plant d) Risk for Infection related to community contamination

a) Risk for Community Contamination related to possible environmental pollution

Which of the following nursing diagnoses has the highest priority when caring for an older adult client with Alzheimer's disease? a) Risk for injury b) Self-care deficit c) Impaired physical mobility d) Impaired memory

a) Risk for injury

The nurse is developing a plan of care for a client with a fractured femur, is in traction, and will be restricted to bed for some time. Which domain should the nurse look to for developing a nursing diagnosis based on this client's musculoskeletal health problems?

a) Self-perception b) Activity/Rest c) Health Promotion d) Nutrition CORRECT ANSWER: B

When documenting information in a client's medical record, what should the nurse do consistently for each entry? a) Sign each entry by name and title. b) Obtain a signature from the physician. c) Report each observation to the physician. d) Provide the day of the week on the entry.

a) Sign each entry by name and title.

A couple with adolescent children is most likely to focus on which of the following developmental tasks? a) Strengthening marital relationships. b) Establishing a mutually satisfying marriage. c) Coping with loss of energy and privacy. d) Adjusting to retirement

a) Strengthening marital relationships: The couple in a family with adolescents and young adults likely has a developmental task to strengthen marital relationships. Establishing a mutually satisfying marriage and coping with the loss of energy and privacy are tasks for a couple with young children. Adjusting to retirement is a developmental task for older adults.

How can the nurse researcher obtain information from a client record? a) Study client records. b) Audit discharge records. c) Interview nursing staff. d) Examine institutional procedures.

a) Study client records. Nursing and health care research is often carried out by studying client records.

The nurse enters the room of an adult client who complains of postoperative abdominal pain. The client states that the pain is severe, but is relieved some when she gets up to go the bathroom. What is the nurse's best determination based on this assessment?

a) That the client's pain is really not that bad because he or she can ambulate. b) Even with pain, the client is ambulatory and therefore ready for discharge. c) That the client should not be ambulating with pain. d) More assessment would be beneficial to determine if pain medication is desirable. CORRECT ANSWER: D

A nurse is providing care for three patients on a medical unit, two of whom are significantly more acute than the third. The nurse is making a concerted effort to ensure that the less acute patient still receives a reasonable amount of time, attention and care during the course of the shift. Which of the following is the nurse attempting to enact? a) justice b) beneficence c) fidelity d) nonmaleficence

a) justice The ethical principle of justice includes an effort to fairly distribute benefits and to minimize discrimination, even when circumstances make this difficult to achieve. This is demonstrated by the nurse's efforts to fairly distribute his or her time and care.

A nurse researcher is studying perceptions of vocational rehabilitation for patients after a spinal cord injury. What type of research method will the researcher use to study the perceptions of this group of individuals? a) qualitative research b) applied research c) quantitative research d) basic research

a) qualitative research The nurse researcher will use qualitative research methods to investigate perceptions, and the researcher will analyze words instead of numbers, which are analyzed in quantitative research. Basic and applied research are quantitative research methods.

Who are the largest group of healthcare providers in the United States? a) registered nurses b) physicians c) physical therapists d) social workers

a) registered nurses Registered nurse are the largest group of healthcare providers in the United States.

A nurse is caring for an elderly male patient who is receiving fluids for dehydration. Which outcome for this patient is correctly written? a) Offer the patient 60 mL fluid every 2 hours while awake. b) During the next 24-hour period, the patient's fluid intake will total at least 2,000 mL. c) Teach the patient the importance of drinking enough fluids to prevent dehydration by 1/15/15 d) At the next visit, 12/23/15, the patient will know that he should drink at least 3 liters of water per day.

b

A nurse is providing care to a 3-year-old child admitted with a diagnosis of infectious diarrhea. The nurse needs to insert an intravenous catheter in order to administer prescribed intravenous fluids. In an attempt to foster communication, the nurse should: a) involve the child's stuffed animal in the educational session. b) show the child the intravenous catheter and explain how it works. c) ask the child's parents to leave the room while the nurse and child talk. d) provide both verbal and written information to the child.

a) involve the child's stuffed animal in the educational session.

A nurse is collecting more patient data to confirm a diagnosis of emphysema for a 68-year-old male patient. What type of diagnosis does this intervention seek to confirm? a) Actual b) Possible c) Risk d) Collaborative

b

A school nurse notices that a female adolescent student is losing weight and decides to perform a focused assessment of her nutritional status to determine if she has an eating disorder. How should the nurse proceed? a) Perform the focused assessment. This is an independent nurse-initiated intervention. b) Request an order from Jill's physician since this is a physician-initiated intervention. c) Request an order from Jill's physician since this is a collaborative intervention. d) Request an order from the nutritionist since this is a collaborative intervention.

A. Performing a focused assessment is an independent nurse-initiated intervention

Nurses use the Nursing Interventions Classification Taxonomy structure as a resource when planning nursing care for patients. What information would be found in this structure? a) Case studies illustrating a complete set of activities that a nurse performs to carry out nursing interventions b) Nursing interventions, each with a label, a definition, and a set of activities that a nurse performs to carry it out, with a short list of background readings c) A complete list of nursing diagnoses, outcomes, and related nursing activities for each nursing intervention d) A complete list of reimbursable charges for each nursing intervention

b

A nurse is obtaining a history from an adult female client. When the nurse asks how many times the client has been pregnant, the client answers, "I have four kids." Which statement, made by the nurse, seeks clarification of the original question? a) "All right, you have four children, is that correct?" b) "I understand you have four kids; how many times have you actually been pregnant?" c) "Were these term births?" d) "How old are your children?"

b) "I understand you have four kids; how many times have you actually been pregnant?"

21. A confused older adult patient is wearing thick glasses and a hearing aid. Which intervention is priority to facilitate communication? a. Focus on tasks to be completed. b. Allow time for the patient to respond. c. Limit conversations with the patient. d. Use gestures and other nonverbal cues.

ANS: B Allowing time for the patient to respond will facilitate communication, especially for an older confused patient. Focusing on tasks to be completed and limiting conversations do not facilitate communication; in fact, they block communication. Using gestures and other nonverbal cues is not effective for visually impaired or cognitively impaired patients.

26. A patient says, "You are the worst nurse I have ever had." Which response by the nurse is the most assertive? a. "If I were you, I'd feel grateful for a nurse like me." b. "I feel uncomfortable hearing that statement." c. "How can you say that when I have been checking on you regularly?" d. "You shouldn't say things like that, it is not right."

ANS: B Assertive responses contain "I" messages such as "I want," "I need," "I think," or "I feel." Giving personal opinions ("If I were you") is nontherapeutic and not assertive. Arguing ("How can you say that?") is not assertive or therapeutic. Showing disapproval (using words like should, good, bad, right) is not assertive or therapeutic.

11. During the initial home visit, a home health nurse lets the patient know that the visits are expected to end in about a month. The nurse is in which phase of the helping relationship? a. Pre-interaction b. Orientation c. Working d. Termination

ANS: B Letting the patient know when to expect the relationship to be terminated occurs in the orientation phase. Pre-interaction occurs before the nurse meets the patient. Working occurs when the nurse and the patient work together to solve problems and accomplish goals. Termination occurs during the ending of the relationship.

18. An elderly patient is wearing a hearing aid. Which technique should the nurse use to facilitate communication? a. Speak clearly and loudly. b. Turn off the television. c. Chew gum. d. Use at least 14-point print.

ANS: B Patients who are hearing impaired benefit when the following techniques are used: Check for hearing aids and glasses, reduce environmental noise, get the patient's attention before speaking, do not chew gum, and speak at normal volume—do not shout. Using at least 14-point print is for sight/visually impaired, not hearing impaired.

6. A nurse is sitting at the patient's bedside taking a nursing history. Which zone of personal space is the nurse using? a. Intimate b. Personal c. Social d. Public

ANS: B Personal space is 18 inches to 4 feet and involves such things as sitting at a patient's bedside, taking a patient's nursing history, or teaching an individual patient. Intimate space is 0 to 18 inches and involves such things as performing a physical assessment, bathing, grooming, dressing, feeding, and toileting a patient. Social zone is 4 to 12 feet and involves such things as making rounds with a physician, sitting at the head of a conference table, or teaching a class for patients with diabetes. Public zone is 12 feet and greater and involves such things as speaking at a community forum, testifying at a legislative hearing, or lecturing.

3. A nurse wants to present information about flu immunizations to the elderly in the community. Which type of communication should the nurse use? a. Interpersonal b. Public c. Transpersonal d. Small group

ANS: B Public communication is interaction with an audience. Nurses have opportunities to speak with groups of consumers about health-related topics, present scholarly work to colleagues at conferences, or lead classroom discussions with peers or students. Intrapersonal communication is a powerful form of communication that occurs within an individual. Transpersonal communication is interaction that occurs within a person's spiritual domain. When nurses work on committees, lead patient support groups, form research teams, or participate in patient care conferences, they use a small group communication process.

8. The nurse asks a patient where the pain is, and the patient responds by pointing to the area of pain. Which form of communication did the patient use? a. Verbal b. Nonverbal c. Intonation d. Vocabulary

ANS: B The patient gestured (pointed), which is a type of nonverbal communication. Gestures emphasize, punctuate, and clarify the spoken word. Pointing to an area of pain is sometimes more accurate than describing its location. Verbal is the spoken word or message. Intonation or tone of voice dramatically affects the meaning of a message. Vocabulary consists of words used for verbal communication.

2. Which types of patients can cause challenging communication situations? (Select all that apply.) a. A male patient who is cooperative with treatments b. A female patient who is outgoing and flirty c. An older adult patient who is demanding d. An elderly patient who can clearly see small print e. A teenager frightened by the prospect of impending surgery f. A child who is developmentally delayed

ANS: B, C, E, F Challenging communication situations include patients who are flirtatious, demanding, frightened, or developmentally delayed. A child who has received little environmental stimulation possibly is behind in language development, thus making communication more challenging. Patients who are cooperative and have good eyesight (see small print) do not cause challenging communication situations.

A nurse has taught the patient how to use crutches. The patient went up and down the stairs using crutches with no difficulties. Which information will the nurse use for the "I" in PIE charting? a. Patient went up and down stairs b. Deficient knowledge regarding crutches c. Demonstrated use of crutches d. Used crutches with no difficulties

ANS: C A second progress note method is the PIE format. The narrative note includes P—Problem, I—Intervention, and E—Evaluation. The intervention is "Demonstrated use of crutches." "Patient went up and down stairs" and "Used crutches with no difficulties" are examples of the E. "Deficient knowledge regarding crutches" is the P.

A nurse is a member of an interdisciplinary team that uses critical pathways. According to the critical pathway, on day 2 of the hospital stay, the patient should be sitting in the chair. It is day 3, and the patient cannot sit in the chair. What should the nurse do? a. Focus charting using the DAR format. b. Add this data to the problem list. c. Document the variance in the patient's record. d. Report a positive variance in the next interdisciplinary team meeting.

ANS: C A variance occurs when the activities on the critical pathway are not completed as predicted, or the patient does not meet expected outcomes. An example of a variance is when a patient develops pulmonary complications after surgery, requiring oxygen therapy and monitoring with pulse oximetry. A positive variance occurs when a patient progresses more rapidly than expected (e.g., use of a Foley catheter may be discontinued a day early). When a nurse is using the problem-oriented medical record, after analyzing data, health care team members identify problems and make a single problem list. A type of narrative format charting is focus charting. It involves the use of DAR notes, which include D—Data (both subjective and objective), A—Action or nursing intervention, and R—Response of the patient (i.e., evaluation of effectiveness).

A nurse developed the following discharge summary sheet. Which critical information should be added? TOPIC DISCHARGE SUMMARY Medication Diet Activity level Follow-up care Wound care Phone numbers When to call the doctor Time of discharge a. Kardex form b. Admission nursing history c. Mode of transportation d. SOAP notes

ANS: C List actual time of discharge, mode of transportation, and who accompanied the patient for discharge summary information. In some settings, a Kardex, a portable "flip-over" file or notebook, is kept at the nurses' station. A Kardex is for nurses, not for patients to take upon discharge. A nurse completes a nursing history form when a patient is admitted to a nursing unit, not when the patient is discharged. SOAP notes are not given to patients who are being discharged. SOAP notes are a type of documentation style.

A hospital is using a computer system that allows all health care providers to use a protocol system to document the care they provide. Which type of system/design will the nurse be using? a. Clinical decision support system b. Nursing process design c. Critical pathway design d. Computerized provider order entry system

ANS: C One design model for Nursing Information Systems (NIS) is the protocol or critical pathway design. With this design, all health care providers use a protocol system to document the care they provide. A clinical decision support system is based on "rules" and "if-then" statements, linking information and/or producing alerts, warnings, or other information for the user. The nursing process design is the most traditional design for an NIS. This design organizes documentation within well-established formats such as admission and postoperative assessments, problem lists, care plans, discharge planning instructions, and intervention lists or notes. Computerized provider order entry (CPOE) is a process by which the health care provider directly enters orders for patient care into the hospital information system.

A nurse is giving a hand-off report to the oncoming nurse. Which information is critical for the nurse to report? a. The patient had a good day with no complaints. b. The family is demanding and argumentative. c. The patient has a new pain medication, Lortab. d. The family is poor and had to go on welfare.

ANS: C Relay to staff significant changes in the way therapies are to be given (e.g., different position for pain relief, new medication). Don't simply describe results as "good" or "poor." Be specific. Don't use critical comments about patient's or family's behavior, such as "Mrs. Wills is so demanding." Don't engage in idle gossip.

D

Based on knowledge of Erikson's stages of growth and development, the nurse plans her nursing care with the knowledge that old age is primarily focused on: A. Intimacy versus Isolation. B. Autonomy versus Shame and Doubt. C. Generativity versus Self-Absorption. D. Ego Integrity versus Despair.

A

Based on knowledge of the developmental tasks of Erikson's Industry versus Inferiority, the nurse emphasizes proper technique for use of an inhaler with a 10-year-old boy so he will: A. Increase his self-esteem with mastery of a new skill. B. Accept changes in his appearance and physical endurance. C. Experience success in role transitions and increased responsibilities. D. Appreciate his body appearance and function.

The nurse is caring for a client who is recovering from a cerebrovascular accident (CVA). When reviewing the client's orders, the nurse notes that one of the physicians wrote orders to ambulate the client while another physician ordered strict bed rest for the client. How would the nurse most appropriately remedy this conflict? a) Assess the client to determine if the client is capable of ambulation. b) Communicate with the physicians to coordinate their orders. c) Collaborate with the physical therapist to determine the client's ability. d) Instruct the client to ask the physicians for clarifications of instructions.

b) Communicate with the physicians to coordinate their orders.

What ensures continuity of care? a) Integration b) Communication c) Critical thinking d) Reassessment

b) Communication Communication ensures continuity of care and provides essential data for revision or continuation of care.

What is the primary purpose of the client record? a) Advocacy b) Communication c) Research d) Education

b) Communication The primary purpose of the client record is to help health care professionals from different disciplines communicate with one another.

A nurse caring for an older adult client who has dementia observes another nurse putting restraints on the client without a physician's order. The client is agitated and not cooperating. What would be the best initial action of the first nurse in this situation? a) Report the nurse applying the restraints to the supervisor. b) Confront the nurse and explain how this could be dangerous for the client. c) File an incident report and have the second nurse sign it. d) Contact the physician for an order for the restraints.

b) Confront the nurse and explain how this could be dangerous for the client.

A client who is a new quadriplegic as the result of a motor vehicle accident, is experiencing multiple physical and emotional problems. To guide the care planning for this client, what type of nursing diagnosis would be most appropriate for the nurse to select? a) A risk nursing diagnosis b) An actual nursing diagnosis c) A syndrome nursing diagnosis d) A possible nursing diagnosis

d) A possible nursing diagnosis Since the client is experiencing multiple problems beyond the scope of a single nursing diagnosis, a syndrome diagnosis is indicated. One actual diagnosis is not sufficient. The client has identified problems, so a possible or risk diagnosis are inappropriate.

Which statement related to the evaluation of outcome attainment for a client is correct? a) The nurse should initially evaluate the plan of care at the time of the client's discharge. b) Evaluation of the client's attainment of outcome goals is determined by the nurse and physician. c) Celebrating outcome achievement with a client often interferes with attainment of future goals. d) Collecting data related to outcome attainment requires the nurse to know when to collect the data, based upon established time criteria.

d) Collecting data related to outcome attainment requires the nurse to know when to collect the data, based upon established time criteria.

Which action is appropriate when evaluating a client's responses to a plan of care? a) Terminate the plan of care upon client discharge. b) Reinforce the plan of care when each expected outcome is achieved. c) Terminate the plan if there are difficulties achieving the goals/outcomes. d) Continue the plan of care if more time is needed to achieve the goals/outcomes.

d) Continue the plan of care if more time is needed to achieve the goals/outcomes. The client's goals/outcomes sometimes are not met or partially met only because more time is needed for the plan of care to be effective.

A client comes into the clinic for a routine postoperative visit. While the nurse is assessing the level of pain, the client states that there is occasional discomfort but that pain levels have improved daily since returning home from the hospital. What should the nurse's response be regarding the client's plan of care? a) Suggest increasing the pain medication. b) Terminate the plan of care. c) Promptly modify the plan of care. d) Continue the plan of care.

d) Continue the plan of care.

A nurse who provides care in a campus medical clinic is conducting a follow-up assessment of a 21-year-old female student who recently contracted genital herpes. Which of the following actions best demonstrates that the nurse is attempting to facilitate the student's coping with her diagnosis? a) Teaching the student how to ensure that she does not infect future partners. b) Explaining the risks and benefits of daily suppressive therapy during outbreaks. c) Assessing the measures that the student takes to protect her overall health. d) Discussing ways that she could broach the subject of her diagnosis in future relationships.

d) Discussing ways that she could broach the subject of her diagnosis in future relationships. Discussing strategies that the student could use to tell others about her diagnosis in a clear but appropriate manner shows a recognition of the psychosocial ramifications of the student's diagnosis and promotes her coping with potential embarrassment. Discussing therapies, health promotion, and ways of protecting others are relevant, but these do not directly address the student's coping with her diagnosis.

The growth in home healthcare is largely attributed to which of the following? a) The inability of hospitals to care for increasing numbers of patients b) Nurses' desire to work in the community c) The nursing shortage in hospitals d) Early discharge of patients from the hospital setting

d) Early discharge of patients from the hospital setting Home healthcare is one of the most rapidly growing areas of the healthcare system. The prospective payment system of reimbursement encourages early discharge from the hospital and has created a new, acutely ill population that needs skilled nursing care at home.

The nurse recognizes the importance of including patients' families in assessment, care, decision-making, education, and discharge planning. When determining who constitutes a patient's family, which of the following criteria should the nurse prioritize? a) Interdependence b) Time commitment c) Genetic relationship d) Emotional bonding

d) Emotional bonding Family can be defined simply as any group of people who live together and are emotionally bonded. This consideration supersedes the significance of genetics, time commitment, or interdependence.

The nurse is admitting a client who is unable to identify person, place, or time. In order to properly analyze this data, what action must the nurse take? a) Determine the client's medical diagnosis for clarification. b) Assess the client's vital signs to determine the client's baseline. c) Ensure precautions are taken to prevent injury to the client. d) Interview the client's family to assess the client's usual level of consciousness.

d) Interview the client's family to assess the client's usual level of consciousness.

Mrs. Shields is a 46-year-old obese woman diagnosed with hypertension and type 2 diabetes. She tells the nurse that she knows she needs to lose weight. She recently visited her local fitness club, obtained a membership and has signed up for their next water aerobics class. According to the Transtheoretical Model of Change, what stage of change is Mrs. Shields in related to her weight loss? a) Precontemplation b) Maintenance c) Contemplation d) Preparation

d) Preparation

The care plan for a postoperative client includes a nursing diagnosis of "Risk for urinary retention." The nurse determines that the client has been voiding adequately. What is the nurse's most appropriate action? a) Continue to observe for urinary retention because of the client's postoperative status. b) Consult with the physician about the revision of the nursing diagnosis. c) Initiate a collaborative problem to address the client's changing status. d) Revise the nursing diagnosis because the client's status has changed.

d) Revise the nursing diagnosis because the client's status has changed.

A nurse enters the room of a patient with cancer. The patient is crying and states, "I feel so alone." Which statement is the most therapeutic? a) The nurse stands at the patient's bedside and states, "I understand how you feel. My mother said the same thing when she was ill." b) The nurse places a hand on the patient's arm and states, "You feel so alone." c) The nurse stands in the patient's room and asks, "Why do you feel so alone? Your wife has been here every day." d) The nurse holds the patient's hand and asks, "What makes you feel so alone?"

d) The nurse holds the patient's hand and asks, "What makes you feel so alone?"

The orderly progression of steps of the nursing process is: a) diagnosis, implementation, assessment, evaluation, and planning. b) planning, assessment, diagnosis, evaluation, and implementation. c) implementation, planning, evaluation, assessment, and diagnosis. d) assessment, diagnosis, planning, implementation, and evaluation.

d) assessment, diagnosis, planning, implementation, and evaluation. The nursing process is a systematic method that directs the nurse and patient and includes the following sequential steps: assessment, diagnosis, planning, implementation, and evaluation.

A patient rings the call bell to request pain medication. Upon performing the pain assessment, the nurse informs the patient that she will return with the pain medication. The nurse's promise to return with the pain medication is an example of which principle of bioethics? a) nonmaleficence b) autonomy c) justice d) fidelity

d) fidelity Fidelity is keeping one's promises and never abandoning a patient entrusted to your care without first providing for the patient's needs. Autonomy respects the rights of patients or their surrogates to make healthcare decisions. Nonmaleficence is avoiding causing harm. Justice involves giving each his or her due and acting fairly.

When admitting a patient to the hospital, the nurse may delegate some activities to other members of the healthcare team. Which activity would be appropriate to delegate? a) collecting information for a health history b) performing a physical assessment c) contacting the physician for medical orders d) preparing the bed ad collecting needed supplies

d) preparing the bed ad collecting needed supplies The nurse may delegate preparation of the bed and collection of needed supplies to unlicensed personnel but would perform the other activities listed.

Nurses focus on the whole client. Which statements acknowledge the client as a human being? Select all that apply. a) "Honey, can you tell me why you are crying?" b) "I am going to cleanse your back; turn over for me, sweetie." c) "Hey buddy, how was your night last night?" d) "I have your medications ready for you Ms. Jackson." e) "Hi Tom, please take your clothes off and put this gown on with the opening in the back." f) "Mr. Smith, I will be taking you to x-ray now."

d, f d) "I have your medications ready for you Ms. Jackson." f) "Mr. Smith, I will be taking you to x-ray now."

Professional Nursing Code of Ethics

-Set of guiding principles that all members of a profession accept. -Helps professional groups settle questions about practice or behavior. -Includes advocacy, responsibility, accountability, and confidentiality.

Access to care

-The number of uninsured in US 46.3 million in 2008. - Mostly women and children.

A nurse uses the following classic elements of evaluation when caring for patients. Which item below places them in their correct sequence? (1) Interpreting and summarizing findings (2) Collecting data to determine whether evaluative criteria and standards are met (3) Documenting one's judgment (4) Terminating, continuing, or modifying the plan (5) Identifying evaluative criteria and standards (what one is looking for when evaluating, e.g., expected patient outcomes)

5, 2, 1,3, 4 The five classic elements of evaluation in order are (1) identifying evaluative criteria and standards (what you are looking for when you evaluate, e.g., expected patient outcomes), (2) collecting data to determine whether these criteria and standards are met, (3) interpreting and summarizing findings, (4) documenting your judgment, and (5) terminating, continuing, or modifying the plan.

A

A 20-year-old patient diagnosed with an eating disorder has a nursing diagnosis of situational low self-esteem. Which of the following nursing interventions would be best to address self-esteem? A. Offer independent decision-making opportunities B. Review previously successful coping strategies C. Provide a quiet environment with minimal stimuli D. Support a dependent role throughout treatment

A

A depressed patient is crying and verbalizes feelings of low self-esteem and self-worth such as "I'm such a failure...I can't do anything right." The best nursing response would be to: A. Remain with the patient until he or she stops crying. B. Tell the patient that is not true and that every person has a purpose in life. C. Review recent behaviors or accomplishments that demonstrate skill ability. D. Reassure the patient that you know how he is feeling and that things will get better.

C

A patient diagnosed with major depressive disorder has a nursing diagnosis of chronic low self-esteem related to negative view of self. Which of the following would be the most appropriate cognitive intervention by the nurse? A. Promote active socialization with other patients B. Role play to increase assertiveness skills C. Focus on identifying strengths and accomplishments D. Encourage journaling of underlying feelings

nursing point of view

All patients in health care system interact with a nurse and some point. Nurses engage with patients over longer periods of time. Involved in intimate physical acts such as bathing, feeding etc, patients feel safer/more comfortable revealing info to nurses rather than physicians.

A patient has a diagnosis of pneumonia. Which entry should the nurse chart to help with financial reimbursement? a. Used incentive spirometer to encourage coughing and deep breathing. Lung congested upon auscultation in lower lobes bilaterally. Pulse ox 86%. Oxygen per nasal cannula applied at 2 L/min per standing order. b. Cooperative, patient coughed and deep breathed using a pillow as a splint. Stated, "felt better." Finally, patient had no complaints. c. Breathing without difficulty. Sitting up in bed watching TV. Had a good day. d. Status unchanged. Remains stable with no abnormal findings. Checked every 2 hours.

ANS: A Accurate documentation of supplies and equipment used assists in accurate and timely reimbursement. Your documentation clarifies the type of treatment a patient receives and supports reimbursement to the health care agency. None of the other options had equipment or supplies listed. Avoid using generalized, empty phrases such as "status unchanged" or "had a good day." Do not enter personal opinions—stating that the patient is cooperative is a personal opinion and should be avoided. "Finally, patient had no complaints" is a critical comment about the patient and if charted can be used as evidence of nonprofessional behavior or poor quality of care.

A nurse needs to begin discharge planning for a patient admitted with pneumonia and a congested cough. When is the best time the nurse should start discharge planning for this patient? a. Upon admission b. Right before discharge c. After the congestion is treated d. When the primary care provider writes the order

ANS: A Ideally, discharge planning begins at admission. Right before discharge is too late for discharge planning. After the congestion is treated is also too late for discharge planning. Usually the primary care provider writes the order too close to discharge, and nurses do not need an order to begin the teaching that will be needed for discharge. By identifying discharge needs early, nursing and other health care professionals can begin planning for home care, support services, and any equipment needs at home.

A nurse has taught the staff about informatics. Which statement indicates that the staff needs more education? a. If a nurse has computer competency, the nurse is competent in informatics. b. To be proficient in informatics, a nurse should be able to discover, retrieve, and use information in practice. c. A nurse needs to know how to acquire, critique, and apply scientific evidence from literature databases. d. Nursing informatics integrates nursing science, computer science, and information science to manage and communicate information in nursing practice.

ANS: A If the staff needs more education, then an incorrect statement is made. Competence in informatics is not the same as computer competency. To become competent in informatics, you need to be able to use evolving methods of discovering, retrieving, and using information in practice. This means that you learn to recognize when information is needed and have the skills to find, evaluate, and use that information effectively. For example, you need to know how to acquire, critique, and apply scientific evidence from literature databases. Nursing informatics is a specialty that integrates nursing science, computer science, and information science to manage and communicate data, information, and knowledge in nursing practice.

A new nurse asks the preceptor why a change-of-shift report is important since care is documented in the chart. What is the preceptor's best response? a. "A hand-off report provides an opportunity to share essential information to ensure patient safety and continuity of care." b. "A change-of-shift report provides the oncoming nurse with data to help set priorities and establish reimbursement costs." c. "A hand-off report provides an opportunity for the oncoming nurse to ask questions and determine research priorities." d. "A change-of-shift report provides important information to caregivers and develops relationships within the health care team."

ANS: A Properly performed, a hand-off report provides an opportunity to share essential information to ensure patient safety and continuity of care. Reimbursement costs and research priorities/opportunities are functions of the medical record. The purpose of the change-of-shift report is not to establish relationships but to ensure patient safety and continuity of care.

A nurse is charting on a patient's record. Which action is most accurate legally? a. Charts legibly b. States the patient is belligerent c. Uses correction fluid to correct error d. Writes entry for another nurse

ANS: A Record all entries legibly. Do not write personal opinions. Enter only objective and factual observations of patient's behavior; quote all patient comments. For example, patient refuses to cough and deep breathe, saying, "I don't care what you say, I will not do it." Do not erase, apply correction fluid, or scratch out errors made while recording. Chart only for yourself.

A nurse prepared an audiotaped exchange with another nurse of information about a patient. Which action did the nurse complete? The nurse completed a a. Report. b. Record. c. Consultation. d. Referral.

ANS: A Reports are oral, written, or audiotaped exchanges of information among caregivers. A patient's record or chart is a confidential, permanent legal document consisting of information relevant to his or her health care. Consultations are another form of discussion in which one professional caregiver gives formal advice about the care of a patient to another caregiver. Nurses document referrals (arrangements for the services of another care provider).

Which situation best indicates that the nurse has a good understanding regarding auditing and monitoring of patients' health records? a. The nurse determines the degree to which standards of care are met by reviewing patients' health records. b. The nurse realizes that care not documented in patients' health records still qualifies as care provided. c. The nurse knows that reimbursement is based on the diagnosis-related groups documented in patients' records. d. The nurse compares data in patients' records to determine whether a new treatment had better outcomes than the standard treatment.

ANS: A The patient record is a valuable source of data for all members of the health care team. Its purposes include communication, legal documentation, financial billing, education, research, and auditing/monitoring. The auditing/monitoring purpose involves nurses auditing records throughout the year to determine the degree to which standards of care are met and to identify areas needing improvement and staff development. The legal documentation purpose involves the concept that even though nursing care may have been excellent, in a court of law, "care not documented is care not provided." The financial billing or reimbursement purpose involves diagnosis-related groups (DRGs) as the basis for establishing reimbursement for patient care. For research purposes, the researcher compares the patient's recorded findings to determine whether the new method was more effective than the standard protocol. Analysis of data from research contributes to evidence-based nursing practice and quality health care.

9. A patient has been admitted to the hospital numerous times. The nurse asks the patient to share a personal story about the care that has been received. Which interaction is the nurse using? a. Narrative b. Socializing c. Nonjudgmental d. SBAR

ANS: A ANS: A In a therapeutic relationship, nurses often encourage patients to share personal stories. Sharing stories is called narrative interaction. Socializing is an important initial component of interpersonal communication. It helps people get to know one another and relax. It is easy, superficial, and not deeply personal. Nonjudgmental acceptance of the patient is an important characteristic of the relationship. Acceptance conveys a willingness to hear a message or acknowledge feelings; it is not a technique that involves personal stories. SBAR is a popular communication tool that helps standardize communication among health care providers. SBAR stands for Situation, Background, Assessment, and Recommendation

20. Which situation will cause the nurse to intervene and follow up on the nurse aide's behavior? a. The nurse aide is calling the older adult patient "honey." b. The nurse aide is facing the older adult patient when talking. c. The nurse aide cleans the older adult patient's glasses. d. The nurse aide allows time for the older adult patient to respond.

ANS: A Communicate with older adults on an adult level, and avoid patronizing or speaking in a condescending manner. Avoid terms of endearment such as "honey," "dear," "grandma," or "sweetheart." Facing an older adult patient, making sure the older adult has clean glasses, and allowing time to respond facilitate communication with older adult patients and should be encouraged, not stopped.

22. The staff is having a hard time getting an older adult patient to communicate. Which technique should the nurse suggest the staff use? a. Allow the patient to reminisce. b. Try changing topics often. c. Involve only the patient in conversations. d. Ask the patient for explanations.

ANS: A Encouraging older adults to share life stories and reminisce about the past has a therapeutic effect and increases their sense of well-being. Avoid sudden shifts from subject to subject. It is helpful to include the patient's family and friends and to become familiar with the patient's favorite topics for conversation. Asking for explanations is a nontherapeutic technique.

17. A nurse is using SOLER to facilitate active listening. Which technique should the nurse use for R? a. Relax b. Respect c. Reminisce d. Reassure

ANS: A In SOLER, the R stands for relax. It is important to communicate a sense of being relaxed and comfortable with the patient. Active listening enhances trust because the nurse communicates acceptance and respect for the patient, but it is not the R in SOLAR. Reminisce is a therapeutic communication technique, especially when used with the elderly. Reassuring can be therapeutic if the nurse reassures patients that there are many kinds of hope, and that meaning and personal growth can come from illness experiences. However, false reassurance can block communication.

1. Nurses who make the best communicators a. Develop critical thinking skills. b. Like different kinds of people. c. Learn effective psychomotor skills. d. Maintain perceptual biases.

ANS: A Nurses who develop critical thinking skills make the best communicators. Just liking people does not make an effective communicator because it is important to apply critical thinking standards to ensure sound effective communication. Just learning psychomotor skills does not ensure that the nurse will use those techniques and communication involves more than psychomotor skills. Critical thinking helps the nurse overcome perceptual biases or human tendencies that interfere with accurately perceiving and interpreting messages from others. Nurses who maintain perceptual biases do not make good communicators.

10. Before meeting the patient, a nurse talks to other caregivers about the patient. The nurse is in which phase of the helping relationship? a. Pre-interaction b. Orientation c. Working d. Termination

ANS: A The time before the nurse meets the patient is called the pre-interaction phase. This phase can involve such things as reviewing available data, including the medical and nursing history, talking to other caregivers who have information about the patient, or anticipating health concerns or issues that can arise

24. A patient is aphasic, and the nurse notices that the patient's hands shake intermittently. Which nursing action is most appropriate to facilitate communication? a. Use a picture board. b. Use pen and paper. c. Use an interpreter. d. Use a hearing aid.

ANS: A Using a pen and paper can be frustrating for a nonverbal (aphasic) patient whose handwriting is shaky; the nurse can revise the care plan to include use of a picture board instead. An interpreter is used for a patient who speaks a foreign language. A hearing aid is used for the hard of hearing, not for an aphasic patient.

Identify the purposes of a health care record. (Select all that apply.) a. Communication b. Legal documentation c. Reimbursement d. Education e. Research f. Nursing process

ANS: A, B, C, D, E The patient record is a valuable source of data for all members of the health care team. Its purposes include communication, legal documentation, financial billing (reimbursement), education, research, and auditing/monitoring. Nursing process is a way of thinking and performing nursing care; it is not a purpose of a health care record.

Which situation will require the nurse to obtain a telephone order? a. As the nurse and primary care provider leave a patient's room, the primary care provider gives the nurse an order. b. At 0100, a patient's blood pressure drops from 120/80 to 90/50 and the incision dressing is saturated with blood. c. At 0800, the nurse and primary care provider make rounds and the primary care provider tells the nurse a diet order. d. A nurse reads an order correctly as written by the primary care provider in the patient's medical record.

ANS: B A registered nurse makes a telephone report when significant events or changes in a patient's condition have occurred. Telephone orders and verbal orders usually occur at night or during emergencies. Because the time is 1AM (0100 military time) and the primary care provider is not present, the nurse will need to call the primary care provider for a telephone order. A verbal order (VO) involves the health care provider giving orders to a nurse while they are standing near each other. Just reading an order that is correctly written in the chart does not require a telephone order.

A nurse is writing nursing diagnoses for patients in a psychiatrist's office. Which nursing diagnoses are correctly written as two-part nursing diagnoses? (1) Ineffective Coping related to inability to maintain marriage (2) Defensive Coping related to loss of job and economic security (3) Altered Thought Processes related to panic state (4) Decisional Conflict related to placement of parent in a long-term care facility (1) and (2) (3) and (4) (1), (2), and (3) All of the above

All of the above

A nurse wants to integrate all pertinent patient information into one record, regardless of the number of times a patient enters the health care system. Which term should the nurse use to describe this system? a. Electronic medical record b. Electronic health record c. Electronic charting record d. Electronic problem record

ANS: B A unique feature of an electronic health record (EHR) is its ability to integrate all pertinent patient information into one record, regardless of the number of times a patient enters a health care system. Although the electronic medical record (EMR) contains patient data gathered in a health care setting at a specific time and place and is a part of the EHR, the two terms are frequently used interchangeably. There are no such terms as electronic charting record or electronic problem record.

A nurse is discussing the advantages of standardized documentation forms in the nursing information system. Which advantage should the nurse describe? a. Varied clinical databases b. Reduced errors of omission c. Increased hospital costs d. More time to read charts

ANS: B Advantages associated with the nursing information system include increased time to spend with patients (not more time to read charts); better access to information; enhanced quality of documentation; reduced errors of omission; reduced, not increased, hospital costs; increased nurse job satisfaction; compliance with requirements of accrediting agencies (e.g., TJC); and development of a common, not varied, clinical database.

A preceptor is working with a new nurse on documentation. Which situation will cause the preceptor to intervene? a. The new nurse uses a black ink pen to chart. b. The new nurse charts consecutively on every other line. c. The new nurse ends each entry with signature and title. d. The new nurse keeps the password secure.

ANS: B Chart consecutively, line by line (not every other line); if space is left, draw a line horizontally through it, and sign your name at the end. Every other line should not be left blank. Record all entries legibly and in black ink. End each entry with your signature and title. For computer documentation, keep your password to yourself. Using black ink, ending each entry with signature and title, and keeping the password secure are all appropriate behaviors.

13. A patient is being discharged home. Which information should the nurse include? a. Acuity level b. Community resources c. Standardized care plan d. Kardex

ANS: B Discharge documentation includes medications, diet, community resources, follow-up care, and whom to contact in case of an emergency or for questions. A patient's acuity level, usually determined by a computer program, is based on the type and number of nursing interventions (e.g., intravenous [IV] therapy, wound care, ambulation assistance) required over a 24-hour period. Acuity level can be used for staffing and billing. Some institutions use standardized care plans to make documentation more efficient. The plans, based on the institution's standards of nursing practice, are preprinted, established guidelines used to care for patients who have similar health problems. In some settings, a Kardex, a portable "flip-over" file or notebook, is kept at the nurses' station. Most Kardex forms have an activity and treatment section and a nursing care plan section, which organize information for quick reference.

A nurse is teaching the staff about health care reimbursement. Which information should the nurse include? a. Sentinel events help determine reimbursement issues for health care. b. Home health, long-term care, and hospital nurses' documentation can affect reimbursement for health care. c. A clinical information system must be installed by 2014 to obtain health care reimbursement. d. HIPAA is the basis for establishing reimbursement for health care.

ANS: B Nurses' documentation practices in home health, long-term care, and hospitals can determine reimbursement for health care. Sentinel events do not determine reimbursement. About 60% of the worst types of medical errors, called sentinel events (involving death or severe physical/psychological injury), relate to communication problems that often arise during telephone reports. A clinical information system (CIS) does not have to be installed by 2014 to obtain reimbursement. CIS programs include monitoring systems; order entry systems; and laboratory, radiology, and pharmacy systems. Diagnosis-related groups (DRGs) are the basis for establishing reimbursement for patient care, not HIPAA. Legislation to protect patient privacy regarding health information is the Health Insurance Portability and Accountability Act (HIPAA).

A nurse in a long-term care setting that is funded by Medicare and Medicaid is completing standardized protocols for assessment and care planning and for meeting quality improvement within and across facilities. Which task did the nurse just complete? a. A focused assessment/specific body system b. The Resident Assessment Instrument/Minimum Data Set c. An admission assessment and acuity level d. An intake assessment form and auditing phase

ANS: B You assess each resident in a long-term care agency receiving funding from Medicare and Medicaid programs using the Resident Assessment Instrument/Minimum Data Set (RAI/MDS). This documentation provides standardized protocols for assessment and care planning and a minimum data set to promote quality improvement within and across facilities. A focused assessment is limited to a specific body system. An admission assessment and acuity level is performed in the hospital. An intake assessment is for home health. There is no such thing as an auditing phase.

15. A patient has trouble speaking words, and the patient's speech is garbled. Which nursing diagnosis is most appropriate for this patient? a. Hopelessness b. Impaired verbal communication c. Hearing loss d. Self-care deficit

ANS: B A patient with impaired verbal communication has defining characteristics such as an inability to articulate words, inappropriate verbalization, difficulty forming words, and difficulty comprehending. Hopelessness implies that the patient has no hope for the future. Hearing loss is not a nursing diagnosis. Just because a patient has garbled speech does not mean that a hearing loss has occurred; a physical problem such as a stroke could cause the garbled speech. Self-care deficit does not apply in this situation because this usually relates to bathing, grooming, etc.

7. A smiling patient angrily states, "I will not cough and deep breathe." How will the nurse interpret this finding? a. The patient's personal space was violated. b. The patient's affect is inappropriate. c. The patient's vocabulary is poor. d. The patient's denotative meaning is wrong.

ANS: B ANS: B An inappropriate affect is a facial expression that does not match the content of a verbal message (e.g., smiling when describing a sad situation). The patient is smiling but is angry, which indicates an inappropriate affect. The patient's personal space was not violated. The patient's vocabulary is not poor. Individuals who use a common language share denotative meaning: baseball has the same meaning for everyone who speaks English, but code denotes cardiac arrest primarily to health care providers. The patient's denotative meaning is correct for cough and deep breathe.

21. A nurse obtained a telephone order from a primary care provider for a patient in pain. Which chart entry should the nurse document? a. 12/16/20XX 0915 Tylenol 3, 2 tablets, every 6 hours for incisional pain. VO Dr. Day/J. Winds, RN, read back. b. 12/16/20XX 0915 Tylenol 3, 2 tablets, every 6 hours for incisional pain. TO J. Winds, RN, read back. c. 12/16/20XX 0915 Tylenol 3, 2 tablets, every 6 hours for incisional pain. TO Dr. Day/J. Winds, RN, read back. d. 12/16/20XX 0915 Tylenol 3, 2 tablets, every 6 hours for incisional pain. TO J. Winds, RN.

ANS: C The nurse receiving a TO writes down the complete order or enters it into the computer as it is being given. Then he or she reads the order back to the health care provider, called read back, and receives confirmation from the person who gave the order that it is correct. An example follows: "10/16/2011: 0815, Tylenol 3, 2 tablets, every 6 hours for incisional pain. TO Dr. Knight/J. Woods, RN, read back." VO stands for verbal order, not telephone order. The doctor's name and read back must be included in the chart entry.

A nurse has provided care to a patient. Which entry should the nurse document in the patient's record? a. "Patient seems to be in pain and states, 'I feel uncomfortable.'" b. Status unchanged, doing well c. Left abdominal incision 1 inch in length without redness, drainage, or edema d. Patient is hard to care for and refuses all treatments and medications. Family present.

ANS: C Use of exact measurements establishes accuracy. Charting that an abdominal wound is "5 cm in length without redness, drainage, or edema" is more descriptive than "large wound healing well." Include objective data to support subjective data, so your charting is as descriptive as possible. Avoid using generalized, empty phrases such as "status unchanged" or "had a good day." It is essential to avoid the use of unnecessary words and irrelevant details or personal opinions. "Patient is hard to care for" is a personal opinion and should be avoided. It is also a critical comment that can be used as evidence for nonprofessional behavior or poor quality of care. Just chart, "Refuses all treatments and medications."

A nurse preceptor is working with a student nurse. Which behavior by the student nurse will require the nurse preceptor to intervene? a. The student nurse reviews the patient's medical record. b. The student nurse reads the patient's plan of care. c. The student nurse shares patient information with a friend. d. The student nurse documents medication administered to the patient.

ANS: C When you are a student in a clinical setting, confidentiality and compliance with the Health Insurance Portability and Accountability Act (HIPAA) are part of professional practice. When a student nurse shares patient information with a friend, confidentiality and HIPAA standards have been violated. You can review your patients' medical records only to seek information needed to provide safe and effective patient care. For example, when you are assigned to care for a patient, you need to review the patient's medical record and plan of care. You do not share this information with classmates and you do not access the medical records of other patients on the unit.

16. Which person is the best referral for a patient who speaks a foreign language? a. A family member b. A speech therapist c. An interpreter d. A mental health nurse specialist

ANS: C ANS: C Interpreters are often necessary for patients who speak a foreign language. A family member can lead to legal issues, speech therapists help patients with aphasia, and mental health nurse specialists help angry or highly anxious patients to communicate more effectively.

2. A nurse believes that the nurse-patient relationship is a partnership, and that both are equal participants. Which term should the nurse use to describe this belief? a. Critical thinking b. Authentic c. Mutuality d. Attend

ANS: C Effective interpersonal communication requires a sense of mutuality, a belief that the nurse-patient relationship is a partnership, and that both are equal participants. Critical thinking in nursing, based on established standards of nursing care and ethical standards, promotes effective communication and uses such standards as humility, self-confidence, independent attitude, and fairness. To be authentic (one's self) and to respond appropriately to the other person are important for interpersonal relationships but do not mean mutuality. Attending is giving all of your attention to the patient.

25. The nurse using critical thinking to enhance communication with patients is one who a. Shows sympathy appropriately. b. Uses automatic responses fluently. c. Self-examines personal communication skills. d. Demonstrates passive remarks accurately.

ANS: C Nurses who use critical thinking skills interpret messages received from others, analyze their content, make inferences about their meaning, evaluate their effects, explain rationales for communication techniques used, and self-examine personal communication skills. Sympathy is concern, sorrow, or pity felt for the patient and is nontherapeutic. Clichés and stereotyped remarks are automatic phrases that communicate that the nurse is not taking concerns seriously or responding thoughtfully. Passive responses serve to avoid conflict or to sidestep issues.

14. A patient was admitted 2 days ago with pneumonia and a history of angina. The patient is now having chest pain with a pulse rate of 108. Using the SBAR, which piece of data will the nurse use for B? a. Having chest pain b. Pulse rate of 108 c. History of angina d. Oxygen is needed.

ANS: C The B in SBAR stands for background information. The background information in this situation is the history of angina. Having chest pain is the Situation (S). Pulse rate of 108 is the Assessment (A). Oxygen is needed is the Recommendation (R).

12. A nurse and patient take action to meet health-related goals. The nurse is in which phase of the helping relationship? a. Pre-interaction b. Orientation c. Working d. Termination

ANS: C The working phase occurs when the nurse and the patient work together to solve problems and accomplish goals. Pre-interaction occurs before the nurse meets the patient. Orientation occurs when the nurse and the patient meet and get to know each other. Termination occurs during the ending of the relationship

1. Which critical thinking standards should the nurse use to ensure sound effective communication with patients? (Select all that apply.) a. Faith b. Supportiveness c. Self-confidence d. Humility e. Independent attitude f. Spiritual expression

ANS: C, D, E A self-confident attitude is important because the nurse who conveys confidence and comfort while communicating more readily establishes an interpersonal helping-trusting relationship. In addition, an independent attitude encourages the nurse to communicate with colleagues and share ideas about nursing interventions. An attitude of humility is necessary to recognize and communicate the need for more information before making a decision. Faith, supportiveness, and spiritual expression are attributes of caring, not critical thinking standards.

A nurse is creating a plan to reduce data entry errors and maintain confidentiality. Which guidelines should the nurse include? (Select all that apply.) a. Create a password with just letters. b. Bypass the firewall. c. Use a programmed speed-dial key when faxing. d. Implement an automatic sign-off. e. Impose disciplinary actions for inappropriate access. f. Shred papers containing personal health information (PHI).

ANS: C, D, E, F When faxing, use programmed speed-dial keys to eliminate the chance of a dialing error and misdirected information. An automatic sign-off is a safety mechanism that logs a user off the computer system after a specified period of inactivity. An automatic sign-off is used in most patient care areas and other departments that handle sensitive data. Disciplinary action, including loss of employment, occurs when nurses or other health care personnel inappropriately access patient information. All papers containing PHI (e.g., Social Security number, date of birth or age, patient's name or address) must be destroyed. Most agencies have shredders or locked receptacles for shredding and later incineration. Strong passwords use combinations of letters, numbers, and symbols that are difficult to guess. A firewall is a combination of hardware and software that protects private network resources (e.g., the information system of the hospital) from outside hackers, network damage, and theft or misuse of information and should not be bypassed.

After providing care, a nurse charts in the patient's record. Which entry should the nurse document? a. Appears restless when sitting in the chair b. Drank adequate amounts of water c. Apparently is asleep with eyes closed d. Skin pale and cool

ANS: D A factual record contains descriptive, objective information about what a nurse sees, hears, feels, and smells. An objective description is the result of direct observation and measurement. For example, "B/P 80/50, patient diaphoretic, heart rate 102 and regular." Avoid vague terms such as appears, seems, or apparently because these words suggest that you are stating an opinion, do not accurately communicate facts, and do not inform another caregiver of details regarding behaviors exhibited by the patient. Use of exact measurements establishes accuracy. For example, a description such as "Intake, 360 mL of water" is more accurate than "Patient drank an adequate amount of fluid."

A nurse is preparing a change-of-shift report for a patient who had chest pain. Which information is critical for the nurse to include? a. Pupils equal and reactive to light b. The family is a "pain" c. Had poor results from the pain medication d. Sharp pain of 8 on a scale of 1 to 10

ANS: D Elements in a change-of-shift report include identification of significant changes in measurable terms (e.g., pain scale) and by observation. Report elements do not include normal findings or routine information retrievable from other sources or derogatory or inappropriate comments about the patient or family, which could possibly lead to legal charges if overheard by the patient or family. This kind of language contributes to prejudicial opinions about the patient. Don't simply describe results as "good" or "poor." Be specific.

A nurse is using the source record and wants to find the daily weights. Where should the nurse look? a. Database b. Medical history and examination c. Progress notes d. Graphic sheet and flow sheet

ANS: D In a source record, the patient's chart has a separate section for each discipline (e.g., nursing, medicine, social work, respiratory therapy) in which to record data. Graphic sheets and flow sheets are records of repeated observations and measurements such as vital signs, daily weights, and intake and output. In the problem-oriented medical record, the database section contains all available assessment information pertaining to the patient (e.g., history and physical examination, the nurse's admission history and ongoing assessment, the dietitian's assessment, laboratory reports, radiologic test results). In the source record, the medical history and examination contain results of the initial examination performed by the physician, including findings, family history, confirmed diagnoses, and medical plan of care. In the source record, the progress notes contain an ongoing record of the patient's progress and response to medical therapy and a review of the disease process; it often is interdisciplinary and includes documentation from health-related disciplines (e.g., health care providers, physical therapy, social work).

A home health nurse is preparing for an initial home visit. Which information should be included in the patient's home care medical record? a. Nursing process form b. Step-by-step skills manual c. A list of possible procedures d. Reports to third party payers

ANS: D Information in the home care medical record includes patient assessment, referral and intake forms, interprofessional plan of care, a list of medications, and reports to third party payers. An interprofessional plan of care is used rather than a nursing process form. A step-by-step skills manual and a list of possible procedures are not included in the record.

Which entry will require follow-up by the nurse manager? 0800 Patient states, "Fell out of bed." Patient found lying by bed on the floor. Legs equal in length bilaterally with no distortion, pedal pulses strong, leg strength equal and strong, no bruising or bleeding. Neuro checks within normal limits. States, "Did not pass out." Assisted back to bed. Call bell within reach. Bed monitor on. ——————-Jane More, RN 0810 Notified primary care provider of patient's status. New orders received. ——————-Jane More, RN 0815 Portable x-ray of L hip taken in room. States, "I feel fine." ——————-Jane More, RN 0830 Incident report completed and placed on chart. ——————-Jane More, RN a. 0800 b. 0810 c. 0815 d. 0830

ANS: D Note that you do not include mention of the incident report in the patient's medical record. Instead you document in the patient's medical record an objective description of what happened, what you observed, and follow-up actions taken. It is important to evaluate and document the patient's response to the error or incident. Always contact the patient's health care provider whenever an incident happens.

A nurse wants to reduce data entry errors on the computer system. Which behavior should the nurse implement? a. Use the same password all the time. b. Share password with only one other staff member. c. Print out and review computer nursing notes at home. d. Chart on the computer immediately after care is provided.

ANS: D To increase accuracy and decrease unnecessary duplication, many health care agencies keep records or computers near a patient's bedside to facilitate immediate documentation of information as it is collected from a patient. A good system requires frequent and random changes in personal passwords to prevent unauthorized persons from tampering with records. When using a health care agency computer system, it is essential that you do not share your computer password with anyone under any circumstances. You destroy (e.g., shred) anything that is printed when the information is no longer needed. Taking nursing notes home is a violation of the Health Insurance Portability and Accountability Act (HIPAA) and confidentiality.

19. When making rounds, the nurse finds a patient who is not able to sleep because of surgery in the morning. Which therapeutic response is most appropriate? a. "It will be okay. Your surgeon will talk to you in the morning." b. "Why can't you sleep? You have the best surgeon in the hospital." c. "Don't worry. The surgeon ordered a sleeping pill to help you sleep." d. "It must be difficult not to know what the surgeon will find. What can I do to help?"

ANS: D "It must be difficult not to know what the surgeon will find. What can I do to help?" is using therapeutic communication techniques of empathy and offering of self. False reassurances ("It will be okay" and "Don't worry") tend to block communication. Patients frequently interpret "why" questions as accusations or think the nurse knows the reason and is simply testing them.

5. A nurse is standing beside the patient's bed. Nurse: How are you doing? Patient: I don't feel good. In this situation, which element is the feedback? a. Nurse b. Patient c. How are you doing? d. I don't feel good.

ANS: D ANS: D "I don't feel good" is the feedback because the feedback is the message the receiver returns. The sender is the person who encodes and delivers the message, and the receiver is the person who receives and decodes the message. The nurse is the sender. The patient is the receiver. "How are you doing?" is the message.

23. The patient that will cause the greatest communication concerns for a nurse is the patient who is a. Alert, has strong self-esteem, and is hungry. b. Oriented, pain free, and blind. c. Cooperative, depressed, and hard of hearing. d. Dyspneic, has a tracheostomy, and is anxious.

ANS: D Facial trauma, laryngeal cancer, or endotracheal intubation often prevents movement of air past vocal cords or mobility of the tongue, resulting in inability to articulate words. An extremely breathless person needs to use oxygen to breathe rather than speak. Persons with high anxiety are sometimes unable to perceive environmental stimuli or hear explanations. People who are alert, have strong self-esteem, and are cooperative and pain free do not cause communication concerns. Although hunger, blindness, and difficulty hearing can cause communication concerns, dyspnea, a tracheostomy, and anxiety all contribute to communication concerns.

4. Which technique will be most successful in ensuring effective communication? The nurse uses a. Interpersonal communication to change negative self-talk to positive self-talk. b. Small group communication to present information to an audience. c. Intrapersonal communication to build strong teams. d. Transpersonal communication to enhance meditation.

ANS: D Transpersonal communication is interaction that occurs within a person's spiritual domain. Many people use prayer, meditation, guided reflection, religious rituals, or other means to communicate with their "higher power." Interpersonal communication is one-on-one interaction between the nurse and another person that often occurs face to face. Meaningful interpersonal communication results in exchange of ideas, problem solving, expression of feelings, decision making, goal accomplishment, team building, and personal growth. Small group communication is interaction that occurs when a small number of persons meet. This type of communication is usually goal directed and requires an understanding of group dynamics. When nurses work on committees, lead patient support groups, form research teams, or participate in patient care conferences, they use a small group communication process. Intrapersonal communication is a powerful form of communication that occurs within an individual. For example, you improve your health and self-esteem through positive self-talk by replacing negative thoughts with positive assertions.

13. A nurse uses SBAR during hand-offs. The purpose of SBAR is to a. Use common courtesy. b. Establish trustworthiness. c. Promote autonomy. d. Standardize communication.

ANS: D When patients move from one nursing unit to another or from one provider to another, also known as hand-offs, a risk of miscommunication arises. Accurate communication is essential to prevent errors. SBAR is a popular communication tool that helps standardize communication among health care providers. Common courtesy is part of professional communication but is not the purpose of SBAR. Being trustworthy means helping others without hesitation. Autonomy is being self-directed and independent in accomplishing goals and advocating for others.

Which behaviors indicate that the student nurse has a good understanding of confidentiality and the Health Insurance Portability and Accountability Act (HIPAA)? (Select all that apply.) a. Writes the patient's room number and date of birth on a paper for school b. Prints/copies material from the patient's health record for a graded care plan c. Reviews assigned patient's record and another unassigned patient's record d. Reads the progress notes of assigned patient's record e. Gives a change-of-shift report to the oncoming nurse about the patient f. Discusses patient care with the hospital volunteer

ANS: D, E When you are a student in a clinical setting, confidentiality and compliance with HIPAA are part of professional practice. Reading the progress notes of an assigned patient's record and giving a change-of-shift report to the oncoming nurse about the patient are behaviors that follow HIPAA and confidentiality guidelines. Students and health care professionals may not discuss a patient's examination, observation, conversation, diagnosis, or treatment with other patients or staff not involved in the patient's care. To protect patient confidentiality, ensure that written materials used in your student clinical practice do not include patient identifiers (e.g., room number, date of birth, demographic information), and never print material from an electronic health record for personal use.

Fidelity

Agreement to keep promises

A

An adolescent attempting to adjust to the physical, emotional, and mental changes of increasing maturity. Internal sense of individuality, wholeness, and consistency of a person over time and in different situations. A. Identity B. Self-esteem stressors C. Role performance D. Role overload

A

An adult woman is recovering from a mastectomy for breast cancer and is frequently tearful when left alone. The nurse's approach should be based on an understanding of which of the following: A. Patients need support in dealing with the loss of a body part. B. The patient's family should take the lead role in providing support. C. The nurse should explain that breast tissue is not essential to life. D. The patient should focus on the cure of the cancer rather than loss of the breast.

B

An appropriate nursing diagnosis for an individual who experiences confusion in the mental picture of his physical appearance is: A. Acute confusion. B. Disturbed body image. C. Chronic low self-esteem. D. Situational low self-esteem.

Moral distress

Anguish experienced when a person feels unable to act according to closely held core values.

Nonmaleficence

Avoidance of harm or hurt.

Justice

Being fair, often used in for access to health care resources.

The nurse assesses a client's blood pressure, which was 160/90. Two hours following the administration of hydrochlorothiazide, the nurse reassesses the blood pressure at 140/78. What action has the nurse implemented?

a) Appraising b) Planning c) Evaluation d) Implementation CORRECT ANSWER: C

Which of the following steps in the nursing process is the careful taking of a history and a nursing examination?

a) Assessment b) Evaluation c) Nursing diagnosis d) Planning

Autonomy

Commitment to include patients in decisions. Freedom from external control.

C

Common in adolescents and employment situations: A. Body image B. Role conflict C. Role ambiguity D. Role strain

A client has had major abdominal surgery and just returned to the unit from the operating room. The nursing priority is to:

Correct Answer: Complete a postoperative assessment.

Utilitarianism

Proposes that the value of something is determined by its usefulness because its main emphasis is on the outcome or consequence of action. Consequentialism, teleology.

value formation

Development of values begins in childhood, shaped by experiences within the family unit. Schools, governments, religious traditions, and other social institutions influence in formation.

Ethics of care

Emphasizes the importance of understanding relationships, especially as they are revealed in personal narratives.

B

Example: a middle-aged woman with teenage children assuming responsibility for the care of her older parents: A. Body image B. Role conflict C. Role ambiguity D. Role strain

B

Example: perceived inability to meet parental expectations, harsh criticism, and inconsistent discipline: A. Identity B. Self-esteem stressors C. Role performance D. Role overload

D

Example: providing care to a family member with Alzheimer disease: A. Body image B. Role conflict C. Role ambiguity D. Role strain

D

Following a bilateral mastectomy, a 50-year-old patient refuses to eat, discourages visitors, and pays little attention to her appearance. One morning the nurse enters the room to see the patient with her hair combed and makeup applied. Which of the following is the best response from the nurse? A. "What's the special occasion?" B. "You must be feeling better today." C. "This is the first time I have seen you look this good." D. "I see that you've combed your hair and put on makeup."

Health Insurance Portability and Accountability Act

HIPAA, 1996. Mandates the protection of patients' personal health information.

B

In planning nursing care for an 85-year-old male, the most important basic need that must be met is: A. Assurance of sexual intimacy. B. Preservation of self-esteem. C. Expanded socialization. D. Increase in monthly income.

A

Includes physical appearance, structure, and function of the body. Examples: Amputation, facial disfigurement, or scars from burns: A. Body image B. Role conflict C. Role ambiguity D. Role strain

Care at end of life

Interventions unlikely to produce benefit to the patient. Acknowledgement of and respect for cultural differences.

Values

Personal beliefs about the worth of a given idea, attitude, custom, or object that set standards that influence behavior.

B

Several staff members complain about a patient's constant questions such as "Should I have a cup of coffee or a cup of tea?" and "Should I take a shower now or wait until later?" Which interpretation of the patient's behavior helps the nurses provide optimal care? A. Asking questions is attention-seeking behavior. B. Inability to make decisions reflects a self-concept issue. C. Dependence on staff must be stopped immediately. D. Indecisiveness is aimed at testing how the staff reacts.

C

Situational transitions; The way in which individuals perceive their ability to carry out significant roles: A. Identity B. Self-esteem stressors C. Role performance D. Role overload

D

The home health nurse is visiting a 90-year-old man who lives with his 89-year-old wife. He is legally blind and is 3 weeks' post right hip replacement. He ambulates with difficulty with a walker. He comments that he is saddened now that his wife has to do more for him and he is doing less for her. Which of the following is the priority nursing diagnosis? A. Self-care deficit, toileting B. Deficient knowledge regarding resources for the visually impaired C. Disturbed body image D. Risk for situational low self-esteem

Advocacy

Support of a particular cause, such as health, safety, rights of patient, privacy.

Beneficence

Taking positive actions to help others

B

The nurse asks the patient, "How do you feel about yourself?" The nurse is assessing the patient's: A. Identity. B. Self-esteem. C. Body image. D. Role performance.

A, B, C

The nurse can increase a patient's self-awareness through which of the following actions? (Select all that apply.) A. Helping the patient define her problems clearly B. Allowing the patient to openly explore thoughts and feelings C. Reframing the patient's thoughts and feelings in a more positive way D. Have family members assume more responsibility during times of stress

A new RN is being oriented to a nursing unit that is currently understaffed and is told that the UAPs have been trained to obtain the initial nursing assessment. What is the best response of the RN? a) Allow the UAPs to do the admission assessment and report the findings to the RN. b) Do his or her own admission assessments but don't interfere with the practice if other professional RNs seem comfortable with the practice. c) Tell the charge nurse that he or she chooses not to delegate the admission assessment until further clarification is received from administration. d) Contact his or her labor representative and complain about this practice.

The nurse should not delegate this nursing admission assessment because only nurses can perform this intervention. The nurse should seek clarification for this policy from the nursing administration.

Ethics

The study of conduct and character. It is concerned with determining what is good or valuable for individuals or society at large.

The nurse has entered a patient's room and observes that the patient is hunched over and appears to be breathing rapidaly. What type of question should the nurse first implement in this interaction? a) a reflective question b) a yes/no question c) a directing question d) an open-ended question

There are times when yes/no questions are appropriate. In this case, the nurse may want to ask, "Do you feel short of breath?" or something similar. Directing questions and reflective questions follow up on earlier communication. An open-ended question may elicit the necessary assessment data but a yes/no question accomplishes this goal more directly.

D

Unsuccessfully attempting to meet the demands of work and family while carving out some personal time: A. Identity B. Self-esteem stressors C. Role performance D. Role overload

Genetic screening

What are the risks and benefits to the individuals and to society of learning about the presence of disease that has not yet caused symptoms, or for which a cure is not yet available?

D

When caring for an 87-year-old patient, the nurse needs to understand that which of the following most directly influences the patient's current self-concept: A. Attitude and behaviors of relatives providing care B. Caring behaviors of the nurse and health care team C. Level of education, economic status, and living conditions D. Adjustment to role change, loss of loved ones, and physical energy

A

When developing an appropriate outcome for a 15-year-old girl, the nurse considers that a primary developmental task of adolescence is to: A. Form a sense of identity. B. Create intimate relationships. C. Separate from parents and live independently. D. Achieve positive self-esteem through experimentation.

A nurse is preparing a clinical outcome for a 32-year-old female runner who is recovering from a stroke that caused right-sided paresis. An example of this type of outcome is: a) After receiving 3 weeks of physical therapy, patient will demonstrate improved movement on the right side of her body. b) By 8/15/15, patient will be able to use right arm to dress, comb hair, and feed herself. c) Following physical therapy, patient will begin to gradually participate in walking/running events. d) By 8/15/15, patient will verbalize feeling sufficiently prepared to participate in running events.

a

A nurse on the unit fails to help a colleague ambulate a client even though she has time to do so. The nurse needing help should approach the nurse with which of the following? (Select all that apply) a) "This client is in need of our assistance and everyone who is free should come together for improved client outcomes" b) "Please come and help and work together as a team. You know that there is zero tolerance for selfish behavior" c) "Your behavior is unacceptable, we all have to work to ether as a team to provide quality care for our client" d) "Never mind, I will get someone else to help"

a) "This client is in need of our assistance and everyone who is free should come together for improved client outcomes" b) "Please come and help and work together as a team. You know that there is zero tolerance for selfish behavior" c) "Your behavior is unacceptable, we all have to work to ether as a team to provide quality care for our client"

A nurse has developed strong rapport with the wife of a client who has been receiving rehabilitation following a debilitating stroke. The wife has just been informed that her husband will be unlikely to return home and will require care that can only be provided in a facility with constant nursing care. The client's wife tells the nurse, "I can't believe it's come to this." How should the nurse best respond? a) "This must be very difficult for you to hear. How do you feel right now?" b) "What would help you accept that this is best for both of you?" c) "Why do you think that the care team has made this recommendation?" d) "Do you understand that everyone here has your husband's best interest at heart?"

a) "This must be very difficult for you to hear. How do you feel right now?"

A nurse has administered 1 unit of glucose to the client as per order. What is the correct documentation of this information? a) 1 Unit of glucose b) 1U of glucose c) One U of glucose d) 1 bottle of glucose

a) 1 Unit of glucose

The nurse participates in a quality assurance program and reviewing evaluation data from the previous year. Which of the following does the nurse recognize as an example of outcome evaluation? a) A 2% reduction in the number of repeat admissions for clients who underwent hip replacement surgery. b) A 4% increase in the number of baccalaureate prepared nurses are employed in the facility. c) A 97% bed occupancy rate in the critical care areas; 92% bed occupancy rate in the non-critical care areas. d) 98% of all hospital admissions had a nursing history completed within 24 hours of admission

a) A 2% reduction in the number of repeat admissions for clients who underwent hip replacement surgery. Quality assurance programs focus on three types of evaluation: structure, process, and outcome. Outcome evaluation focuses on measurable changes in the health status of clients, such as a 2% reduction in the number of repeat admissions for clients who underwent hip replacement surgery. Structure evaluation focuses on the environment in which care is provided, such as the number of baccalaureate-prepared nurses employed in the facility and bed occupancy rates. Process evaluation focuses on the nature and sequence of activities carried out by nursing implementing the nursing process, such as 98% of all hospital admissions had a nursing history completed within 24 hours of admission.

Which purpose best describes managed care as a framework for healthcare? a) A design to control the cost of care while maintaining the quality of care. b) Care coordination to maximize positive outcomes to contain costs. c) The delivery of services from initial contact through ongoing care. d) Based on a philosophy of ensuring death in comfort and dignity.

a) A design to control the cost of care while maintaining the quality of care. Managed care is a way of providing care, designed to control costs while maintaining the quality of care.

A nursing instructor is discussing differences between helping relationships and social relationships with a group of nursing students. Which statement is a characteristic of a helping relationship? a) A helping relationship is characterized by an unequal sharing of information. b) The person being helped in the helping relationship is accountable for the outcomes of the relationship. c) A helping relationship is built on the needs of the helping person. d) A helping relationship occurs spontaneously with random individuals.

a) A helping relationship is characterized by an unequal sharing of information. The patients shares information related to personal health problems, and the nurse shares information in terms of a professional role. The helping relationship does not occur spontaneously and occurs for a specific purpose, with a specific person, and for a specific period of time. The person providing the assistance is professionally accountable for the outcomes of the relationship, and the relationship is built on the needs of the person being helped.

A nurse is using the SOAP format of documentation to document care of a patient who is diagnosed with type 2 diabetes. Which source of information would be the nurse's focus when completing this documentation? a) A patient problem list b) Notes describing the patient's condition c) Overall trends in patient status d) Planned interventions and patient outcomes

a) A patient problem list When using the SOAP format, the problem list at the front of the chart alerts all caregivers to patient priorities. Narrative notes allow nurses to describe a condition, situation, or response in their own terms. Abnormal status can be seen immediately when using charting by exception, and planned interventions and patient expected outcomes are the focus of the case management model.

Which of the following hospital patients would likely need a formal discharge plan and referral to another agency? (Select all that apply) a) A patient who received a kidney transplant b) A patient who had a cholecystectomy (gallbladder removal) c) A patient whose sepsis required 3 weeks of care in the intensive care unit. d) A patient who is being discharged with a stage II pressure ulcer e) A patient who has been recently diagnosed with diabetes

a) A patient who received a kidney transplant c) A patient whose sepsis required 3 weeks of care in the intensive care unit. d) A patient who is being discharged with a stage II pressure ulcer e) A patient who has been recently diagnosed with diabetes Patients who have experienced major surgery or extensive hospital treatment or who have a recent diagnosis of a chronic condition require written discharge planning and likely a referral. Similarly, a patient whose wound will require care in the community will require these measures. Patients who have had a less major surgery, such as cholecystectomy, are less likely to have these requirements.

After many years of advanced practice nursing, a nurse has recently enrolled in a nurse practitioner program. This nurse has been attracted to the program by the potential to provide primary care for patients after graduation, an opportunity that is most likely to exist in which of the following settings? a) A rural health center b) A university hospital c) A community hospital d) A long-term care facility

a) A rural health center Many rural health centers employ few healthcare providers and primary care is often provided by a nurse practitioner (NP). A nurse practitioner may provide care in a long-term care facility or hospital, but in these setting, the NP is less likely to be the provider of primary care to patients.

The nurse recognizes that health problems that can be prevented by independent nursing interventions are called what? a) Actual or potential nursing diagnoses b) Syndrome nursing diagnoses c) Dependent nursing diagnoses d) Collaborative nursing diagnoses

a) Actual or potential nursing diagnoses

A nurse has developed a plan of care for an adult client. What is a nursing function that is important when using a nursing diagnosis to guide the care of this client?

a) Add a new nursing diagnosis in the nurse's own words to individualize the plan of care. b) Keep resolved nursing diagnoses as part of the plan of care in case the problem returns. c) Prioritize the nursing diagnoses. d) Do not allow the client to review his or her nursing diagnoses. CORRECT ANSWER: C

A nurse administers intravenous fluids to a client diagnosed with dehydration. After the fluids are completed the client's blood pressure is increased and pulse is decreased. During the final phase of the nursing process, what should the nurse do?

a) Administer an additional liter of intravenous fluids. b) Formulate a plan of care based on risk for dehydration. c) Check the client's skin turgor. d) Determine whether the prescribed treatment was effective. CORRECT ANSWER: D

A client states to the nurse, "I understand that I need a mastectomy for the treatment of my breast cancer, but I am fearful of learning about the drains I will need to empty." This is an example of what kind of learning? a) Affective learning b) Psychomotor learning c) Cognitive learning d) Behavioral learning

a) Affective learning

Which of the following phenomena underlie the present and predicted nursing shortage in the United States? Select all that apply. a) Aging of the American population b) Increased prevalence and incidence of chronic illnesses c) Decreased health literacy among Americans d) Increased average lengths of hospital stays e) Increased prevalence and incidence of disabilities f) Decreased enrollment in schools of nursing

a) Aging of the American population b) Increased prevalence and incidence of chronic illnesses e) Increased prevalence and incidence of disabilities The aging of the American population coupled with increases in both chronic illnesses and disabilities are expected to exacerbate the nursing shortage over the next decade. Hospital stays are shorter than in the past and this is not expected to change. Neither the health literacy of Americans nor enrollment in schools of nursing is predicted to significantly decline in the future.

When developing a nursing plan of care and associated client outcomes, the nurse recognizes which of the following? Select all that apply.

a) All plans of care are the same for each client with certain medical diagnosis. b) A plan of care should be comprehensive, including the initial, ongoing, and discharge planning. c) Outcomes can be short- and long-term. d) Only the client is involved in outcome setting, not the family. e) Outcome setting allows for individualization of the plan of care. CORRECT ANSWERS: B, C and E

Which of the following clinical events constitute areas of potential liability for the nurses involved? Select all that apply. a) An elderly patient develops skin breakdown on his coccyx because he was turned infrequently. b) A patient with no known cardiac history suffers an unwitnessed cardiac arrest and dies. c) An anemic patient experiences a febrile reaction to a transfusion of packed red blood cells. d) A patient experiences a seizure after a missed dose of his scheduled anticonvulsant. e) A confused patient experiences a fall because her bedrails were left in a lowered position.

a) An elderly patient develops skin breakdown on his coccyx because he was turned infrequently. d) A patient experiences a seizure after a missed dose of his scheduled anticonvulsant. e) A confused patient experiences a fall because her bedrails were left in a lowered position. Liability exists when the elements of duty, breach of duty, causation, and damages exist. Failure to turn an immobile patient, missing a dose of medication, and leaving a vulnerable patient's bedrails lowered all constitute breaches of nurses' duties that result in damages. Not every untoward event is evidence of liability; however, febrile blood reactions or unexpected cardiac arrests may occur without any inappropriate causation by care providers.

Which therapist is most likely to focus on teaching a patient to hold a toothbrush with an adaptive device and brush his teeth? a) An occupational therapist b) A physical therapist c) A speech therapist d) A respiratory therapist

a) An occupational therapist An occupational therapist assists physically challenged patients to adapt to limitations and use a variety of adaptive devices and strategies to aid patients in carrying out the activities of daily living. The occupational therapist is the healthcare professional who will assist the patient in using an adaptive device to brush his teeth. A speech therapist assists patients with speaking and swallowing problems. A physical therapist seeks to restore function or prevent further disability after an injury or illness. A respiratory therapist is trained in techniques that improve lung function and oxygenation.

Carl Rogers (1961) studied the process of therapeutic communication. Through his research, the elements of a "helpful" person were described. They include all of the following except which choice? a) Analysis b) Empathy c) Positive regard d) Comfortable sense of self

a) Analysis

The nurse is caring for a client that presents with polydipsia, polyphagia, and polyuria. The clients labs reveal in increased Hgb A1C, which could indicate increased blood glucose levels. What is the next step for the nurse to take based on the nursing process?

a) Analyze data and create an individualized nursing diagnosis. b) Identify outcomes for the client with his or her input. c) Follow-up with the client to see if the lab work improves. d) Administer a prescribed medication to decrease the client's blood sugar. CORRECT ANSWER: A

A nurse is taking care of a 15-year-old man with cystic fibrosis. The nurse is at the start of her shift and she goes in to the client's room to introduce herself and perform a safety check. The nurse notices that the client is receiving IV fluids with potassium. When the nurse double checks to see if this is what he is supposed to be on, she notices that these fluids were supposed to have been stopped 32 hours ago. What should the nurse not do in this situation? a) Attach a copy of the incident report to the chart. b) Stop the infusion and document the time. c) Report error to primary provider. d) Fill out an incident report.

a) Attach a copy of the incident report to the chart. For legal reasons the nurse should not attach a copy of the incident report to the chart. The nurse should, however, fill out an incident report, stop the infusion and document the time, and report the error to the primary provider.

A nurse working with patients in the community is aware that which of the following is a true statement related to environmental factors in a community? a) Barriers to accessing healthcare within a community may include lack of transportation. b) Lack of health insurance is a negative environmental factor affecting one's access to healthcare. c) The quality of air and water are relatively consistent when comparing urban and rural environments. d) Environmental factors focus on the harmful effects on an individual's health.

a) Barriers to accessing healthcare within a community may include lack of transportation: Environmental barriers to accessing healthcare within a community include lack of transportation, distance to services, and location of services.

A nurse is attempting to improve care on the pediatric ward of a hospital. Which nursing improvements might the nurse employ when following the recommendations of the Institute of Medicine's Committee on Quality of Health Care in America? Select all that apply. a) Basing patient care on continuous healing relationships b) Customizing care to reflect the competencies of the staff c) Using evidence-based decision making d) Having a charge nurse as the source of control e) Using safety as a system priority f) Recognizing the need for secrecy to protect patient privacy

a) Basing patient care on continuous healing relationships c) Using evidence-based decision making e) Using safety as a system priority

A nurse is evaluating the plan of care for a client and determines that the achievement of goals is difficult to evaluate. Which of the following might the nurse do in evaluating the plan to see that the outcomes are correctly written? Select all that apply. a) Be certain that the subject is the client or some part of the client. b) See if the client's expected behavior is written in observable, measurable terms, c) Rewrite the plan of care so that the client meets the expected outcomes. d) Be sure that the criteria for appropriate response are clearly specified. e) Specify time limits in the plan.

a) Be certain that the subject is the client or some part of the client. b) See if the client's expected behavior is written in observable, measurable terms, d) Be sure that the criteria for appropriate response are clearly specified. e) Specify time limits in the plan.

The nurse is caring for a 60-year-old client with an improper bowel movement regimen. Which of the following is the most appropriate method the nurse should use to conduct new learning? a) Begin the session with a reference to the client's actual experience. b) Create small groups and facilitate group discussions. c) Begin the session with identifying each learner by his/her name. d) Talk to the client's relatives and get a detailed account of the client's history.

a) Begin the session with a reference to the client's actual experience. Beginning the session with a reference to the client's actual experience will help provide a link to which the new learning can connect. Beginning the session by identifying each learner by his/her name, creating small groups and facilitating group discussions, and talking to the client's relatives and getting a detailed client history help the nurse to know more about the client; however, these are not the most appropriate methods to connect new learning.

Nurses formulate different types of goals for clients when planning client care. What is considered a psychomotor client goal? a) By 8/18/15, client will demonstrate improved motion in left arm. b) By 8/18/15, client will list three foods that are low in salt. c) By 8/18/15, client will learn three exercises designed to strengthen leg muscles. d) By 8/18/15, client will value his health sufficiently to quit smoking.

a) By 8/18/15, client will demonstrate improved motion in left arm.

A nurse has been doing discharge teaching to a client with diabetes mellitus type 1. Which of the following outcomes indicates that the teaching has been effective? a) By a certain date, client will verbalize signs and symptoms of hypoglycemia. b) By a certain date, client will talk to a dietician regarding information for a diabetic diet. c) Before discharge, client will understand proper foot care and eye care. d) Before discharge, client will attempt to administer a subcutaneous injection.

a) By a certain date, client will verbalize signs and symptoms of hypoglycemia.

A nurse follows the guidelines for a healthy lifestyle. How can this promote health in others? a) By being a role model for healthy behaviors. b) By not requiring sick days from work. c) By never exposing others to any type of illness. d) By spending less money on food.

a) By being a role model for healthy behaviors. Good personal health enables the nurse to serve as a role model for patients and families.

The student nurse has been assigned to a pediatric hospital floor next week. The student understands that he or she is expected to be able to use the syringe pump with the clinical instructor when giving medications. The student has never used this pump before and is anxious. What is the most appropriate way for the student to lessen the anxiety associated with the clinical rotation?

a) Call other students and ask them about the equipment in order to become more familiar with the procedure. b) Practice using the pump in the lab setting if it is available and with instructor permission. c) Don't stress about it, wait until the day of the rotation and inform the instructor that you do not know how to use the equipment. d) Attempt to use the equipment even without practice, eventually it can be figured out. CORRECT ANSWER: B

The nurse is caring for a client with a nursing diagnosis of deficient fluid volume. The nurse has implemented the plan of care and upon evaluation finds that the client continues to exhibit symptoms of deficient fluid volume. What should the nurse do next?

a) Change the nursing diagnosis because the client's problem was falsely identified. b) Add additional nursing diagnosis to meet the client's health needs. c) Modify the plan of care and interventions to meet the client's needs. d) Reassess the client for more symptoms of deficient fluid volume. CORRECT ANSWER: C

Which of the following nursing actions is most dependent on technical skill? a) Changing a patient's wound dressing while maintaining asepsis. b) Creating rapport with a group of homeless youth who have high-risk lifestyles. c) Determing the Glasgow Coma Scale score for a patient who has a head injury. d) Identifying the learning needs of a woman who is slated for a bilateral mastectomy.

a) Changing a patient's wound dressing while maintaining asepsis. Technical skill is exhibited by the skillful manipulation of equipment and hand-eye coordination. Performing a kinesthetic task such as doing a dressing change with appropriate technique requires such skill. Identifying learning needs and creating rapport with individuals are examples of interpersonal skills. Assessing a patient according to standardized criteria requires the cognitive skill to know and apply the instrument in question.

A nurse is part of a team that will be working in a new orthopedic unit to determine the most appropriate method for documentation. The team agrees to initiate the practice of an abbreviated form of documentation that requires less nursing time and readily detects changes in client status. Which documentation method would the group most likely suggest? a) Charting by exception b) Problem, intervention, and evaluation note c) Narrative notes d) FOCUS data, action, and response note

a) Charting by exception The team would most likely suggest the use of charting by exception, which is an abbreviated form of documentation.

While working as part of an interdisciplinary group developing a client's plan of care, a nurse asks the question, "Can you give me an example?" The nurse is demonstrating which standard for judging thinking?

a) Clarity b) Relevance c) Accuracy d) Precision CORRECT ANSWER: A

When interacting with a patient, the nurse answers, "I am sure everything will be fine. You have nothing to worry about." This is an example of what type of inappropriate communication technique? a) Cliché b) Giving advice c) Being judgmental d) Changing the subject

a) Cliché Telling a patient that everything is going to be all right is a cliché. This statement gives false assurance and gives the patient the impression that the nurse is not interested in the patient's condition.

A nurse writes the following outcome for a patient who is trying to lose weight: "The patient can explain the relationship between weight loss, increased exercise, and decreased calorie intake." This is an example of what type of outcome? a) Cognitive b) Psychomotor c) Affective d) Physical changes

a) Cognitive Cognitive outcomes involve increases in patient knowledge;

The nurse has educated the client on the pathophysiology of osteoarthritis and degenerative joint disease. This is an example of what learning theory? a) Cognitive learning theory b) Behavioral learning theory c) Adaptive learning theory d) Developmental learning theory

a) Cognitive learning theory

What ensures continuity of care? a) Communication b) Critical thinking c) Reassessment d) Integration

a) Communication

To determine the significance of a blood pressure reading of 148/100, it is first necessary for the nurse to: a) Compare this reading to standards. b) Check the taxonomy of nursing diagnoses for a pertinent label. c) Check a medical text for the signs and symptoms of high blood pressure. d) Consult with colleagues.

a) Compare this reading to standards. A standard, or a norm, is a generally accepted rule, measure, pattern, or model to which data can be compared in the same class or category. For example, when determining the significance of a patient's blood pressure reading, appropriate standards include normative values for the patient's age group, race, and illness category. Deviation from an appropriate norm may be the basis for writing a diagnosis.

What role will the nurse have when admitting a patient to a hospital for outpatient surgery that will result in discharge the same day? a) Complete regular admission procedures b) Prepare for long-term care needs c) Provide detailed information on the procedure d) Schedule the patient for screening tests

a) Complete regular admission procedures Patients entering the hospital setting for outpatient surgery have regular admission procedures conducted by the nurse. Scheduling of screening tests and initial teaching is completed in the days prior to the surgery. Same-day surgery and discharge may require community-based follow-up, but it generally does not require long-term care. Detailed information on the procedure will be provided by the physician performing the procedure.

A newly graduated nurse is unable to determine the significance of data obtained during an assessment. What would be the nurse's most appropriate action? a) Consult with a more experienced nurse. b) Contact the client's health care provider. c) Document the data for future reference. d) Continue to collect assessment data.

a) Consult with a more experienced nurse.

The nurse formulates the following nursing diagnosis: Disturbed Body Image related to decreased ability to cope with surgical removal of right breast. AEB client refuses to look at surgical site and client statement, "I'm ugly. My husband will no longer find me desirable." Which of the following is the etiology? a) Decreased ability to cope with surgical removal of right breast b) Disturbed body image c) "I'm ugly. My husband will no longer find me desirable." d) Refusal of client to look at surgical site

a) Decreased ability to cope with surgical removal of right breast The etiology identifies the factors that contribute to the unhealthy client response or problem. Disturbed Body Image identifies what is unhealthy about the client, indicating the need for change. The client's statements and refusal to look at the surgical site are defining characteristics that validate the existence of the problem.

The parents of a school-age child are meeting with the nurse for health promotional education for their child. The child has the following assessment data: 7-year-old male, diabetes mellitus type 1 with a hemoglobin A1C of 8.3%, BMI of 31.7, BMI percentile of 99. What are the most appropriate learning diagnoses for this first session? a) Deficient Knowledge: Imbalanced nutrition: more than body requirements, and ineffective health maintenance. b) Deficient Knowledge: Risk for chronic low self esteem, and risk for unstable blood glucose level. c) Deficient Knowledge: Risk for imbalanced nutrition: more than body requirements, and sedentary life style. d) Deficient Knowledge: Readiness for enhanced nutrition, and risk for disturbed body image.

a) Deficient Knowledge: Imbalanced nutrition: more than body requirements, and ineffective health maintenance.

A nurse is caring for a client admitted with dehydration after completing a triathlon in a hot, dry climate. The nurse identifies an appropriate nursing diagnosis for this client as "Deficient Fluid Volume related to insufficient fluid intake as evidenced by blood pressure 84/46, heart rate 145, concentrated urine, and client stating that he drank 200 mL of water during the 4-hour event." Identify the problem statement in this nursing diagnosis. a) Deficient fluid volume b) Insufficient fluid intake c) Hot, dry climate d) Blood pressure 84/46, heart rate 145, concentrated urine, and tented skin turgor

a) Deficient fluid volume The problem statement is "Deficient Fluid Volume." "Insufficient fluid intake" is the etiology in this nursing diagnosis. Defining characteristics include "blood pressure 84/46, heart rate 145, concentrated urine, and client stating that he drank 200 mL of water during the 4-hour event." The phrase "hot, dry climate" is not a component of this nursing diagnosis statement.

A nurse is preparing to teach a 45-year-old male patient with asthma how to use his inhaler. Which teaching tool is one of the best methods to teach the patient this skill? a) Demonstration b) Lecture c) Discovery d) Panel session

a) Demonstration Demonstration of techniques, procedures, exercises, and the use of special equipment is an effective patient teaching strategy for a skill. Lecture can be used to deliver information to a large group of patients but is more effective when the session is interactive; it is rarely used for individual instruction, except in combination with other strategies. Discovery is a good method for teaching problem-solving techniques and independent thinking. Panel discussions can be used to impart factual material but are also effective for sharing experiences and emotions.

A nurse caring for a patient who is hospitalized following a double mastectomy is preparing a discharge plan for the patient. Which action should be the focus of this termination phase of the helping relationship? a) Determining the progress made in achieving established goals b) Clarifying when the patient should take medications c) Reporting the progress made in teaching to the staff d) Including all family members in the teaching session

a) Determining the progress made in achieving established goals The termination phase occurs when the conclusion of the initial agreement is acknowledged. Discharge planning coordinates with the termination phase of a helping relationship. The nurse should determine the progress made in achieving the goals related to the patient's care.

The nurse manager on an orthopedic unit has determined that the nurses are not keeping the nursing diagnoses up-to-date on client care plans and, in turn, are not using the plan of care. What is a feasible approach to correcting this problem? a) Develop a process for periodic review of care plans that focuses on deleting and updating the nursing diagnoses. b) Request that a staff development nurse instruct the nurses on concept mapping to use instead of care planning. c) Delegate the updating of nursing diagnoses for all clients on the unit to one nurse for each shift. d) Provide an in-service on interviewing and physical assessment skills; discuss the importance of these skills with the staff.

a) Develop a process for periodic review of care plans that focuses on deleting and updating the nursing diagnoses.

What interpersonal skill is displayed by a nurse who is attentive and responsive to the health care needs of individual patients and ensures the continuity of care when leaving the patient?

a) Developing accountability b) Establishing caring relationships c) Enjoying the rewards of mutual interchange d) Developing ethical/legal skills CORRECT ANSWER: A

The nurse has measured from the tip of the client's nose to his earlobe and then down to the xiphoid process before inserting a nasogastric (NG) tube and attaching it to low suction. Which component of the nursing process has the nurse demonstrated?

a) Diagnosing; implementing b) Implementing; evaluation c) Planning; implementing d) Assessing; diagnosing CORRECT ANSWER: C

A nurse is involved in selecting the most appropriate nursing diagnoses for a client. Which technique would the nurse most likely use? Select all that apply.

a) Diagnostic validation b) Cluster interpretation c) Interviewing d) Inspection e) Cue clustering CORRECT ANSWERS: A, B and E

During the initial assessment of a 30-year-old patient who has sought care for the first time, the nurse notes that the patient smokes cigarettes regularly. Which of the following nursing actions best demonstrates the principles of primary health promotion? a) Dialoguing with the patient about his or her history of smoking and reasons for smoking and discussing the health implications. b) Performing auscultation and percussion of the patient's anterior and posterior chest and measuring the patient's oxygen saturation. c) Teaching the patient about the relationship between smoking and lung cancer and chronic obstructive pulmonary disease (COPD). d) Referring the patient to a respiratory therapist for lung function testing.

a) Dialoguing with the patient about his or her history of smoking and reasons for smoking and discussing the health implications. Health promotion often involves discussing the nature and contributing factors towards a health behavior and then addressing the implications and potential consequences of these. A discussion of underlying factors should ideally precede any teaching about the health consequences. Assessment and diagnostic testing are associated with secondary prevention.

The nurse has developed a strong therapeutic partnership with a 44-year-old electrician who suffered severe burns while working on an industrial site. Which of the nurse's following actions most directly addresses the patient's self-actualization needs? a) Discussing the patient's strengths and dialoguing with him about his body image. b) Encouraging the patient to talk about his previous accomplishments and his goals for the future. c) Reorganization of his care and facilitating a day pass so the patient can spend Thanksgiving with his family. d) Encouraging the patient's friends and family to take an active role in his care at the hospital.

a) Discussing the patient's strengths and dialoguing with him about his body image. Aspects of self-actualization include focusing on patient's strengths and fostering a positive body image. Addressing accomplishments and goals is likely to meet patients' self-esteem needs. Facilitating contact and connection between patients and their families is an action that promotes love and belonging needs.

What is meant by intellectual and affective activities in which individuals engage to explore their experiences in order to lead to new meanings and appreciations?

a) Reminiscing b) Memorization c) Evangelization d) Reflection CORRECT ANSWER: D

A nurse is forming an education plan for a client who is being discharged from the nursing unit after cardiac catheterization. Which diagnosis and intervention are most appropriate for this client? a) Knowledge Deficient: Risk for altered perfusion secondary to re-occlusion b) Knowledge Deficient: Impaired mobility related to lying flat for 8 hours c) Knowledge Deficient: Altered urinary output related to catheterization d) Knowledge Deficient: Risk for ineffective breathing pattern related to incisional pain

a) Knowledge Deficient: Risk for altered perfusion secondary to re-occlusion Urinary output should not be changed from a cardiac catheterization. The incision for this procedure may require 8 hours of lying flat, but mobility returns to baseline before discharge home. The risk for ineffective breathing pattern would not be due to incisional pain, which would be in the groin or elbow. Educating the client to be aware for the safety issue of chest pain resulting from the newly opened coronary arteries becoming re-occluded and blocking blood flow to the heart is the highest priority focus.

The nurse who is caring for a child admitted after an automobile accident recognizes the importance of including the child's family in the plan of care. Inclusion of the family meets which of Maslow's basic human needs? a) Love and belonging. b) Physiologic. c) Self-actualization. d) Self-esteem

a) Love and Belonging: Love and belonging needs include the understanding and acceptance of others in both giving and receiving love, and the feeling of belonging to families, peers, friends, a neighborhood, and a community. The inclusion of family and friends in the care of a patient is a nursing intervention to meet this need.

The nurse is caring for Mr. H., a 35-year-old man who is hospitalized following a motorcycle accident. He has a traumatic brain injury. The nurse is working with Mr. H. on self-care behaviors. The following would help the nurse to assess the success of the nursing interventions except which of the following? a) Model self-care behaviors for the client. b) Collect data on the number of self-care activities performed that day. c) Check with the client to ensure personal goals are met. d) Ask client to discuss his goals for the day at the start of the shift.

a) Model self-care behaviors for the client.

The nurse is caring for a client who is postoperative and has pain that is an 8 on a scale of 0-10. There is an order for intravenous (IV) pain medication every 4 hours PRN. The nurse administers the prescribed pain medication to the client. What should the nurse do next to assist in meeting this client's desired outcome of a pain scale score less than 4 on a scale of 0-10?

a) Modify the plan of care to include an additional pain medication because the client's pain is so severe. b) Call the surgeon to report the pain level. c) Evaluate the client's pain level after the appropriate amount of time has elapsed for the pain medication to take effect. d) Continue making rounds on her other clients and let the client rest. CORRECT ANSWER: C

A new nurse who is being oriented to the subacute care unit is expected to follow existing standards when providing patient care. Which nursing actions are examples of these standards? Select all that apply. a) Monitoring patient status every hour b) Using intuition to troubleshoot patient problems c) Turning a patient on bed rest every 2 hours d) Becoming a nurse mentor to a student nurse e) Administering pain medication ordered by the physician f) Becoming involved in community nursing events

a) Monitoring patient status every hour c) Turning a patient on bed rest every 2 hours e) Administering pain medication ordered by the physician

If you wanted to find a list of the violations that can result in disciplinary actions against a nurse, you should read which of the following? a) Nurse Practice Act b) Code of Ethics for Nurses c) Nurses' Bill of Rights d) American Journal of Nursing

a) Nurse Practice Act Each stated has a Nurse Practice Act that protects the public by broadly defining the legal scope of nursing practice. Practicing beyond those limits makes you vulnerable to charges of violating the state Nurse Practice Act. Nurse Practice Acts list the violations that can result in disciplinary actions against a nurse and also serve to exclude untrained or unlicensed people from practicing nursing.

A nurse enters the client's room and introduces himself stating, "Hello, Mr. Alonso. My name is Anthony Bader. I will be your registered nurse today. I will be providing your nursing care and I will be with you until 3:30 PM. If you need anything, please call me on my phone or put your light on." He then gives the client a printed card with this information. In the helping relationship, what does this represent? a) Orientation phase b) Intimate phase c) Termination phase d) Working phase

a) Orientation phase

A nurse demonstrates clinical reasoning during which phase of the nursing process? Select all that apply.

a) Outcome identification b) Planning c) Diagnosis d) Implementation e) Evaluation f) Assessment

A nurse sees a patient walking to the bathroom with a stooped gait, facial grimacing, and gasping sounds. It is important that the nurse assess the patient for: a) Pain b) Anxiety c) Depression d) Fluid volume deficit

a) Pain

The nurse recognizes that the most appropriate example of the assessment phase of the nursing process is which of the following?

a) Palpating a mass in the right lower quadrant of the abdomen b) Documenting the administration of a medication provided for pain c) Evaluating the temperature of a client given medication for a fever d) Including a nursing diagnosis of Acute Pain in the client's plan of care CORRECT ANSWER: A

Which of the following would a nurse know is a part of an evaluative statement? Select all that apply. a) Patient data that supports how the outcome was met b) Name of client's physician c) Client's health history d) Description of how the patient outcome was met

a) Patient data that supports how the outcome was met d) Description of how the patient outcome was met

Nurses are involved in many types of evaluation. All of the following are activities that are related to evaluation, but which of the following is the priority concern for nurses? a) Patients and their care b) Helping targeted groups of patients to achieve their specific outcomes c) Measuring the competence of individual nurses d) Measuring patient outcome achievement

a) Patients and their care

Nurse Mayweather is auscultating lung sounds. She notes crackles in the LLL which were not present at the start of the shift. Nurse Mayweather is engaged in which type of nursing intervention? a) Maintenance intervention b) Surveillance intervention c) Educational intervention d) Psychomotor intervention

b) Surveillance intervention

Which of the following principles underlies the ethical conduction of research in a hospital setting? a) Patients must grant informed consent if they are to participate. b) The patient must directly and personally benefit from the research. c) Descriptive studies are more ethical than experimental studies. d) Any interventions must be approved by the Food and Drug Administration (FDA).

a) Patients must grant informed consent if they are to participate. Informed consent is an absolute prerequisite for patients who are asked to participate as subjects in a research study. Many interventions in healthcare settings lie beyond the scope of the FDA. The risks and benefits of research are considered carefully in light or ethical principles, but this does not necessarily mean that every participant in a study stands to benefit from it. Ethical standards are applicable and achievable in every type of research, and descriptive studies are not necessarily more ethical than experiments.

A group of nurses of the orthopedic floor of a hospital wish to improve their clinical performance. The nurse manager suggests a program in which the nurses will evaluate each other and provide feedback for improved performance. The nurses know that this program is termed which of the following? a) Peer review b) Quality and Safety Educatin for Nurses (QSEN) c) Hospital Consumer Assessment of Healthcare Providers and Systems (HCAHPS) d) American Association of Critical-Care Nurses (AACN)

a) Peer review

The nurse has arranged for a stroke patient to participate in a daily group rehabilitation program that aims to improve the mobility, independence, and activities of daily living (ADLs) of participants. This program is likely to address which of the following needs? Select all that apply. a) Physiologic needs b) Safety and security needs c) Love and belonging needs d) Self- esteem needs e) Self-actualization needs

a) Physiologic needs b) Safety and security needs c) Love and belonging needs d) Self-esteem needs e) Self-actualization needs A rehabilitation program is likely to be multidimensional, addressing the physiology of the patient's movement while prioritizing and ensuring safety. Improving independence and ADLs is likely to benefit the patient's self-esteem and self-actualization after the effects of stroke. The group format of the program is likely to provide some measure of belonging with those who have had similar experiences, thus preventing isolation.

A school nurse is discussing bike and outdoor safety measures with a group of Boy Scouts. What type of health education and counseling is the nurse providing to this group of children? a) Preventing illness b) Restoring health c) Promoting health d) Facilitating coping

a) Preventing illness

A nurse is maintaining a problem-oriented medical record for a client. Which of the following components of the record describes the client's responses to what has been done and revisions to the initial plan? a) Progress notes b) Plan of care c) Data base d) Problem list

a) Progress notes

What role will the nurse play in transferring a patient to a long-term care facility? a) Provide a verbal report to the nurse at the long-term care facility on the patient, the hospital care, and the patient's current condition. b) Inform that patient that transferring should be a stress-free situation. c) Arrange for the patient's belongings to remain at the hospital until discharge from the long-term care facility. d) Assure that the patient's original chart accompanies the patient.

a) Provide a verbal report to the nurse at the long-term care facility on the patient, the hospital care, and the patient's current condition. The nurse at the hospital will provide a verbal report to the nurse at the long-term facility. The patient's belongings will accompany the patient to the long-term facility, and the nurse should assure that the belongings are sent with the patient. The original chart will not accompany the patient, but copies of the chart or sections of the chart may be sent based upon agency protocols. The nurse should also recognize and inform the patient that while a transfer may be a welcome event, it also can be stressful.

A patient is being transferred from the ICU to a regular hospital room. What must the ICU nurse be prepared to do as part of this transfer? a) Provide a verbal report to the nurse on the new unit. b) Provide a detailed written report to the unit secretary. c) Delegate the responsibility for providing information. d) Make a copy of the patient's medical record.

a) Provide a verbal report to the nurse on the new unit. The ICU nurse gives a verbal report about the patient's condition and nursing care needs to the nurse on the new unit. This information is not given to a unit secretary, nor is it delegated to others. the medical record is transferred with the patient; a copy is not made.

A professional nurse committed to the principle of autonomy would be careful to: a) Provide the information and support a patient needed to make decisions to advance her own interests. b) Treat each patient fairly, trying to give everyone his or her due. c) Keep any promises made to a patient or another professional caregiver. d) Avoid causing harm to a patient.

a) Provide the information and support a patient needed to make decisions to advance her own interests. The principle of autonomy obligates us to provide the information and support patients and their surrogates need to make decisions that advance their interests.

A quality-assurance program reveals a higher incidence of falls and other safety violations on a particular unit. A nurse manager states, "We'd better find the folks responsible for these errors and see if we can replace them." This is an example of: a) Quality by inspection b) Quality by punishment c) Quality by surveillance d) Quality by opportunity

a) Quality by inspection Quality by inspection focuses on finding deficient workers and removing them. Quality as opportunity (d) focuses on finding opportunities for improvement and fosters an environment that thrives on teamwork, with people sharing the skills and lessons they have learned. Answers b and c are distractors.

A community group has requested the public health nurse to present a program describing the advised schedule of immunizations for children. To plan for this program, what nursing diagnosis would be most appropriate for the nurse to select? a) Readiness for enhanced knowledge: childhood immunizations b) Ineffective Health Maintenance related to lack of knowledge of childhood immunizations c) Risk for Infection Transmission related to lack of immunizations d) Risk for Complications related to childhood illnesses

a) Readiness for enhanced knowledge: childhood immunizations

A nurse demonstrates critical thinking when applying the nursing process to client care. Which behavioral component would the nurse most likely use during the assessment phase? Select all that apply.

a) Recognizing issues b) Recognizing assumptions c) Asking relevant questions d) Exploring ideas e) Interpreting evidence CORRECT ANSWER: A, C and D

An elderly resident of a long-term care facility has developed diarrhea and dehydration as a result of exposure to clostridium difficile during a recent outbreak. The resident's primary care provider has consequently prescribed the antibiotic metronidazole (Flagyl). Which model of health promotion and illness prevention is most clearly evident in these events? a) The Agent-Host-Environment Model b) The Health Belief Model c) The Health Promotion Model d) The Health-Illness Continuum

a) The Agent-Host-Environment Model` The presence of an infectious microorganism and the act of treating that agent are associated with the Agent-Host-Environment Model of health promotion and illness prevention. The patient's beliefs about health are not central in this scenario and health promotion and the pursuit of health are not the most important priorities during this active treatment of illness. This patient is not being characterized as existing on a point on a health continuum.

Which client outcome is a cognitive outcome? Select all that apply. a) The client describes how to perform progressive muscle relaxation. b) The client correctly ambulates with a walker. c) The client identifies signs and symptoms of hypoglycemia. d) The client lists the side effects of digoxin (Lanoxin). e) The client reports cycling 30 minutes three times each week.

a) The client describes how to perform progressive muscle relaxation. c) The client identifies signs and symptoms of hypoglycemia. d) The client lists the side effects of digoxin (Lanoxin).

The nurse is caring for a client who underwent surgery 1 day ago. Which client problem can be addressed by independent nursing diagnoses? a) The client has diminished breath sounds. b) The client has a temperature of 101°F. c) The client is requesting medication for pain. d) The client has a blood pressure of 160/95.

a) The client has diminished breath sounds.

A nurse has been caring for a client who suffered a myocardial infarction 2 days ago. During the morning assessment, the nurse asks the client how he feels. Which scenario warrants further investigation? a) The client stares at the floor and states, "I feel fine." b) The client is sitting in a chair and states, "I feel a lot better than I did yesterday. c) The client smiles at the nurse and states, "I cannot wait to go home." d) The client looks at the nurse and states, "I am still not feeling my best."

a) The client stares at the floor and states, "I feel fine."

Which of the following outcomes does the nurse recognize as being the most appropriate for the client with a nursing diagnosis of risk for infection and an outcome that the client will maintain a normal temperature and exhibit no signs of infection?

a) The client takes the prescribed antibiotic. b) The client understands what symptoms to monitor for. c) The client takes his temperature daily. d) The client has a normal temperature and no signs and symptoms of infection. CORRECT ANSWER:

Which client outcome is an example of a physiologic outcome? a) The client's pulse oximetry reading is 97% on room air 30 minutes after removal of a nasal cannula. b) The client explains how to administer a vaginal cream. c) The client reports walking for 30 minutes each day. d) The client demonstrates active range of motion exercises with left upper extremity.

a) The client's pulse oximetry reading is 97% on room air 30 minutes after removal of a nasal cannula.

The children of a 78-year-old female patient with a recent diagnosis of early-stage Alzheimer's disease are attempting to convince their mother to move into an assisted living facility, a move to which the patient is vehemently opposed. Both the patient and her children have expressed to the nurse how they are entrenched in their position. Which of the following statements expresses a utilitarian approach to this dilemma? a) The decision should be made in light of consequences. b) The patient has a right to self-determination that is the ultimate priority. c) Benefits and burdens should be evenly distributed between the children and the patient. d) The patient's autonomy and independence are the priority considerations.

a) The decision should be made in light of consequences. Utilitarian is the theory of ethics that weighs rightness and wrongness according to consequences and outcomes for all those who are affected. Utilitarianism prioritizes these consequences and outcomes over principles such as autonomy and justice, principles that underlie the other statements addressing the patient's right to self-determination and fair distribution of benefits and burdens.

Which nurse is using criteria to determine expected standards of performance? a) The new graduate nurse consults the policies and procedures of the institution prior to skill implementation. b) The nurse manager provides the staff nurse feedback regarding job performance for the previous year. c) The nurse seeks information from the unlicensed assistive personnel (UAP) regarding the family's response to the nurse's education. d) The nurse preceptor provides feedback to the new graduate nurse after 6 weeks of orientation.

a) The new graduate nurse consults the policies and procedures of the institution prior to skill implementation. Standards are the levels of performance accepted and expected by the nursing staff and other health team members, such as institutional policies and procedures.

The nursing process provides a framework for the client and nurse to work together. Recording prioritized outcomes in the plan of care ensures which benefit?

a) The nurse knows what the client wants. b) Continuity of care can be provided to the client. c) The client will reach the goals of the care plan. d) Each nurse can select which priorities to accomplish. CORRECT ANSWER:

The nurse is caring for a morbidly obese client with a very high body mass index (BMI). After analyzing the assessment data, a nursing diagnosis of Altered Nutrition: More than Body Requirements is included in the client's plan of care. What is the best example of outcome identification and planning for this client using the nursing process?

a) The nurse only allows the client to set long-term goals because there is no way the client will be able to meet the outcomes quickly. b) The client is involved in developing a comprehensive and individualized plan of care with specific outcomes. c) Short-term goals describe multiple client behaviors so that they can be accomplished quickly. d) Long-term goals are not required because many times they are unrealistic. CORRECT ANSWER: B

Nurse M. has recently completed the administration of seasonal influenza vaccinations for the residents of a long-term care facility. Which of the following aims of nursing has Nurse M. most clearly demonstrated? a) preventing illness b) facilitating coping c) restoring health d) promoting health

a) preventing illness Vaccinations are one of the most concrete measures that nurses take to prevent illness in patients, as the antibody production that is induced by vaccination actively prevents the recipient from developing the influenza.

A registered nurse assumes the role of nurse coach to provide teaching to patients who are recovering from strokes. One example of an intervention the nurse may provide related to this role is: a) The nurse uses discovery to identify the patients' personal goals and create an agenda that will result in change. b) The nurse is the expert in providing teaching and education strategies to provide dietary and activity modifications. c) The nurse becomes a mentor to the patients and encourages them to create their own fitness programs. d) The nurse assumes an authoritative role to design the structure of the coaching session and support the achievement of patient goals.

a) The nurse uses discovery to identify the patients' personal goals and create an agenda that will result in change. A nurse coach establishes a partnership with a patient and, using discovery, facilitates the identification of the patient's personal goals and agenda to lead to change rather than using teaching and education strategies with the nurse as the expert. A nurse coach explores the patient's readiness for coaching, designs the structure of a coaching session, supports the achievement of the patient's desired goals, and with the patient determines how to evaluate the attainment of patient goals.

A group of nurse researchers has proposed a study to examine the efficacy of a new wound care product. Which of the following aspects of the methodology demonstrates that the nurses are attempting to maintain the ethical principle of nonmaleficence? a) The nurses are taking every reasonable measure to ensure that no participants experience impaired wound healing as a result of the study intervention. b) The nurses have organized the study in such a way that the foreseeable risks and benefits are distributed as fairly as possible. c) The nurses have completed a literature review that suggests the new treatment may result in decreased wound healing time. d) The nurses have given multiple opportunities for potential participants to ask questions and have been following the informed consent process systematically.

a) The nurses are taking every reasonable measure to ensure that no participants experience impaired wound healing as a result of the study intervention. The principle of nonmaleficence dictates that nurses avoid causing harm. In this study, this may appear in the form of taking measures to ensure that the intervention will not cause more harm than good. The principle of justice addresses the distribution of risks and benefits and the informed consent process demonstrates that autonomy of the therapeutic value of the intervention show a respect for the principle of beneficence.

Which of the following does a nurse recognize is among the rules suggested by the Institute of Medicine's (IOM) Committee on Quality of Health Care in America to improve health care? a) The patient as the source of control b) Cooperation among clinicians c) Providing client care based on a standardized plan of care d) Anticipation of clients' needs e) Safety as a system priority

a) The patient as the source of control b) Cooperation among clinicians d) Anticipation of clients' needs e) Safety as a system priority Each client should have his or her own customized plan of care.

Which of the following activities takes place during the working phase of the nurse-patient relationship? Select all that apply. a) The patient participates actively in the relationship. b) The patient genuinely expresses his or her concerns to the nurse. c) The patient identifies the goals accomplished in the relationship. d) The patient describes the role that the nurse plays in the relationship. e) The patient and nurse identify goals of the relationship.

a) The patient participates actively in the relationship. b) The patient genuinely expresses his or her concerns to the nurse. The working phase of the nurse-patient relationships involves active participation toward goals and genuine expression of concerns and feelings. Identification of goals and relationships occur in the orientation phase while identifying that goals have been accomplished is characteristic of the termination phase.

During a clinical placement on a subacute, geriatric medicine unit, a student nurse fed a stroke patient some beef broth despite the fact that the patient's diet was restricted to thickened fluids. As a result, the patient aspirated and developed pneumonia. Which of the following statements underlies the student's potential liability in this situation? a) The same standards of care that apply to a registered nurse apply to the student. b) The patient's primary nurse is liable for failing to ensure that delegated care was appropriate. c) The student and the nursing instructor share liability for this lapse in care. d) The student's potential liability is likely negated by the insurance carried by the school of nursing.

a) The same standards of care that apply to a registered nurse apply to the student. Despite the fact that their knowledge and skills are still under development, nursing students are held to the same standards of care as registered nurses. Consequently, primary liability does not lie with the student's instructor or the patient's primary nurse. Insurance may be carried by the school of nursing, but this does not negate the student's legal responsibility to provide care at a high standard.

The new nursing graduate correctly states which of the following is true about the implementation phase of the nursing process? Select all that apply.

a) This phase promotes wellness and restores health. b) Care provided during implementation should be documented in the client's chart. c) Implementation is the process of carrying out the plan of care. d) All interventions carried out during this phase must be accompanied by a physicians order. e) Implementation is only carried out by nursing professionals. CORRECT ANSWER: A, B, and C

A client is admitted to the hospital with an abscess on his leg that will not heal after multiple treatment options as an outpatient. The nurse knows from past experiences that the appearance of this type of wound in clients heavily suggests a resistant bacterial infection and the need for contact isolation and IV antibiotics, so she begins to prepare for this admission. What type of problem solving does this exhibit?

a) Trial and Error b) Intuitive c) Scientific d) Experiential CORRECT ANSWER: B

Which of the following activities would the nurse consider to be lifestyle risk factors for the development of health problems? Select all that apply. a) avoiding exercise b) family history of hypertension c) living in an area with high smog levels d) participating in extreme sports e) maintaining a stressful job

a) avoiding exercise d) participating in extreme sports e) maintaining a stressful job A sedentary lifestyle, high stress, and participation in risky sports are all examples of lifestyle-related risk factors for disease and illness. A family history of an illness is a genetic risk factor while pollution is an environmental consideration.

Which of the nurse's following actions is most likely to promote a patient's compliance with her antiretroviral drug regimen for the treatment of human immunodeficiency virus (HIV)? a) Use interactive and learner-appropriate teaching techniques. b) Refer to the characteristics of the HIV virus when explaining how antiretrovirals treat HIV. c) Emphasize the consequences of noncompliance at various points in patient teaching. d) Ensure that teaching materials are evidence-based and explicitly referenced.

a) Use interactive and learner-appropriate teaching techniques. Noncompliance has serious consequences for the health of patients. It can often be prevented by ensuring that patient teaching is interactive and appropriate to the needs and development of each individual learner. Patient education should indeed be evidence-based, but this does not necessarily enhance compliance. Overemphasizing the negative and grounding the education in virology or other forms of science will also not guarantee compliance.

The National Academy of Sciences (2003) has proposed five core competencies necessary for healthcare professionals to meet the needs of the 21st-century healthcare system. Which of the following are among these competencies? Select all that apply. a) Using informatics b) Using evidence-based practice c) Providing patient-centered care d) Implementing cost-containment measures

a) Using informatics b) Using evidence-based practice c) Providing patient-centered care The competencies identified by the National Academy of Science (NAS) are: (1) to provide patient-centered care, (2) to work in interdisciplinary teams, (3) to use evidence-based practice, (4) to apply quality improvement, and (5) to use informatics. Nurses are likely to face increasing pressure to participate in the planning and execution of cost-containment measures in the future, but this is not a core competency identified by the NAS.

The nurse is caring for an underweight client diagnosed with a new food allergy to wheat, rye, and oats. What is the most appropriate intervention for this client with a nursing diagnosis of imbalanced nutrition: less than body requirements?

a) Weigh client PRN b) Monitor for allergies c) Administer 2500 calorie (10,500 kJ) diet, excluding wheat, rye, and oats d) Administer a daily multivitamin CORRECT ANSWER: C

An experienced nurse tells you not to bother studying too hard, since most clinical reasoning becomes "second nature" and "intuitive" once you start practicing. What thinking below should underlie your response? a) When intuition is used alone, there are increased risks and fewer benefits. Intuition often moves problem solving forward quickly, but it might result in a lot of trial-and-error approaches. b) For nursing to remain a science, nurse must continue to be vigilant about stamping out intuitive reasoning. c) The emphasis on logical, scientific, evidence-based reasoning has held nursing back for year.s It's time to champion intuitive, creative thinking! d) It's simply a matter of preference. Some of us are logical, scientific thinkers, and some are intuitive, creative thinkers.

a) When intuition is used alone, there are increased risks and fewer benefits. Intuition often moves problem solving forward quickly, but it might result in a lot of trial-and-error approaches. When intuition is used alone, there are increased risks and fewer benefits. Intuition often moves problem solving forward quickly, but it might result in a lot of trial-and-error approaches. Answer (b) is incorrect because there is a place for intuitive reasoning in nursing, but it will never replace logical, scientific reasoning (c). Critical thinking is contextual and changes depending on the circumstances, not on personal preference (d).

A nursing student is reading a research article from a nursing journal. The student is aware that the paragraph at the beginning of the article that summarizes the article and the findings of the research is a good place to start when reading this article. What part of the article is the nursing student reading? a) abstract b) review of the literature c) results d) conclusions

a) abstract The abstract is located at the beginning of the article, and it summarizes the entire article and usually provides the purpose of collection and data analysis, and a summary of important findings. The review of literature discusses relevant studies that have been conducted in the area of this study, and it is included in the introduction. The results are the findings of the study and are often presented both in words and charts, tables, or graphs. The conclusions are the discussions of the results and suggestions for further study.

A nurse is attempting to complete an admission database. While taking the history, the nurse notices the client appears uncomfortable and slightly tachypneic. The nurse should: a) allow the client to set the pace. b) use only open-ended questions. c) ask questions as quickly as possible. d) tell the client to rest and allow a family member to answer.

a) allow the client to set the pace.

Select the best description of how the nurse applies the nursing process in caring for clients. The nurse:

a) applies intuition and routine care for clients. b) uses critical thinking to direct care for the individual client. c) uses scientific problem solving to meet client problems. d) employs communication to meet the client's needs. CORRECT ANSWER: B

An evening shift nurse is caring for a client scheduled for a colon resection in the morning. The client tells the nurse she is afraid of waking up during surgery. The best response by the nurse is to: a) ask the client why she thinks she will wake up during surgery. b) state "everyone is afraid of that." c) look directly at the client and state, "you are afraid of waking up during surgery." d) ask the surgeon to come to the bedside to reassure the client.

a) ask the client why she thinks she will wake up during surgery.

An elderly patient has been recently diagnosed with vascular dementia. Because he lives alone and has poorly controlled hypertension, he has begun to receive home healthcare. This new aspect of his care is characteristic of which stage of illness? a) assuming a dependent role b) assuming the sick role c) experiencing the symptoms d) achieving recovery and rehabilitation

a) assuming a dependent role The stage of assuming a dependent role often requires assistance in carrying out activities of daily living. As well, the patient often requires care, which may be provided in the home. Experiencing symptoms and assuming a sick role may precede (or accompany) this process. Recovery and rehabilitation are not evident in the patient's present circumstances.

Use of the nursing process in health care allows the nurse to address the needs of the client. The nursing process:

a) provides a universally applicable framework for nursing activities. b) targets desired outcomes for particular illnesses, procedures, or conditions. c) was developed for use by students in nursing assignments. d) is a method of nursing established in 1955. CORRECT ANSWER: A

The patient asks the nurse about the side effects of a drug that is newly prescribed. This is a new drug on the market, and the nurse is not familiar with the side effects and responds, "I am not familiar with the side effects of this drug, so I will look it up and let you know." What type of disposition is the nurse exhibiting during this exchange? a) being intellectually humble b) being creative c) thinking independently d) being fair-minded

a) being intellectually humble The nurse is demonstrating the characteristic of being intellectually humble. Intellectually humble nurses are learning all the time, often from patients and other caregivers, colleagues, and the media. Independent thinkers are careful not to allow the status quo or persuasive individuals to control their thinking. Nurses who are fair-minded are open to different points of view. Being creative may require brainstorming or "thinking outside the box."

A nurse exits the room of a confused patient without raising the side rails on the bed. The failure to raise the side rails would be which of the following elements of liability related to malpractice? a) breach of duty b) damages c) causation d) duty

a) breach of duty Breach of duty is failing to meet the standard of care, and in this case, it was the failure to execute and document the use of appropriate safety measures. Causation is the failure to use appropriate safety measures that results in injury to the patient. Duty refers to an obligation to use due care and is defined by the standard of care appropriate for the nurse-patient relationship. Damages are the actual harm or injury resulting to the patient.

When interacting with a patient, the nurse answers, "I am sure everything will be fine. You have nothing to worry about." This is an example of which of the following inappropriate communication techniques? a) cliche b) giving advice c) being judgmental d) changing the subject

a) cliche Telling a patient that everything is going to be all right is a cliche. The use of this statement is giving false assurance. The use of cliches gives the patient the impression that the nurse is not interested in the patient's condition.

The nurse is examining the assessment data of a client and diagnoses a problem of impaired tissue perfusion based on the following assessment data cues: left foot cool and pale with capillary refill > 3 seconds, diminished dorsalis pedis and posterior tibial pulses, client reports cramping pain in left foot. The nurse is doing what? a) clustering significant data cues b) validating the nursing diagnosis c) identifying contributing factors d) formulating a nursing diagnosis

a) clustering significant data cues Data clustering involves grouping client data or cues that point to the existence of a client health problem. When formulating a nursing diagnosis, the nurse identifies the client health problem related to an etiology and includes subjective and objective data that support the existence of the actual or potential health problem. The nurse identifies contributing factors in the etiology portion of the nursing diagnosis. The nurse validates the nursing diagnosis, often with the client, after a tentative one is formulated.

A patient has received a temporary ostomy during her treatment for colon cancer. Which of the following techniques is most likely to facilitate the patient's ability to independently empty and change the ostomy after discharge? a) demonstration b) lecture c) discussion d) a brochure

a) demonstration Demonstration of a technique allows for cognitive, psychomotor, and effective learning to simultaneously take place. Lectures, brochures, and discussions can foster cognitive learning but have significant limitations in the psychomotor domain and, to a lesser degree, in effective learning.

A nurse is preparing to teach a 45-year-old patient with asthma how to use his inhaler. One of the best methods to teach the patient this skill is by: a) demonstration b) lecture c) discussion d) panel session

a) demonstration Demonstration of techniques, procedures, exercises, and the use of special equipment is an effective patient teaching strategy.

During discharge teaching, the nurse should: a) determine the progress made in established goals b) clarify when the patient should take medications c) report the progress made in teaching to the staff d) include all family members in the teaching session

a) determine the progress made in established goals The discharge planning phase coordinates with the termination phase of a helping relationship. It is important that the nurse determine the progress made in achieving the goals related to the patient's care.

There are various levels of preparation for nursing practice. Which degree program will best prepare a nurse to work as a nurse researcher? a) doctoral b) baccalaureate c) master's d) diploma

a) doctoral Nurses with doctoral degrees meet requirements for academic advancement and are prepared to carry out research necessary to advance nursing theory and practice. A master's degree prepares advanced practice nurses to function in educational settings, managerial roles, as clinical specialists, and in various advanced practice areas. Baccalaureate programs provide a general educational base and incorporate the sciences and humanities. Graduates of a diploma program have a strong emphasis on clinical experiences in direct patient care and are prepared to work in acute, long-term, and ambulatory care facilities.

Upon assessment of a patient with multiple sclerosis, the nurse notes that the patient is difficult to understand due to an inability to produce speech sounds related to poor respiratory control and impaired movement of the lips and tongue. The nurse will document the finding on the patient's chart as: a) dysarthria. b) esophageal speech. c) impaired hearing. d) aphasia.

a) dyarthria Dysarthria interferes with normal control of the speech mechanism. Speech may be slurred or difficult to understand due to lack of ability to produce speech sounds coreectly, maintain good breath control, or coordinate the movements of the lips, tongue, palate, and larynx. Aphasia results in a reduced ability to understand what others are saying, to express oneself, or to be understood. Esophageal speech is often learned after a laryngectomy and requires the use of an electronic device or surgical implant of a voice prosthesis. This scenario does not define a hearing problem.

In the delivery of care, the nurse acts in accordance with nursing standards and the code of ethics and reports a medication error that she has made. The nurse is most clearly demonstrating which of the following professional values? a) integrity b) social justice c) human dignity d) altruism

a) integrity The nurse is demonstrating integrity, which is defined as acting in accordance with an appropriate code of ethics and accepted standards of practice. Seeking to remedy errors made by self or others is an example of integrity. Altruism is a concern for the welfare and being of others. Social justice is upholding moral, legal and humanistic principles. Human dignity is respect for the inherent worth and uniqueness of individuals and populations.

A nurse has come on day shift and is assessing the patient's intravenous setup. The nurse notes that there is a mini-bag of the patient's antibiotic hanging as a piggyback, but that the bag is still full. The nurse examines the patient's medication administration record (MAR) and concludes that the night nurse likely hung the antibiotic but failed to start the infusion. As a result, the antibiotic is 3 hours late and the nurse has consequently filled out an incident report. In doing so, the nurse has exhibited which of the following? a) ethical/legal skills b) technical skills c) interpersonal skills d) cognitive skills

a) ethical/legal skills Reporting problems and unacceptable practices is an aspect of ethical/legal skills. Technical skills enable the sage performance of kinesthetic tasks while interpersonal skills are the manifestations of caring. Cognitive skills encompass knowledge and critical thinking.

Using the nursing process to make ethical decisions involves following several steps. Which step is the nurse implementing when he or she reflects on the decision-making process and the role it will play in making future decisions? a) evaluating b) planning c) diagnosing d) implementing

a) evaluating Evaluating an ethical decision involves reflecting on the process and evaluating those elements that will be helpful in the future. The nurse may also question how this experience can improve reasoning and decision making in the future. Diagnosing the ethical problem involves stating the problem clearly. Planning includes identifying the options and exploring the probable short-term and long-term consequences. Implementing includes the implementation of the decision and comparing the outcomes of the actions with what was considered and hoped for in advance.

The Joint Commission has established standards for admission to a hospital dictating that: (select all that apply) a) every patient should be assessed thoroughly by a registered nurse upon admission. b) the admission process may be safely delegated to unlicensed care providers. c) high-acuity patients must be admitted to a single-bed room. d) a physician must perform a physical assessment of a patient within 3 hours of admission.

a) every patient should be assessed thoroughly by a registered nurse upon admission. b) the admission process may be safely delegated to unlicensed care providers. The Joint Commission has established standards for hospital admission stating that each patient must be assessed by and RN upon admission. This initial assessment cannot safely be delegated. Patients are frequently required to share rooms, even when their acuity is high, and a physician does not necessarily perform a physical assessment after each patient's admission.

A patient has been admitted to the hospital for the treatment of diabetic ketoacidosis, a problem that was accompanied by a random blood glucose reading of 575 mg/dL, vomiting, and shortness of breath. This patient has experienced which of the following phenomena? a) exacerbation b) risk factor c) infection d) morbidity

a) exacerbation This patient has experienced a significant exacerbation of his chronic disease (diabetes mellitus), which has manifested as an acute threat to his health. Morbidity is an epidemiological statistic of the frequency of a disease. His problem does not have an infectious etiology and while risk factors underlie his present condition, they are not the essence of his current state.

A nurse researcher is examining the cause-and-effect relationship between the consumption of tap water containing minimal amounts of bleach and the incidence of cancer in rats. The research is taking place in a laboratory setting. What type of quantitative research is the researcher using based upon this description? a) experimental research b) correlational research c) quasi-experimental research d) descriptive research

a) experimental research Experimental research examines the cause-and-effect relationships between variables under highly controlled conditions. Experimental research studies often occur in a laboratory setting. Descriptive research explores and describes events in a real-life situation to identify relationships between and among events. Correlational research examines the type and degree of relationships between two or more variables. quasi-experimental research examines the cause-and-effect relationships between selected variables and is typically situated outside of a laboratory setting.

The client is an 18-month-old in the pediatric intensive care unit. He is scheduled to have a subgaleal shunt placed tomorrow, and his mother is quite nervous about the procedure. The nurse feels for the mother and tells her that the surgeon "has done this a million times. Your son will be fine." This is an example of what type of nontherapeutic communication? a) false reassurance b) being moralistic c) giving advice d) rescue feelings

a) false reassurance

Which activity is the clearest example of the evaluation step in the nursing process?

a) giving the client a p.r.n. (as needed) dose of captopril (an antihypertensive) in light of this blood pressure reading b) recognizing that the client's blood pressure of 172/101 is an abnormal finding c) taking a client's blood pressure on both arms at the beginning of a shift d) checking the client's blood pressure 30 minutes after administering captopril. CORRECT ANSWER: D

The US Department of Health and Human Services document Healthy People 2010 establishes guidelines aimed at increasing the quality and years of healthy life for Americans as well as eliminating health disparities. What do these goals represent important components of? a) health promotion b) health restoration c) facilitation of coping d) illness prevention

a) health promotion Healthy People 2010 establishes goals and indicators that are reflective of the principles of health promotion. While these goals and the document certainly do not preclude illness prevention, health restoration, and improved coping, the principles and major guidelines are rooted in the aim of health promotion.

A nurse is evaluating the plan of care for a client in the clinic. Which actions will she perform as a classic element of evaluation? Select all that apply. a) identifying evaluative criteria and standards b) interpreting and summarizing findings c) documenting only the facts related to the plan of care d) terminating, continuing, or modifying the plan of care e) collecting data to determine if criteria or standards are being met

a) identifying evaluative criteria and standards b) interpreting and summarizing findings d) terminating, continuing, or modifying the plan of care e) collecting data to determine if criteria or standards are being met

Where do individuals learn their health belief's and values? a) in the family b) in school c) from school nurses d) from peers

a) in the family Healthcare activities, health beliefs, and health values are learned within one's family.

Which of the following actions most clearly demonstrates a nurse's commitment to social justice? a) lobbying for an expansion of Medicare eligibility and benefits b) ensuring that a hospital patient's diet is culturally acceptable c) answering a patient's questions about her care clearly and accurately d) documenting nursing care in a timely, honest, and thorough manner

a) lobbying for an expansion of Medicare eligibility and benefits Social justice is a professional value that encompasses efforts to promote universal access to healthcare, such as the expansion of publicly funded programs like Medicare. Culturally competent care is a reflection of human dignity while answering patients' questions and documenting accurately are expressions of the value of integrity.

The nursing instructor recognizes that a student understands the nursing process when the student states that the purpose of the assessment phase of the nursing process is to:

a) make a judgment about the client's health status, ability to manage his or her own health care, and need for nursing. b) develop an individualized plan of nursing care. c) develop a prioritized list of nursing diagnoses. d) continue, modify, or terminate nursing care CORRECT ANSWER:

A nurse gives a speech on nutrition to a group of pregnant women. What is the speech itself known as? a) message b) channel c) stimulus d) source

a) message

The nursing student recognizes that the most appropriate intervention for a newly diagnosed diabetic client with a nursing diagnosis of deficient knowledge is to:

a) monitor for hypo-/hyperglycemia. b) monitor blood sugar before meals. c) teach the client how to administer his own insulin. d) administer insulin as prescribed. CORRECT ANSWER: C

A malpractice case involving an incident in the operating room has entered litigation. A scrub nurse has been asked to present her firsthand knowledge of the incident based upon recollection of the incident and documentation in the patient record. The nurse's role in this malpractice case is: a) nurse as a fact witness. b) nurse as the plaintiff. c) nurse as the defendant. d) nurse as an expert witness.

a) nurse as a fact witness. The nurse will act as a fact witness. Fact witnesses, who are placed under oath, must base their testimony on only firsthand knowledge of the incident and not on assumptions. The nurse will be asked if the testimony is based on independent recollection of the incident or on documentation in the patient's record. The nurse as defendant will work closely with an attorney while preparing the defense. The nurse as an expert witness will explain to the judge and jury what happened based upon the patient's record and offer an opinion about whether the nursing care met acceptable standards.

When the nurse administers pain medication to a postoperative client, the phase of the nursing process that is occurring is:

a) nursing diagnosis. b) assessment. c) implementation. d) planning. ANSWER: C

An obese client is in the clinic to be started on a weight loss plan. The client tells the nurse that she loves to eat, her favorite food is hamburgers, and she does not like to exercise. The nurse creates a nursing diagnosis of ineffective health maintenance to include in the plan of care. What is the most appropriate outcome for this nursing diagnosis for this client? The client will:

a) only eat three meals per day. b) exercise every day for at least 30 minutes. c) stop eating meat and walk every day after dinner. d) create an exercise plan that is realistic and valued. CORRECT ANSWER: D

During an assessment of a newly admitted patient the nurse asks the patient many questions. The nurse begins the assessment by asking, "How many times have you been hospitalized this year for your back pian?" This is an example of which type of question? a) open-ended question b) sequencing question c) closed question d) reflective question

a) open-ended question An open-ended question is often used when the nurse is obtaining a nursing history and allows the patient to reply with a wide range of possible responses and encourages free verbalization. A closed question is answered by one or two words and often "yes" or "no." A sequencing question is used to place events in a chronological order and to investigate a possible cause-and-effect relationship. A reflective question involves repeating what the person has said or describing the person's feelings.

The focus of a hospital's current quality assurance program is a comparison between the health status of clients upon admission and at the time of discharge. This form of quality assurance is characteristic of: a) outcome evaluation. b) nursing audit. c) process evaluation. d) structure evaluation.

a) outcome evaluation.

A nurse identifies the following: "The client will report a pain rating of 4 or less within 30 to 45 minutes of receiving prescribed analgesic." The nurse has identified:

a) outcome. b) nursing diagnosis. c) subjective data. d) intervention. CORRECT ANSWER: A

A patient walking to the bathroom with a stooped gait is noted with facial grimacing. IT is important that the nurse assess the patient for which of the following? a) pain b) anxiety c) depression d) fluid volume deficit

a) pain A patient who presents with nonverbal communication o f a stooped gait and facial grimacing is most likely experiencing pain. It is important that the nurse clarify this nonverbal behavior.

After reviewing information about the four concepts common to nursing theories, the students demonstrate understanding of the information when they identify which concept as most important? a) person b) environment c) health d) nursing

a) person Of the four concepts, the most important is the person.

Which of the following levels of basic human needs is most basic? a) physiologic b) safety and security c) love and belonging d) self-actualization

a) physiologic Physiologic needs are the most basic and must be met at least minimally to sustain life.

Which type of theory focuses on clinical nursing practice? a) prescriptive theory b) descriptive theory c) developmental theory d) systems theory

a) prescriptive theory Prescriptive theories address nursing interventions and are designed to control, promote, and change clinical nursing practice.

The community health nurse performs blood pressure screenings at a senior center twice a month. This activity encompasses which aim of nursing? a) restoring health b) facilitating coping with disability and death c) preventing illness d) promoting health

a) restoring health Performing diagnostic measurements and assessments that detect an illness, such as blood pressure screenings, is an example of restoring health. The object of illness-prevention activities is to reduce the risk for illness through educational programs, resources that encourage healthy lifestyles, and information on good health habits. Promoting health focuses on health-related activities, health awareness, and a patient's self-awareness. Facilitating coping with disability and death assists the patient and family in dealing with altered function, life crisis, and death.

The agent-host-environment model of health and illness is based on what concept? a) risk factors b) infectious diseases c) behaviors to promote health d) stages of illness

a) risk factors The interaction of the agent-host-environment creates risk factors that increase the probability of disease.

The nurse is conducting a community health promotion class and has developed scenarios that will involve active participation by the class attendees. What type of teaching strategy is the nurse incorporating into her class? a) role-playing b) panel discussion c) role modeling d) programmed instruction

a) role-playing Role-playing allows the learner to experience, relive, or anticipate an event. The nurse explains the scenario and then allows the individual to play out the scene. Role modeling involves the nurse behaviors and the patient observing and learning from these behaviors. Programmed instruction incorporates the use of books as the instructor, independent of study with a teacher. A panel discussion involves a presentation of information by two or more people.

What is the primary purpose of the identification bracelet worn by hospitalized patients? a) safety b) communication c) advocacy d) legality

a) safety The identification bracelet is a safety measure ensuring that the correct patient is given medications, has surgery, and so forth.

After reviewing several research articles, the clinical nurse specialist on a medical-surgical unit rewrites the procedure on assessing placement of a nasogastric tube. What source of nursing knowledge did the nurse use in this situation? a) scientific knowledge b) authoritative knowledge c) traditional knowledge d) philosophical knowledge

a) scientific knowledge The clinical nurse specialist utilized scientific knowledge, which is arrived at through the research-based scientific method. Philosophical knowledge is not a source of nursing knowledge, but is a type of general knowledge. Authoritative knowledge comes from an expert and is accepted as truth based on the person's perceived expertise. Traditional knowledge is that part of nursing practice passed down from generation to generation and is not based upon scientific inquiry.

Which of the following is involved in the implementation step in the nursing process?

a) selecting nursing interventions b) documenting the plan of care c) identifying measurable outcomes d) documenting the nursing care and client responses CORRECT ANSWER: D

Nurse Practice Acts are examples of which type of laws? a) statutory laws b) common law c) constitutional laws d) administrative laws

a) statutory laws Nurse Practice Acts are statutory laws. Statutory laws must be in keeping with both the federal constitution and the state constitution.

In understanding the historical influences on nursing knowledge, nursing as a profession struggled for years to establish its own identity and to receive recognition for its contributions to healthcare because: a) the conceptual and theoretical basis for nursing practice came from outside the profession. b) women were independent and refused to work collectively. c) nurses spent most of their time in laboratory settings conducting research. d) nurses were too busy working in practice to increase the public awareness associated with the role of the nurse.

a) the conceptual and theoretical basis for nursing practice came from outside the profession. Despite Nightingale's belief in the uniqueness of nursing, the training of nurses was initially carried out under the direction and control of the medical profession. Because the conceptual and theoretical basis for nursing practice came from outside the profession, nursing struggled for years to establish its own identity and to receive recognition for its significant contributions to healthcare.

A nursing student has been asked to correlate her clinical experiences with two different theories of nursing. The student will recognize that which of the following concepts are common to all theories of nursing? Select all that apply. a) the patient b) the environment c) illness d) needs e) nursing f) health

a) the patient b) the environment e) nursing f) health While nursing theories vary significantly in their conceptualizations, the elements that are common to all include the patient (person), the environment, health, and nursing. The concepts of needs and illness are addressed by some theories while these are not explicitly defined by other theories.

In the clinical setting, a nurse is working on developing higher-level reflection skills. With which activity would the nurse most likely be engaged?

a) thinking about relationships involved b) reevaluating experience in light of ideas c) recalling a sequence of events d) identifying a positive situation CORRECT ANSWER: B

Which of the following factors is most important in the development of rapport between nurse and patient? a) trust b) kindness c) happiness d) skill

a) trust Rapport is a feeling of mutual trust between nurse and patient. Other traits and abilities, such as kindness, happiness, and skill, may foster rapport in particular patient interactions, but trust remains the central component of rapport.

Nurses use the nursing process to plan care for clients. In which case is the nursing process applicable? Select all that apply.

a) when families are not supportive and do not wish to participate in care. b) when clients are totally dependent on the nurse for care c) when nurses work with clients who are able to participate in their care d) when families are clearly supportive and wish to participate in care CORRECT ANSWERS: ALL OF THEM NURSING PROCESS IS ALWAYS APPLICABLE

A nurse is assessing a patient's nutritional intake prior to admission based upon information that indicates the patient has lost 10 pounds over the last 2 monts. An appropriate therapeutic communication technique to gain information is which of the following? a) "What factors have contributed to your weight loss over the last few months?" b) "You have lost a lot of weight. Do you neglect you nutrition at home?" c) "I hear from you husband that you have recently lost a lot of weight. Is that true?" d) "You are thin, so why did you lose 10 pounds over the last 2 months?"

a)"What factors have contributed to your weight loss over the last few months?" An appropriate way to gain assessment information is to ask an open-ended, nonjudgmental question. Barriers to communication include the use of cliches, rumors, giving false assurance, questions that use the words "how" or "why," questions that require "yes" or "no" answers, and leading questions that suggest a response the interviewer wishes to hear.

A client has been admitted to a hospital suffering from an acute psychotic episode. What assessment data would the nurse identify as this client's strengths? Select all that apply. a) The client has ample financial resources. b) The client is willing to attend counseling sessions. c) The client has been living on the street for 3 weeks. d) The client is male and 35 years old. e) The client refuses to take the ordered medication.

a, b The client's financial resources and willingness to attend counseling will be positive factors in the client's recovery. The lack of a stable living environment would lessen the chance for compliance with the health care regimen once discharged. The client's refusal to take medication would not allow the health care providers to utilize an important part of this client's treatment. The fact that the client is male and 35 is neither a strength nor a weakness.

The nurse is collecting health data and avoids using closed-ended questions. Which of the following are examples of closed-ended questions? Select all that apply. a) "Do you smoke cigarettes?" b) "Are you ready to get out of bed?" c) "Is there any chance you might be pregnant?" d) "Does it hurt when I touch you here?" e) "What plans do you have after you are discharged?" f) "What sorts of things do you do for fun?"

a, b, c, d

Which are principles of documentation? Select all that apply. a) Timely b) Objective c) Confidentiality d) Accuracy e) Subjective

a, b, c, d The principles of proper documentation include confidentiality, accuracy, completeness, concise, objective, organized, timely, and legibility.

What information should the nurse document in the medication record when administering a non-narcotic pain medication? Select all that apply. a) Reason given b) Effectiveness of medication c) Dose d) Vital signs e) Time

a, b, c, e The nurse should document the medication given, time, route, dose, reason given, and effectiveness of the medication on the medication administration record.

A nurse is engaged in teaching a 5-year-old boy that is newly diagnosed with type 1 diabetes. His mother is his primary caregiver and is present with him. The nurse is preparing to teach insulin injections. The client's mother informs the nurse that she has a fear of needles. What is appropriate for a successful education session? Select all that apply. a) ensuring 20 minutes of uninterrupted teaching time b) using a doll so that the client can practice this skill c) asking the client's mother to leave the room because she will be a distraction d) providing the client's mother with an informational pamphlet about insulin injection

a, b, d

A nurse is using critical pathway methodology for choosing interventions for a patient who is receiving chemotherapy for breast cancer. Which nursing actions are characteristics of this system being used when planning care? Select all that apply. a) The nurse uses a minimal practice standard and is able to alter care to meet the patient's individual needs. b) The nurse uses a binary decision tree for stepwise assessment and intervention. c) The nurse is able to measure the cause-and-effect relationship between pathway and patient outcomes. d) The nurse uses broad, research-based practice recommendations that may or may not have been tested in clinical practice. e) The nurse uses preprinted provider orders used to expedite the order process after a practice standard has been validated through research. f) The nurse uses a decision tree that provides intense specificity and no provider flexibility.

a, c

A nurse is communicating the plan of care for a client who is unconscious. Which nursing actions best facilitate this process? Select all that apply. a) The nurse is careful what is said in the client's presence since hearing is the last sense to go. b) The nurse speaks with the client before touching him. c) The nurse assumes the client can hear and discusses things that would ordinarily be discussed. d) The nurse does not use touch to communicate with the client. e) The nurse speaks to the client in a louder than normal voice. f) The nurse raises environmental noises to help stimulate the client.

a, c, d The nursing actions that best facilitate communication with a client who is unconscious would include being careful what is said in the client's presence because hearing in believed to be the last sense to go. The nurse would assume the client can hear and discuss things that would ordinarily be discussed. The nurse would speak with the client before touching them. The nurse would not speak to the client in a louder than normal voice. The nurse would minimize environmental noises to facilitate communication. The nurse would use touch to communicate with the client.

A nurse is communicating the plan of care to a client who is cognitively impaired. Which nursing actions facilitate this process? Select all that apply. a) The nurse keeps communication simple and concrete. b) The nurse communicates in a busy environment to hold the client's attention. c) The nurse maintains eye contact with the client. d) If there is no response, the nurse does not repeat what is said and takes a break. e) The nurse shows patience with the client and gives the client time to respond. f) The nurse gives lengthy explanations of the care that will be given.

a, c, e,

During a nursing staff meeting, the nurses resolve a problem of delayed documentation by agreeing unanimously that they will make sure all vital signs are reported and charted within 15 minutes following assessment. This is an example of which characteristics of effective communication? Select all that apply. a) Group decision making b) Group leadership c) Group power d) Group identity e) Group patterns of interaction f) Group cohesiveness

a, d, e, f a) Group decision making d) Group identity e) Group patterns of interaction f) Group cohesiveness

A nurse is using the implementation step of the nursing process to provide care for patients in a busy hospital setting. Which nursing actions best represent this step? Select all that apply. a) The nurse carefully removes the bandages from a burn victim's arm. b) The nurse assesses a patient to check nutritional status. c) The nurse formulates a nursing diagnosis for a patient with epilepsy. d) The nurse turns a patient in bed every 2 hours to prevent pressure ulcers. e) The nurse checks a patient's insurance coverage at the initial interview. f) The nurse checks for community resources for a patient with dementia

a, d, f. During the implementing step of the nursing process, nursing actions planned in the previous step are carried out. The purpose of implementation is to assist the patient in achieving valued health outcomes: promote health, prevent disease and illness, restore health, and facilitate coping with altered functioning.

A nurse is providing teaching to patients in a short-term rehabilitation facility. Which examples are common teaching mistakes made by health care professionals? (Select all that apply.) a) The nurse fails to accept that patients have the right to change their minds. b) The nurse negotiates goals with the patient. c) The nurse evaluates what the patient has learned. d) The nurse reviews educational media when planning learner objectives. e) The nurse ignores the restrictions of the patient's environment. f) The nurse uses medical jargon frequently when discussing the teaching plan.

a, e, f

Nurses must take into considerations factors that influence communication when communicating with patients. Which examples accurately reflect communication that considers these variables? (Select all that apply.) a) A nurse takes into consideration a patient's culture when planning how far away to sit when communicating. b) A female nurse communicates in the same manner to both male and female patients. c) A nurse uses the intimate zone when explaining painful procedures to young children. d) A nurse communicates in the same manner to patients of different cultures to avoid stereotyping patients. e) A nurse checks a patient's occupation for clues on how this will affect communication but avoids stereotyping the person. f) A nurse uses simple language and demonstrations to explain a procedure to a preschooler.

a, e, f

A nurse is prioritizing the following patient diagnoses according to Maslow's hierarchy of human needs: (1) Disturbed Body Image (2) Ineffective Airway Clearance (3) Spiritual Distress (4) Impaired Social Interaction Which answer choice below lists the problems in order of highest priority to lowest priority based on Maslow's model? a) 2, 4, 1, 3 b) 3, 1, 4, 2 c) 2, 4, 3, 1 d) 3, 2, 4, 1

a. 2, 4, 1, 3. Because basic needs must be met before a person can focus on higher ones, patient needs may be prioritized according to Maslow's hierarchy: (1) physiologic needs, (2) safety needs, (3) love and belonging needs, (4) self-esteem needs, and (5) self-actualization needs. #2 is an example of a physiologic need, #4 is an example of a love and belonging need, #1 is an example of a self-esteem need, and #3 is an example of a self-actualization need.

A student nurse is on a clinical rotation at a busy hospital unit. The RN in charge tells the student to change a surgical dressing on a patient while she takes care of other patients. The student has not changed dressings before and does not feel confident with the procedure. What would be the student's best response? a) Tell the RN that he or she lacks the technical competencies to change the dressing independently. b) Assemble the equipment for the procedure and follow the steps in the procedure manual. c) Ask another student nurse to work collaboratively with him or her to change the dressing. d) Report the RN to his or her instructor for delegating a task that should not be assigned to student nurses.

a. Student nurses should notify their nursing instructor or nurse mentor if they believe they lack any competencies needed to safely implement the plan of care.

A 61-year-old male patient has been admitted to a medical unit with a diagnosis of pancreatitis secondary to alcohol use. Which of the patient's following statements suggests that nurses' teaching has resulted in effective learning? a) "My intravenous drip will keep me from getting dehydrated right now." b) "I'm starting to see how my lifestyle has caused me to end up here." c) "I can see how things could have been much worse if I hadn't got to the hospital when I did." d) "I understand why they're not letting me eat anything for the time being."

b) "I'm starting to see how my lifestyle has caused me to end up here." The patient's understanding of his contribution to his problem demonstrates a shift in attitude and feelings that is characteristic of effective learning. Understanding the treatment, course, and prognosis of his illness are aspects of cognitive learning.

A nursing student is preparing to administer morning care to a patient. What is the most important question that the nursing student should ask the patient about personal hygiene? a) "Would you prefer a bath or a shower?" b) "May I help you with a bed bath now or later this morning?" c) "I will be giving you your bath. Do you use soap or shower gel?" d) "I prefer a shower in the evening. When would you like your bath?"

b) "May I help you with a bed bath now or later this morning?"

A nursing student is preparing to administer morning care to the patient. What is the most important question that the nursing student should ask the patient? a) "Would you prefer a bath or a shower?" b) "May I help you with a bed bath now or later this morning?" c) "I will be giving you your bath. Do you use soap or shower gel?" d) "I prefer a shower in the evening. When would you like your bath?"

b) "May I help you with a bed bath now or later this morning?" It is important that the nurse asks permission to assist the patient with a bath. This allows for consent to assist the patient with care that invades the patient's private zones.

A nurse in the rehabilitation division states to her head nurse, Mr. Tyler, "I need the day off and you didn't give it to me!" The head nurse replies, "Well, I wasn't aware you needed the day off, and it isn't possible since staffing is so inadequate." Instead of this exchange, what communication by the nurse would have been more effective? a) "Mr. Tyler, I placed a request to have August 8th off, but I'm working and I have a doctor's appointment." b) "Mr. Tyler, I would like to discuss my schedule with you. I requested the 8th of August off for a doctor's appointment. Could I make an appointment?" c) "Mr. Tyler, I will need to call in on the 8th of August because I have a doctor's appointment." d) "Mr. Tyler, since you didn't give me the 8th of August off, will I need to find someone to work for me?"

b) "Mr. Tyler, I would like to discuss my schedule with you. I requested the 8th of August off for a doctor's appointment. Could I make an appointment?"

A nurse on the rehabilitation division states to her head nurse, "I need the day off and you didn't give it to me!" The head nurse replies, "Well, I wasn't aware you needed the day off, and it isn't possible since staffing is so inadequate." In the act of sending the message, which statement would be considered more effective? a) "Mr. Tyler, I placed a request to have August 8th off, but I'm working and I have a doctor's appointment." b) "Mr. Tyler, I would like to discuss my schedule with you. I requested the 8th of August off for a doctor's appointment. Could I make an appointment?" c) "Mr. Tyler, I will need to call in on the 8th of August because I have a doctor's appointment." d) "Mr. Tyler, since you didn't give me the 8th of August off, will I need to find someone to work for me?"

b) "Mr. Tyler, I would like to discuss my schedule with you. I requested the 8th of August off for a doctor's appointment. Could I make an appointment?" Effective communication by the sender involves the implementation of nonthreatening information by showing respect to the receiver. The nurse should identify the subject of the meeting and be sure it occurs at a mutually agreed upon time.

A friend of a nurse calls and asks if she is still working at Memorial Hospital. The nurse replies, "Yes." The friend tells the nurse that his girlfriend's father was just admitted as a patient and he wants the nurse to find out how he is. The friend states, "Sue seems unusually worried about her dad, but she won't talk to me and I want to be able to help her." What is the best initial response the nurse should make? a) "You shouldn't be asking me to do this. I could be fined or even lose my job for disclosing this information." b) "Sorry, but I'm not able to give information about patients to the public—even when my best friend or a family member asks." c) "Because of the Health Insurance Portability and Accountability Act, you shouldn't be asking for this information unless the patient has authorized you to receive it! This could get you in trouble!" d) "Why do you think Sue isn't talking about her worries?"

b) "Sorry, but I'm not able to give information about patients to the public—even when my best friend or a family member asks."

A client is scheduled for thoracentesis. The nurse assesses that the client appears anxious about the procedure and needs honest support and reassurance. What is the most appropriate response by the nurse to this client? a) "The procedure may take only 2 minutes, so you might get through it by mentally counting up to 120." b) "The needle causes discomfort or pain when it goes in, but I will be by your side throughout and will help you hold your position." c) "I will be by your side throughout the procedure; the procedure will be painless if you don't move." d) "You may feel very uncomfortable when the needle goes in, but you should breathe rhythmically."

b) "The needle causes discomfort or pain when it goes in, but I will be by your side throughout and will help you hold your position."

A nurse who is caring for a morbidly obese male teenager forms a contractual agreement with him to achieve his weight goals. Which statement best describes the nature of this agreement? a) "This agreement forms a legal bond between the two of us to achieve your weight goals." b) "This agreement will motivate the two of us to do what is necessary to meet your weight goals." c) "This agreement will help us determine what learning outcomes are necessary to achieve your weight goals." d) "This agreement will limit the scope of the teaching session and make stated weight goals more attainable."

b) "This agreement will motivate the two of us to do what is necessary to meet your weight goals." A contractual agreement is a pact two people make setting out mutually agreed-on goals. Contracts are usually informal and not legally binding. When teaching a patient, such an agreement can help motivate both the patient and the teacher to do what is necessary to meet the patient's learning outcomes. The agreement notes the responsibilities of both the teacher and the learner, emphasizing the importance of the mutual commitment.

The daughter of an elderly female patient has asked the nurse why a urine specimen was collected from her mother earlier that morning. How can the nurse respond to the daughter's query? a) "Sometimes sick urine can make the whole person sick, and this might be causing her fever." b) "We want to test your mother's urine to make sure she doesn't have a urinary tract infection." c) "Your mother's doctor ordered a urine C&S to rule out a UTI." d) "We want to do everything we can to get your mother healthy again."

b) "We want to test your mother's urine to make sure she doesn't have a urinary tract infection." In order to communicate effectively, the nurse needs to avoid the use of jargon or abbreviations ("C&S") that are unfamiliar to those outside the healthcare system. At the same time, accuracy is important, and vague and "dumbed-down" answers ("we want to do everything we can," "sick urine") are inappropriate.

Which of the following statements or questions would be appropriate in establishing a discharge plan for a patient who has had major abdominal surgery? a) "I'll bet you will be so glad to be home in your own bed." b) "What are your expectations for recovery from your surgery?" c) "Be sure and take your pain medications and change your dressing." d) "You will just be fine! Please stop worrying."

b) "What are your expectations for recovery from your surgery?" It is important to assess the expectations of the patient (and family) when assessing healthcare needs for discharge planning.

The nurse has been working with a patient for several days during the patient's recovery in hospital from a femoral head fracture. How should a nurse best evaluate whether patient education regarding falls prevention in the home has been effective? a) "Do you have any questions about the fall prevention measures that we've talked about?" b) "What changes will you make around your house to reduce the chance of future falls?" c) "In light of what we've talked about, why is it important that you remove the throw rugs i your house?" d) "Do you think that the safety measures I taught you are clear and realistic?"

b) "What changes will you make around your house to reduce the chance of future falls?" An open-ended question that requires the patient to apply the information that has been taught is often the most accurate way to evaluate the effectiveness of patient teaching. Yes/no questions are much less effective ("Do you have any questions?"; "Do you think that the safety measures I taught you are clear and realistic?"). Asking the patient about the importance of falls prevention does not directly assess what the patient will actually do to prevent falls.

A 70-year-old female client had a cholecystectomy four days ago. The client's daughter tells the nurse, "My mother seems confused today." Which question would be best for the nurse to ask in order to assess the client's orientation? a) "Are you in a hospital?" b) "What day of the week is it?" c) "Is your name Evelyn?" d) "Is today the first day of the month?"

b) "What day of the week is it?"

A nurse writes nursing diagnoses for patients and their families visiting a community health clinic. Which nursing diagnoses are correctly written as three-part nursing diagnoses? (1) Disabled Family Coping related to lack of knowledge about home care of child on ventilator (2) Imbalanced Nutrition: Less Than Body Requirements related to inadequate caloric intake while striving to excel in gymnastics as evidenced by 20-pound weight loss since beginning the gymnastic program, and greatly less than ideal body weight when compared to standard height weight charts (3) Need to learn how to care for child on ventilator at home related to unexpected discharge of daughter after 3-month hospital stay as evidenced by repeated comments "I cannot do this," "I know I'll harm her because I'm not a nurse," and "I can't do medical things" (4) Spiritual Distress related to inability to accept diagnosis of terminal illness as evidenced by multiple comments such as "How could God do this to me?," "I don't deserve this," "I don't understand. I've tried to live my life well," and "How could God make me suffer this way?" (5) Caregiver Role Strain related to failure of home health aides to appropriately diagnose needs of family caregivers and initiate a plan to facilitate coping as evidenced by caregiver's loss of weight and clinical depression a) (1) and (3) b) (2) and (4) c) (1), (2), and (3) d) All of the above

b) (2) and (4) (1) is a two-part diagnosis, (3) is written in terms of needs and not an unhealthy response, and (5) is a legally inadvisable statement.

You are a nurse taking care of a 49-year-old man who was admitted for acute pancreatitis. He was admitted yesterday and has been on IV fluids, has been kept NPO (nothing by mouth), and has been given IV pain medications since. During your nursing assessment he tells you that his pain is a 4/10 on the pain scale. It starts out in his epigastric area and radiates to his back. He also has been nauseous this morning. Your facility charts by exception. You finish filling out the flow sheet and have to write a progress note. Which of the following is an example of writing a progress note that represents charting by exception for this patient? a) 4/10 pain located in epigastric area and radiating to back with nausea; on IV fluids b) 4/10 pain on pain scale, epigastric radiating to back; also complains of nausea c) NPO, 4/10 pain, epigastric radiating to back, nausea d) Forty-nine-year-old man, 125 mL/h of normal saline, NPO, pain 4/10 on pain scale with 2 mg IV dilaudid every 4 hours

b) 4/10 pain on pain scale, epigastric radiating to back; also complains of nausea Charting by exception charts only that which falls outside the standard of care and norms.

Which statement best describes the client most motivated to learn? a) A 52-year-old male; hired to drive client home from clinic b) A 70-year-old female; learning care so spouse can come home c) A 29-year-old male; significant other insisting on education d) A 25-year-old female; just completed a course of physical therapy

b) A 70-year-old female; learning care so spouse can come home

Which of the following nursing actions is the clearest example of a nurse applying the principles of community-based care? a) A nurse organizes a blood pressure screening clinic at a busy shopping mall. b) A nurse provides wound care for a patient's venous ulcer at an ambulatory clinic near the patient's home. c) A nurse cares for a high-risk pregnant patient from a rural area at a tertiary care hospital that is in a large, urban center. d) A nurse educates high school students on safer sex practices at a series of inschool workshops.

b) A nurse provides wound care for a patient's venous ulcer at an ambulatory clinic near the patient's home. A central component of community-based care is providing care for acute and chronic illnesses in the geographic region near the patient's home. Blood pressure screening and health-maintenance education are population-based and community health initiatives that are distinct from community-based care.

Nurses use social media to share ideas, develop professional connections, access educational offerings and forums, receive support, and investigate evidence-based practices. What is an example of the proper use of social media by a nurse? a) A nurse posts pictures of a patient who accomplished a goal of losing 100 pounds and later deletes the photo. b) A nurse uses a disclaimer to verify that any views expressed on Facebook are his or hers alone and not the employer's. c) A nurse describes a patient on Twitter by giving the room number rather than the name of the patient. d) A nurse describes a patient on Twitter by giving the patient's diagnosis rather than the patient's name.

b) A nurse uses a disclaimer to verify that any views expressed on Facebook are his or hers alone and not the employer's.

The nurse caring for a client with obesity would like to address the possible health problems that can develop related to obesity. To plan care for this client, what type of nursing diagnosis would the nurse formulate? a) A possible nursing diagnosis b) A risk nursing diagnosis c) A wellness diagnosis d) An actual nursing diagnosis

b) A risk nursing diagnosis

A 20-year-old pregnant patient is visibly distraught when the labor and delivery nurse tells her that she requires a urinary catheter. The patient is embarrassed by her anxiety and eventually tells the nurse that this is because of "what my father used to do to me." In addition to fulfilling any legal obligation to report the patient's statement, which of the nurse's following actions is most appropriate? a) Thank the patient for her candor and explore the possibility of the nurse providing long-term counseling. b) Acknowledge the significance of the patient's statement and facilitate a counseling referral. c) Explore the matter in greater detail with the patient and encourage her to report this to the authorities. d) Begin motivational counseling to help the patient overcome her difficult past.

b) Acknowledge the significance of the patient's statement and facilitate a counseling referral. The gravity of the patient's statement indicates a need for professional counseling that is beyond the scope of the labor and delivery nurse. Acknowledgement of the patient's statement is necessary, and there is a legal responsibility for the nurse to report the matter in most jurisdictions; it would be inappropriate to place this onus solely on the patient.

A nurse is treating a client with congestive heart failure. The client informs the nurse that he is having difficulty walking up the stairs in his home. He is frustrated because he used to be a runner, and now he can barely walk to the store. Which is an accurate actual nursing diagnosis for this client? a) Activity Intolerance as evidenced by inability to walk up and down stairs and inability to walk to the store. b) Activity Intolerance related to congestive heart failure as evidenced by inability to walk up and down stairs. c) Impaired Coping related to new diagnosis of congestive heart failure. d) Risk for Impaired Coping related to congestive heart failure.

b) Activity Intolerance related to congestive heart failure as evidenced by inability to walk up and down stairs.

The nurse observes that a client responds better to health education when the nurse motivates him and assures him about the benefits of the teaching. In which of the following learning domains does the client's learning style fall? a) Psychomotor domain b) Affective domain c) Interpersonal domain d) Cognitive domain

b) Affective domain

A nurse is caring for a client with dementia. Which documentation by the nurse best follows documentation guidelines? a) Confused, belligerent, and uncooperative with care b) Alert and oriented to self only, hitting staff members with newspaper, did not follow commands to brush teeth c) Yelling at staff members, dementia worse today, refused breakfast d) Inappropriate behavior during breakfast, screamed during the shower, smiled while kicking other clients

b) Alert and oriented to self only, hitting staff members with newspaper, did not follow commands to brush teeth Nursing documentation should focus on behaviors and avoid words such as better, normal, or worse. Using the terms inappropriate behavior or belligerent are judgmental statements. The actual behaviors witnessed should be documented.

When caring for a client at the health care facility, the nurse observes that the client is having difficulty understanding the health education. Which action is most appropriate? a) Replace one-on-one teaching with written materials. b) Assess for cultural differences. c) Boost the morale of the client. d) Delegate the health education to a colleague.

b) Assess for cultural differences. When the client is having difficulty learning, it may be possible that the client does not understand the language that the nurse speaks. In such a case, the nurse should take the necessary steps to break the cultural barrier and then proceed with the education. Written materials can enhance many clients' learning, but will not necessarily overcome many of the common barriers to understanding, including cultural and linguistic factors. The nurse should take action to overcome any barriers to the learning process before delegating to a colleague.

Nurses formulate physiologic goals for patients when providing patient care. What are examples of physiologic goals? (Select all that apply.) a) By 4/6/15, the baby will demonstrate adequate sleep-wakefulness patterns. b) Before discharge, the baby will have reached a target weight gain of 8 lb (birth weight: 7 lb, 6 oz). c) By 4/6/15, the parents will list appropriate resources in case questions arise after discharge. d) By 4/6/15, the baby will show an adequate comfort level indicating satisfactory parenting. e) Before discharge, the parents of the baby will verbalize decreased anxiety about taking care of a newborn. f) Before discharge, the parents will demonstrate confidence in bathing and feeding their baby.

b) Before discharge, the baby will have reached a target weight gain of 8 lb (birth weight: 7 lb, 6 oz). d) By 4/6/15, the baby will show an adequate comfort level indicating satisfactory parenting. f) Before discharge, the parents will demonstrate confidence in bathing and feeding their baby. Physiologic goals meet the need of normal, healthy body functioning. An infant's sleep-wake patterns, comfort level, and weight are all examples of physiologic goals/outcomes.

A client is about to leave the hospital after having surgery for a fractured left femur. It is now in a plaster cast. The client asks how long before the cast will be dry. The nurse notes on his plan of care a learning outcome stating "Client will verbalize appropriate cast care upon discharge." This represents what type of outcome? a) Affective b) Cognitive c) Psychomotor d) Physical change

b) Cognitive

The mother of an infant comes to the clinic and asks the nurse if the infant can eat bananas now. The outcome statement on the infants plan of care states "The mother will explain proper nutrition for infants." this is an example of what type of outcome statement? a) Physical changes b) Affective c) Psychomotor d) Cognitive

d) Cognitive

A client has just learned that she has Stage 2 breast cancer. She appears distant and withdrawn. Her shoulders are slumped. She explains "I just never thought this could happen to me." Which answer best describes the client's response? a) Verbal communication b) Congruent communication c) Nonverbal communication d) Incongruent communication

b) Congruent communication The client's actions, expressions, and words all "fit together" following the receipt of a difficult diagnosis. Her communication would be incongruent in this same scenario if she was described as "beaming from ear to ear." Verbal communication is not the best choice because it does not take into account the whole picture. Although verbal communication is used as a part of the client's response, the nurse must also consider her posture and her facial expressions. Nonverbal communication is not the best choice either because it does not take into account the whole picture. Although nonverbal communication is used as a part of the client's response, the nurse must also consider the client's words.

A nurse in the emergency room, who is unfamiliar with pediatric clients, assesses the vital signs of a 1-month old infant with a heart rate of 124 and a respiratory rate of 36. What would be the most appropriate measure for the nurse to take to analyze the significance of the infant's vital signs? a) Perform a complete physical assessment to determine the cause of the elevated vital signs. b) Consult reference materials to determine the normal vital signs for 1-month old infants. c) Ask the mother if the infant's heart rate is higher than normal. d) Report the vital signs and allow the emergency room physician to determine the significance.

b) Consult reference materials to determine the normal vital signs for 1-month old infants.

When deciding what information the patient needs to meet the learner objectives successfully, the nurse is planning which part of the teaching plan? a) Teaching strategies b) Content c) Learning activities d) Learning domains

b) Content The nurse is planning the content when the nurse decides what information the client needs to meet the learner objectives successfully. The teaching strategies would be what the nurse would include to be sure the teaching was effective. The learning activities would be designed by the nurse to meet the needs of the client. Learning domains, including cognitive, affective, psychomotor, are the different types of learning.

A client has returned to the clinic for a postoperative visit. The nurse reviews the plan of care and could choose to take which action based on the client's previous responses to the current plan of care? Select all that apply. a) Explain the plan of care to significant others and advise them that there is an expectation that the client will achieve outcomes within a reasonable amount of time. b) Continue the plan of care if more time could result in achievement of outcomes. c) Terminate the plan of care if outcomes have been achieved. d) Modify the plan of care if difficulty has been encountered with achieving outcomes.

b) Continue the plan of care if more time could result in achievement of outcomes. c) Terminate the plan of care if outcomes have been achieved. d) Modify the plan of care if difficulty has been encountered with achieving outcomes.

Which of the following is not a purpose of the medical record? a) Reimbursement b) Contract c) Legal document d) Care planning

b) Contract Medical records are legal documents, communication tools, and assessment tools. They are used for care planning purposes, quality assurance purposes, for reimbursement, research, and education.

A student nurse is attempting to improve her communication skills. Which of the following is an appropriate therapeutic communication skill? a) Use cliches to enhance a patient's understanding of information. b) Control the tone of the voice to avoid hidden messages. c) Be precise and inflexible regarding the intent of the conversation. d) Avoid the use of periods of silence.

b) Control the tone of the voice to avoid hidden messages. Conversation skills involve controlling the tone of one's voice so that exactly what is intended is conveyed and there is no hidden message. Periods of silence have an important role in conversations because they allow for periods of reflection. Cliches should be avoided, and the conversation should be flexible.

According to the Health Insurance Portability and Accountability Act of 1996, if a health institution wants to release a patient's health information (PHI) for purposes other than treatment, payment, and routine health care operations, the patient must be asked to sign an authorization. The nurse is aware that there are exceptions to this requirement. In which of the following cases is an authorization form not needed? Select all that apply. a) News media are preparing a report on the condition of a public figure. b) Data are needed for the tracking and notification of disease outbreaks. c) Protected health information is needed by a coroner. d) Child abuse and neglect are suspected. e) Protected health information is needed to facilitate organ donation. f) The sister of a patient with Alzheimer's wants to help provide care.

b) Data are needed for the tracking and notification of disease outbreaks. c) Protected health information is needed by a coroner. d) Child abuse and neglect are suspected. e) Protected health information is needed to facilitate organ donation. Authorization is not required for tracking disease outbreaks, providing PHI to a coroner, reporting incidents of child abuse, or facilitating organ donations. Under no circumstance can a nurse provide information to a news reporter without the patient's express authorization. An authorization form is still needed to provide PHI for a patient who has Alzheimer's disease.

A nurse has taught a diabetic patient how to administer his daily insulin. The nurse should evaluate the teaching-learning process by: a) Determining the patient's motivation to learn b) Deciding if the learning outcomes have been achieved c) Allowing the patient to practice the skill he has just learned d) Documenting the teaching session in the patient's medical record

b) Deciding if the learning outcomes have been achieved The nurse cannot assume that the patient has actually learned the content unless there is some type of proof of learning. The key to evaluation is meeting the learner outcomes stated in the teaching plan.

A nurse has taught a diabetic patient how to administer his daily insulin. The nurse should evaluate the teaching-learning process by: a) Determining the patient's motivation to learn. b) Deciding if the learning outcomes have been achieved. c) Allowing the patient to practice the skill he has just learned. d) Documenting the teaching session in the patient's medical record.

b) Deciding if the learning outcomes have been achieved. The nurse cannot assume that the patient has actually learned the content unless there is some type of proof of learning. The key to evaluation is meeting the learner outcomes stated in the teaching plan.

The nurse is performing a routine assessment of a male patient who has an artificial arm as a result of a small plane crash many years earlier. How should the nurse best understand this patient's health? a) The loss of his limb prevents the patient from achieving wellness, though he may be healthy. b) Despite the loss of his limb, the patient may consider himself to be health. c) The patient may be well, but his loss of limb means that he is unhealthy. d) Because the patient's injury is far in the past, it does not have a bearing on his health or wellness.

b) Despite the loss of his limb, the patient may consider himself to be health. Individuals who live with chronic conditions, such as the loss of a limb, may accommodate their condition fully and consider themselves to be healthy and well. This is not a certainty, however, and the passage of time does not guarantee such acceptance.

A couple has just learned that their newborn infant has a congenital cardiac anomaly that will require many lifestyle modifications, surgical corrections, and hospital stays. Place the following aspects of the couple's patient education in the correct order that nurses should conduct them. a) Draft learning outcomes. b) Determine their emotional readiness to learn. c) Implement various educational techniques. d) Revise the learning plan if needed. e) Select teaching strategies. f) Determine what the couple has learned.

b) Determine their emotional readiness to learn. a) Draft learning outcomes. e) Select teaching strategies. c) Implement various educational techniques. f) Determine what the couple has learned. d) Revise the learning plan if needed. The teaching process parallels the learning process. It begins with assessment of variables such as readiness to learn and learning needs that lead to a determination of learning outcomes. Teaching strategies are selected, implemented, evaluated, and then revised if necessary.

A patient states that his recent fall was caused by the fact that his scheduled antihypertensives were mistakenly administered by two different nurses, an event that is disputed by both of the nurses identified by the patient. Which of the following measure should the nurses prioritize when anticipating that legal action may follow? a) Enlist support from nursing and nonnursing colleagues from the unit. b) Document the patient's claims and the events surrounding the alleged incident. c) Consult with practice advisors from the state board of nursing. d) Liaise with the hospital's legal department as soon as possible.

b) Document the patient's claims and the events surrounding the alleged incident. It is imperative for nurses to carefully and accurately document assessment findings and the nursing care that they provide, a fact that is especially salient when legal action is anticipated. This thorough and accurate assessment should precede consultation with the legal department, the state board of nursing, and colleagues.

A nurse is documenting the care given to a 56-year-old patient diagnosed with an osteosarcoma, whose right leg was amputated. The nurse accidentally documents that a dressing changed was performed on the left leg. What would be the best action of the nurse to correct this documentation? a) Erase or use correcting fluid to completely delete the error. b) Draw a single line through the entry and rewrite it above or beside it. c) Use a permanent marker to block out the mistaken entry and rewrite it. d) Remove the page with the error and rewrite the data on that page correctly.

b) Draw a single line through the entry and rewrite it above or beside it.

Which qualities in a nurse help the nurse to become effective in providing for a client's needs while remaining compassionately detached? a) Commiseration b) Empathy c) Sympathy d) Kindness

b) Empathy

Which of the following nursing actions best exemplifies the nurse's role in promoting health? a) Performing deep suctioning on a patient who has a tracheotomy and copious secretions. b) Encouraging a group of junior high school students to engage in regular physical activity. c) Facilitating a support group for the friends and families of patients affected by stroke. d) Administering a beta-adrenergic blocker and diuretic to a patient who has a history of hypertension.

b) Encouraging a group of junior high school students to engage in regular physical activity Health promotion involves encouraging individuals to be proactive in taking measures to foster their well-being and maximize optimal functioning. Exercise is one of the most common, and accessible, activities that can help achieve this. Support groups often focus on coping, while treating hypertension pharmacologically and removing secretions that can threaten oxygenation help to restore health and prevent illness.

The nurse is visiting a hospice client in his home. He is explaining the difficulties he is having with his home infusion pump. By making statements such as "I see" and "go on" during the conversation, the nurse is utilizing which therapeutic nurse-client communication technique? a) Clarification b) Encouraging elaboration c) Restating d) Reflection

b) Encouraging elaboration

In preparing for a patient's admission to the unit, what is the nurse's responsibility? a) Ensuring that all staff caring for the patient are in the patient's room when he/she arrives onto the unit. b) Ensuring that room preparation responsibilities that may have been delegated to ancillary staff have been completed c) Greeting the patient in the emergency room or admitting office d) Delegating the admission assessment to a nursing assistant

b) Ensuring that room preparation responsibilities that may have been delegated to ancillary staff have been completed Although the nurse might delegate most of the activities in preparing the room for admission, it is the nurse's responsibility to ensure that the other personnel complete the preparation. It is not necessary for all care staff to be present when the patient arrives and, in fact, it might be quite overwhelming to the patient to have them all present. The nurse will greet the patient and family members upon their arrival to the unit. An admission assessment is the responsibility of the nurse, not a nursing assistant, who is not educated to perform the skill.

A nurse is providing care based on Maslow's hierarchy of basic human needs. For which of the following nursing activities is this approach useful? a) Making accurate nursing diagnoses. b) Establishing priorities of care. c) Communicating concerns more concisely. d) Integrating science into nursing care.

b) Establishing priorities of care. Maslow's hierarchy of basic human needs is useful for establishing priorities of care.

A nurse is arranging for home care for clients and reviews the Medicare reimbursement requirements. Which client meets one of these requirements? a) a client whose rehabilitation potential is not good b) a client who is homebound and needs skilled nursing care c) a client who is not making progress in expected outcomes of care d) a client whose status is stabilized

b) a client who is homebound and needs skilled nursing care

Which of the following statements best conveys the concept of ethical agency? a) A nurse's understanding and execution of ethical practice is primarily a result of increased years of experience. b) Ethical practice requires a skill set that must be conscientiously learned and nurtured. c) Individuals who enter the nursing profession often innately possess ethical characteristics. d) Ethical practice is best learned and fostered by surrounding oneself with people who exhibit ethical character.

b) Ethical practice requires a skill set that must be conscientiously learned and nurtured. Ethical agency must be cultivated in the same way that nurses cultivate the ability to do the scientifically right thing in response to a physiologic alteration. It is inaccurate to assume that it will passively develop from the presence of other ethical practitioners or from years of experience and it is not an innate characteristic of personality.

Once a nurse has collected and interpreted the data on a client's outcome achievement, the nurse will then make a judgment and document a statement summarizing those findings. This is called which of the following? a) Evidence-based practice b) Evaluative statement c) Standard d) Criteria

b) Evaluative statement

Nurses play a vital role in patient teaching because of their: a) Need for self-actualization b) Expertise in healthcare c) Desire to help others d) Ability to provide illness care

b) Expertise in healthcare Nurses who are skilled educators and counselors can improve patients' health and well-being. This is a function of their education and experience and their use of cognitive, technical, interpersonal, and ethical/legal skills.

A patient has been admitted to the hospital the morning of his scheduled transurethral prostatic resection (TUPR). The Health Insurance Portability and Accountability Act of 1996 (HIPAA) requires the care team to do which of the following upon the patient's admission? a) Explain the potential risks and benefits of the patient's scheduled surgery and have him sign a document acknowledging these. b) Explain how the patient's health information will be protected and obtain his signature acknowledging this teaching. c) Teach the patient what measures are taken to ensure that all members of the care team are accountable for the care they provide. d) Explain to the patient that his choice of health insurance provider and level of coverage will not impact the care he receives.

b) Explain how the patient's health information will be protected and obtain his signature acknowledging this teaching. HIPAA requires that all patients entering a healthcare setting of any type must sign a statement that they understand the protection of their medical information. The informed consent process is not within the scope of HIPAA and the care team is not obliged to explain the specific measures taken to foster accountability for practice. Variations in health insurance coverage are known to have tangible effects on the care that patients receive.

In caring for a patient, the nurse recognizes which of the following as the primary educational and support structure for an individual? a) Teachers and coaches. b) Family members. c) Clergy. d) Peers

b) Family members: The family is the primary educational and support structure for an individual. The family, as a social unit, provides the environment and relationships necessary for members to meet their basic human needs.

A nurse is caring for a client 72 years of age with arthritis. Which action is the highest priority for the nurse when conducting the health education for the client? a) Identify how long the education session will last. b) Find out what the client wants to know. c) Divide information into manageable amounts. d) Provide an environment that promotes learning.

b) Find out what the client wants to know. Finding out what the client wants to know helps the nurse in showing personal interest, which facilitates better learning to an adult client. Dividing information into manageable amounts, providing an environment that promotes learning, and identifying how long the education session will last can be done only when the assessment of the client is completed.

A team of nurse educators are preparing to implement an evidence-based practice (EBP) change in the way that change-of-shifts reports are organized and conducted at their hospital. Place the following stages of the EBP process that the educators will follow in the correct sequence. a) Critically appraise the evidence. b) Formulate a clear clinical question. c) Integrate evidence with clinical expertise and patient preferences. d) Collect the most relevant and best evidence. e) Evaluate the practice decision or change.

b) Formulate a clear clinical question. d) Collect the most relevant and best evidence. a) Critically appraise the evidence. c) Integrate evidence with clinical expertise and patient preferences. e) Evaluate the practice decision or change.

A patient has a complex medical history involving the consequences of type 1 diabetes. As a result of his diabetic nephropathy, he is now a patient of the local hospital's dialysis program and he has been referred to an ophthalmologist by his primary care physician following the onset of vision problems. In addition, he receives homecare nursing for the treatment of a foot ulcer that is slow to heal. This patient's situation characterizes which of the following phenomena? a) Managed care b) Fragmentation of care c) Case management d) Primary care

b) Fragmentation of care Fragmentation of care occurs when multiple, specialized practitioners are involved in various aspects of a complex patient's care. This creates the potential for miscommunication and conflicting advice and treatment with the lack of a unified plan of care. This situation is not indicative of primary care and it is not necessarily a consequence or manifestation of case management and managed care.

An ethical conflict exists around a female patient's expressed desire to have a neighbor make her treatment decisions, an individual who the patient's children characterize as a predator. Place the steps that the nurse should follow in resolving this ethical conflict in the correct order. a) Make and evaluate a decision b) Gather relevant data bout the situation c) Apply ethical principles to the situation d) Identify the different options e) Clearly identify the ethical problem

b) Gather relevant data about the situation e) Clearly identify the ethical problem d) Identify the different options c) Apply ethical principles to the situation a) Make and evaluate a decision The nursing process of assessment, diagnosis, planning, implementation, and evaluation can be applied to appropriately respond to many ethical dilemmas.

Of the following types of qualitative research, which method developed in anthropology? a) historical b) ethnography c) grounded theory d) phenomenology

b) ethnography Ethnographic research was developed by the discipline of anthropology and is used to examine issues of culture of interest to nursing.

A 79-year-old male patient who is postoperative day three following hip replacement surgery has been approached by a nurse researcher and asked to participate in a research study that will test a new rehabilitation strategy. What aspect of the nursing research process addresses the patient's understanding of the potential risks and benefits of this study? a) Obtaining the patient's witnesses signature on a permission document. b) Going through the informed consent process with the patient. c) Giving the patient the opportunity to ask questions about the study. d) Meeting with the hospital's institutional review board (IRB).

b) Going through the informed consent process with the patient. Informed consent is a process in which the details of the study and the patient's rights are explained and discussed in detail. This goes beyond simply obtaining the patient's signature or allowing him to ask questions. Meeting with the IRB is a necessary step in the research process but this does not directly affect the patient's understanding of risks and benefits.

Of the following statements, which is most true of health and illness? a) Health and illness are the same for all people. b) Health and illness are individually defined by each person. c) People with acute illnesses are actually healthy. d) People with chronic illnesses have poor health beliefs.

b) Health and illness are individually defined by each person. Each person defines health and illness individually, based on a number of factors.

The nurse is preparing for the admission of a patient on a stretcher. In what position should the nurse place the bed? a) Middle position b) Highest position c) Lowest position d) High-Fowler's position

b) Highest position Place the bed in the highest position, if the patient will arrive on a stretcher. For ambulatory patients, the bed should be in the lowest position. The High-Fowler's position is often used for patients with respiratory difficulties.

Mr. Singh's wife has recently been diagnoses with early-stage Alzheimer's disease and he has asked the nurse to recommend websites that may supplement his learning about her diagnosis. How should the nurse respond to Mr. Singh's request? a) Direct Mr. Singh to online databases such as Cumulative Index to Nursing and Allied Health Literature (CINAHL) b) Identify and recommend some credible websites appropriate to his learning needs. c) Provide Mr. Singh with print-based materials that are clearly referenced and reflect his learning style. d) Encourage Mr. Singh to avoid online resources due to the unregulated nature of the internet.

b) Identify and recommend some credible websites appropriate to his learning needs. Web-based resources can be a useful tool in patient education, but suitability and accuracy need to be carefully considered. Many online databases exist for health professionals, but these are less likely to be accessible or appropriate for those outside the health professions.

In the development of a nursing diagnosis for a client who has cachexia and decreased weight, what would be an appropriate nursing diagnosis? a) Lack of adequate nutrition related to decreased calories b) Imbalanced nutrition: less than body requirements c) Anorexia nervosa and bulimia d) Weight loss related to abdominal discomfort

b) Imbalanced nutrition: less than body requirements

A nurse is providing instruction to a patient regarding the procedure to change his colostomy bag. During the teaching session, he asks, "What type of foods should I avoid to prevent gas?" The patient's question allows for what type of communication? a) A closed-ended answer b) Information clarification c) The nurse to give advice d) Assertive behavior

b) Information clarification

Two nurses have disagreed about the role in intuition in nursing practice, with one nurse characterizing it as "hocus-pocus" and the other nurse advocating it as a superior problem-solving strategy. Which of the following statements best conveys the role of intuition in nurses' problem solving? a) In experienced nurses, intuition can be a valid replacement for scientific problem solving. b) Intuition can be a clinically useful adjunct to logical problem solving. c) Intuition is an unreliable mode of thinking that should be avoided. d) Intuition is reliable when those nurses implementing it have a special gifting.

b) Intuition can be a clinically useful adjunct to logical problem solving. Creative, intuitive thinking can be useful supplements to more "in-the-box" methods of problem solving. While it should not be discouraged outright, it should also not be thought of as a replacement for logical or scientific problem solving. Intuition is not dependent on a special gifting, but is thought to be a product of experience and unconscious patter recognition.

A nurse manager notes an increase in the frequency of client falls during the last month. To promote a positive working environment, how would the nurse manager most effectively deal with this problem? a) Reprimand the nursing personnel responsible for the clients when the falls occurred. b) Investigate the circumstances that contributed to client falls. c) Institute a new policy on the prevention of client falls on the unit. d) Determine if client falls have increased on other nursing units in the hospital.

b) Investigate the circumstances that contributed to client falls.

A class of nursing students is studying the history of nursing. Who does the class learn that the founder of public health nursing is? a) Dorothea Dix b) Lillian Wald c) Clara Barton d) Linda Richards

b) Lillian Wald Lillian Wald established a neighborhood nursing service for the sick poor of the Lower East Side in New York City and was the founder of public health nursing. Clara Barton established the Red Cross in the United States in 1882. Linda Richards was the first trained nurse in the United States. Dorothea Dix was a pioneering crusader for the reform of the treatment of the mentally ill.

Which healthcare setting provides care with activities of daily living for people of any age who are physically or mentally unable to care for themselves independently? a) Homeless shelters b) Long-term care facilities c) Rural health centers d) Ambulatory care centers

b) Long-term care facilities Long-term care facilities provide healthcare and help with the activities of daily living for people of any age who are physically or mentally unable to care for themselves independently. This care may extend for periods ranging from days to years. Rural and ambulatory care centers provide primary care services while homeless shelters provide housing for people who do not have regular shelter.

When looking at a model for evidence-based practice, what is the final step of the process? a) appraising evidence b) evaluating practice change c) searching the literature d) formulating a clinical question

b) evaluating practice change The fifth and final step in the process of implementing evidence-based practice is to evaluate and critically appraise the change in practice. Formulating a clinical question and searching and appraising the literature precede this step.

The implementation of diagnosis-related groups (DRGs) by Medicare in 1983 affected hospitals in which of the following ways? a) Benefits and reimbursement cannot be changed annually. b) Medicare pays only the amount of money preassigned to a treatment for a diagnosis. c) Part B of Medicare covers most inpatient and outpatient costs. d) Part A of Medicare is voluntary and is paid for by a monthly premium.

b) Medicare pays only the amount of money preassigned to a treatment for a diagnosis. The implementation of diagnosis-related groups (DRGs) by Medicare in 1983 pays the hospital a fixed amount that is predetermined by the medical diagnosis or specific procedure, rather than the actual cost of hospitalization and care. The plan pays only the amount of money preassigned to the treatment for a diagnosis. If the cost for hospitalization is greater than that assigned, the hospital must absorb the additional cost. If the cost is less than that assigned, the hospital makes a profit. Medicare is federally funded and benefits may change annually according to decisions related to the federal budget. Part A of Medicare, which pays most inpatient hospital costs, is paid by the federal government. Part B, which is voluntary, is paid by a monthly premium and covers most outpatient costs.

A student nurse is anticipating graduation and is aware of the changing nature of healthcare and nursing practice. Which of the following statements most accurately describes an aspect of contemporary nursing practice in the United States? a) Most nurses work in long-term care facilities. b) Most nurses are still employed in hospitals. c) Most nurses now provide care in outpatient settings. d) Most nurses provide home healthcare.

b) Most nurses are still employed in hospitals. While the percentage of nurses who are employed in hospitals decline, 59% of nurses are still employed in a hospital setting.

The primary purpose of a contractual agreement between nurses and patients when beginning a teaching plan is to: a) Create a formal and legal bond between the nurse and patient. b) Motivate both the patient and nurse to do what is necessary to meet the patient's learning outcomes. c) Outline the patient's learning outcomes. d) Limit the scope of the teaching session.

b) Motivate both the patient and nurse to do what is necessary to meet the patient's learning outcomes. A contractual agreement is a pact two people make setting out mutually agreed-on goals.

A nurse is discussing the benefits of smoking cessation with a client. The nurse informs the client that smoking cessation will reduce his risk for cancer, improve his respiratory status, and enhance the quality of his life. The nurse also shares her story of smoking cessation, provides information on other individuals who have successfully quit, and encourages the client to attend a support group for smoking cessation. The client discusses his feelings on smoking cessation and verbalizes a desire to quit smoking. What type of counseling did the nurse provide to this client? a) Developmental counseling b) Motivational counseling c) Long-term counseling d) Situational counseling

b) Motivational counseling

Mr. S was diagnosed with type 2 diabetes mellitus 3 years ago, but has failed to integrate regular blood glucose monitoring or dietary modifications into his lifestyle. He has been admitted to the hospital for treatment of acute renal failure secondary to diabetic nephropathy, an event that has prompted Mr. S. to reassess his values. Which of the following actions most clearly demonstrates that Mr. S. is engaging in the step of prizing within his valuing process? a) Mr. S. states that he will now begin to check his blood glucose before each meal and at bedtime. b) Mr. S. expresses pride that he now has the knowledge and skills to take control of his diabetes management. c) Mr. S. is now able to explain how his choices have contributed to his renal failure. d) Mr. S. expresses remorse at how his failure to make lifestyle changes has adversely affected his health.

b) Mr. S. expresses pride that he now has the knowledge and skills to take control of his diabetes management. Within the value process, expressions of pride and happiness are considered to be indications of prizing. Resolving to make changes is an aspect of choosing while expressing insight about his role in his current diagnosis demonstrates that Mr. S. has the desire to re-examine his values.

A family that consists of two homosexual parents and three children living in the same house is an example of which type of family? a) Blended. b) Nuclear. c) Single-parent. d) Extended

b) Nuclear: The nuclear family is also known as the traditional family and is composed of two parents and their children. The parents might be heterosexual or homosexual, are often married or in a committed relationship, and all members of the family live in the same house until the children leave home as young adults.

After reporting to work for a night shift, Nurse L. learns that the unit will be understaffed because two RNs called out sick. As a result, each nurse on the unit will need to provide care for an additional four acute patients in addition to his or her regular patient assignment. Which of the following statements is true of Nurse L. when working in understaffed circumstances? a) Nurse L. must document that float staff, nurses on overtime, and part-time staff were contacted in an effort to fill the gaps in care. b) Nurse L. will be legally held to the same standards of care as when staffing levels are normal. c) Nurse L. is legally obliged to refuse to provide care when understaffing creates the potential for unsafe conditions. d) Understaffing constitutes an extenuating circumstance that creates a temporarily lower expectation for care for Nurse L.

b) Nurse L. will be legally held to the same standards of care as when staffing levels are normal. The claim of being overworked does not constitute a legal defense, and both the potential for liability and standards of care remain unchanged despite and increased patient assignment. While it is prudent to make all realistic attempts to fill the gaps in staffing, documenting these efforts does not change Nurse L.'s legal position. A nurse has the right to refuse an unsafe patient assignment but the nurse is not legally obliged to withhold care.

What guidelines do nurses follow to identify the patient's healthcare needs and strengths, to establish and carry out a plan of care to meet those needs, and to evaluate the effectiveness of the plan to meet established outcomes? a) Nurse Practice Acts b) Nursing process c) ANA Standards of Professional Performance d) Evidence-based practice guidelines

b) Nursing process The nursing process is one of the major guidelines for nursing practice. Nurses implement their roles through the nursing process. The nursing process is used by the nurse to identify the patient's healthcare needs and strengths, to establish and carry out a plan of care to meet those needs, and to evaluate the effectiveness of the plan to meet established outcomes.

The client identifies three strategies for minimizing leakage of an ileostomy bag. This is an example of what type of outcome? a) Physiologic outcome b) Affective outcome c) Psychomotor outcome d) Cognitive outcome

d) Cognitive outcome

This text is based upon a notion of blended skills. Simply described, what does this mean? a) Nursing works best when nurses competently use the intellectual and technical skills that achieve patient outcomes. Nursing has been held back by outdated notions of care and compassion (interpersonal skills), which can be done by anyone. b) Nursing works best when each nurse competently uses the intellectual, interpersonal, technical, and ethical/legal skills demanded by each situation. c) All of the blended skills are important, but not every nurse has to be skilled in each area. We benefit patients by knowing what we do best. I might be utterly deficient in interpersonal skills but excellent in intellectual skills. d) Every nursing situation demands the same blend of basic nursing skills, intellectual, technical, interpersonal, and ethical/legal.

b) Nursing works best when each nurse competently uses the intellectual, interpersonal, technical, and ethical/legal skills demanded by each situation. Every nurse must be competent in all four basic skill areas and judge what skills each situation needs. Each situation is unique and might call for a different "blend" of basic skills.

Which of the following is one element of a healthy community? a) Meets all the needs of its inhabitants b) Offers access to healthcare services c) Has mixed residential and industrial areas d) Is little concerned with air and water quality

b) Offers access to healthcare services A healthy community offers access to healthcare services to treat illness and to promote health.

The nurse is creating a nursing care plan for a patient who has numerous needs in the various dimensions of his life. As a result, the nurse is faced with the challenge of prioritizing certain needs over others. Place the following levels of needs in the correct ascending order. a) Self-esteem needs b) Physiologic needs c) Self-actualization needs d) Love and belonging e) Safety and security needs

b) Physiologic needs e) Safety and security needs d) Love and belonging needs a) Self-esteem needs c) Self-actualization needs

After assessing a patient who is recovering from a stroke in a rehabilitation facility, a nurse interprets and analyzes the patient data. Which of the four basic conclusions has the nurse reached when identifying the need to collect more data to confirm a diagnosis of situational low self-esteem? a) No problem b) Possible problem c) Actual nursing diagnosis d) Clinical problem other than nursing diagnosis

b) Possible problem When a possible problem exists, such as situational low self-esteem related to effects of stroke, the nurse must collect more data to confirm or disprove the suspected problem. The conclusion "no problem" means no nursing response is indicated. When an actual problem exists, the nurse begins planning, implementing, and evaluating care to prevent, reduce, or resolve the problem. A clinical problem other than nursing diagnosis requires that the nurse consult with the appropriate health care professional to work collaboratively on the problem.

Which principle should guide the nurse's documentation of entries on the client's medical record? a) Documentation does not include photographs. b) Precise measurements should be used rather than approximations. c) Correcting fluid is used rather than erasing errors. d) Nurses should not refer to the names of physicians.

b) Precise measurements should be used rather than approximations. Precise measurements and times must be used whenever possible. It is appropriate to use the names of physicians, and photographs can constitute documentation. Handwritten entries should be struck through with a single line, not covered with correcting fluid or erased.

A nurse is teaching first aid to counselors of a summer camp for children with asthma. This is an example of what aim of health teaching? a) Promoting health b) Preventing illness c) Restoring health d) Facilitating coping

b) Preventing illness Teaching first aid is a function of the goal to prevent illness. Promoting health involves helping patients to value health and develop specific health practices that promote wellness. Restoring health occurs once a patient is ill, and teaching focuses on developing self-care practices that promote recovery. When facilitating coping, nurses help patients come to terms with whatever lifestyle modification is needed for their recovery or to enable them to cope with permanent health alterations.

Mr. Young considers himself affluent and places a high priority on maintaining his freedom to choose which physicians and healthcare services he utilizes. Which of the following group plans is most likely to meet Mr. Young's needs? a) Long-term care insurance b) Private insurance c) A preferred provider organization (PPO) d) A health maintenance organization (HMO)

b) Private insurance Private insurance is normally more expensive than alternative group plans such as PPOs, HMOs, and LTC insurance. However, it typically allows the policyholder to have significantly more autonomy around which practitioners and services to utilize.

A nurse is part of a team that will be working in a new orthopedic unit to determine the most appropriate method for documentation. The team agrees to initiate the practice of an abbreviated form of documentation that requires less nursing time and readily detects changes in client status. Which documentation method would the group most likely suggest? a) FOCUS data, action, and response note b) Problem, intervention, and evaluation note c) Charting by exception d) Narrative notes

b) Problem, intervention, and evaluation note

In preparation for discharge, the nurse is reviewing information related to new dietary guidelines with the patient. This is an example of which step in discharge planning? a) Making home healthcare referrals b) Providing patient teaching c) Developing goals with the patient d) Assessing the patient's needs and identifying problems

b) Providing patient teaching The nurse is teaching the patient important information about self-care at home prior to the patient's discharge. The initial step in discharge planning is collecting and organizing data about the patient, as this provides information on the patient's healthcare needs. Home referrals may be made after the teaching process, based upon orders provided by the physician. Developing goals may occur after the teaching process, as the goals need to be realistic.

A nurse is evaluating the outcome of the plan of care after teaching a client how to prepare and administer an insulin pen. Which of the following types of outcome is the nurse addressing? a) Cognitive b) Psychomotor c) Physiological d) Affective

b) Psychomotor

The primary purpose for evaluating data about a client's care according to a functional health approach is to: a) determine implementation of medical orders. b) revise or modify the nursing care plan. c) meet accreditation standards. d) evaluate the need for health care consultations.

b) revise or modify the nursing care plan.

After one nursing unit with an excellent safety record meets to review the findings of the audit, the nurse manager states, "We're doing well, but we can do better! Who's got an idea to foster increased patient well-being and satisfaction?" This is an example of leadership that values: a) Quality assurance b) Quality improvement c) Process evaluation d) Outcome evaluation

b) Quality improvement Unlike quality assurance, quality improvement is internally driven, focuses on patient care rather than organizational structure, focuses on processes rather than individuals, and has no end points. Its goal is improving quality rather than assuring quality. Answers c and d are types of quality-assurance programs.

A nurse assesses a patient and formulates the following nursing diagnosis: Risk for Impaired Skin Integrity related to prescribed bedrest as evidenced by reddened areas of skin on the heels and back. Which phrase represents the etiology of this diagnostic statement? a) Risk for Impaired Skin Integrity b) Related to prescribed bedrest c) As evidenced by d) As evidenced by reddened areas of skin on the heels and back

b) Related to prescribed bedrest "Related to prescribed bedrest" is the etiology of the statement. The etiology identifies the contributing or causative factors of the problem. "Risk for Impaired Skin Integrity" is the problem, and "as evidenced by reddened areas of skin on the heels and back" are the defining characteristics of the problem.

The nurse is sharing information about a client at change of shift. The nurse is performing what nursing action? a) Verification b) Reporting c) Documentation d) Dialogue

b) Reporting

A nurse makes a clinical judgment that an African American male patient in a stressful job is more vulnerable to developing hypertension than White male patients in the same or similar situation. The nurse has formulated what type of nursing diagnosis? a) Actual b) Risk c) Possible d) Wellness

b) Risk A clinical judgment that an individual, family, or community is more vulnerable to develop the problem than others in the same or similar situation is a Risk nursing diagnosis.

The community health nurse is creating a plan of care for a patient with Parkinson's disease. The patient's spouse has provided care to the patient for the past 5 years and the patient's care needs are increasing. The nurse determines that an appropriate nursing diagnosis for the patient and family is: a) Parental Role Conflict. b) Risk for Caregiver Role Strain. c) Readiness for Enhanced Family Processes. d) Health Seeking Behaviors

b) Risk for Caregiver Role Strain: Long-term care of a family member with a chronic illness may lead to caregiver role strain, so the most appropriate nursing diagnosis is "Risk for Caregiver Role Strain"

Which statement appropriately identifies an at-risk nursing diagnosis for a woman 78 years of age who is confined to bed? a) Ineffective airway clearance related to bed rest b) Risk for impaired skin integrity related to bed rest c) Immobility related to confinement to bed d) Potential for pneumonia related to inactivity

b) Risk for impaired skin integrity related to bed rest

Which expected client outcome is an example of a psychomotor outcome? Select all that apply. a) Reporting increased confidence in testing blood sugar. b) Safely ambulating using a walker. c) Identifying signs and symptoms of infection. d) Rating pain as a 2 on a 10-point scale. e) Accurately drawing up insulin.

b) Safely ambulating using a walker. e) Accurately drawing up insulin.

A client has just been admitted to the clinical unit. The nurse is providing her with the expectations she may have of the health care she will receive. She is told that she will not be harmed by any errors that might be made and she can expect to be safe in the facility. This assurance represents which expectation of the health care environment? a) Transparency b) Safety c) Control d) Individualization

b) Safety

A pediatric nurse provides education to numerous clients in her care. Which group of children benefits most from being involved in the teaching-learning process? a) Infants b) School-age children c) Toddlers d) Preschoolers

b) School-age children

A nurse is caring for a patient with heart failure, a chronic illness. Which of the following characteristics is not a part of chronic illness? a) Permanent change in body structure or function. b) Self-treatment that relieves symptoms. c) Long period of treatment and care. d) Often has remissions and exacerbations.

b) Self-treatment that relieves symptoms. Acute illnesses are often self-treated to relieve symptoms.

A nurse is caring for a 42-year-old male patient who is admitted to the hospital with injuries sustained in a motor vehicle accident. While he is in the hospital, his wife tells him that the bottom level of their house flooded, damaging their belongings. When the nurse enters his room, she notes that the patient is visibly upset. The nurse is aware that the patient will most likely be in need of which type of counseling? a) Long-term developmental b) Short-term situational c) Short-term motivational d) Long-term motivational

b) Short-term situational Short-term counseling might be used during a situational crisis, which occurs when a patient faces an event or situation that causes a disruption in life, such as a flood. Long-term counseling extends over a prolonged period; a patient experiencing a developmental crisis, for example, might need long-term counseling. Motivational interviewing is an evidence-based counseling approach that involves discussing feelings and incentives with the patient. A caring nurse can motivate patients to become interested in promoting their own health.

The parents raising two school-aged children incorporate their religious beliefs into the family's daily life. The family's beliefs regarding religion include dietary considerations, worship practices, attitudes, and values. This is an example of which function of the family? a) Reproductive. b) Socialization. c) Affective and coping. d) Physical

b) Socialization: Through socialization, the family teaches; transmits beliefs, values, attitudes, and coping mechanisms; provides feedback; and guides problem solving. Incorporating religious beliefs, values, and attitudes is an example of socialization.

The nurse is working with Ms. V. today. Ms. V. is having a difficult time accepting her new diagnosis of type 2 diabetes. Thenurse pulls up a chair next to Ms. V.'s bed and holds her hand while listening to her story. What type of nursing intervention is the nurse engaging in? a) Psychosocial intervention b) Supportive intervention c) Supervisory intervention d) Coordinating intervention

b) Supportive intervention

A client, eager to go home from an acute care facility, calls out to have discharge education completed. The nurse is not able to get to the client's room until an hour later, and finds the client asleep. The client's significant other states, "She will be out for a couple hours after that pain medication." Which of the following best describes what must happen with the education session? a) The nurse cannot determine the subject matter to teach if the client is asleep. b) The client is not demonstrating readiness to learn due to the effects of medication. c) The client can be awakened when she falls asleep during the session. d) The significant other can be taught now, and then teach the client later on.

b) The client is not demonstrating readiness to learn due to the effects of medication. The nurse knows the client's plan of care and should know what needs to be taught to this individual. The client is experiencing a side effect of pain medication and is not ready to be taught at this time. The nurse, not the significant other, is responsible for teaching the client about self-care. If the client cannot remain awake, then the client is not able to learn.

A nurse is developing a contractual agreement with a client. Which statement is true of a contractual agreement? a) The contract is a formal agreement. b) The contract serves to meet the client's learning outcomes. c) The contract serves to meet nursing goals. d) The contract is legally binding.

b) The contract serves to meet the client's learning outcomes. A contractual agreement is a pact between two people setting out mutually agreed-upon goals. The contracts are usually informal and not legally binding. The contract can serve to motivate the client and nurse to do what is necessary to meet the learning needs of the client; the contract does not serve to meet nursing goals.

A nurse is developing a contractual agreement with a patient. Which of the following statements is true of a contractual agreement? a) The contract serves to meet nursing goals. b) The contract serves to meet the patient's learning outcomes. c) The contract is legally binding. d) The contract is a formal agreement.

b) The contract serves to meet the patient's learning outcomes. A contractual agreement is a pact between two people setting out mutually agreed-on goals. The contracts are usually informal and not legally binding. The contract can serve to motivate the patient and nurse to do what is necessary to meet the learning needs of the patient and not nursing goals.

When a nurse is planning for learning, who must decide who should be included in the learning sessions? a) The client and the client's family b) The nurse and the client c) The doctor and nurse d) The health care team

b) The nurse and the client

Which nursing action reflects evaluation? a) The nurse performs a colostomy irrigation. b) The nurse assesses the client's response to pain medication. c) The nurse identifies that the client has wound drainage. d) The nurse sets an anxiety level of 3 or less with the client.

b) The nurse assesses the client's response to pain medication.

Which of the following nursing actions reflects evaluation? a) The nurse auscultates the client's lungs and abdomen. b) The nurse assesses urine output following administration of a diuretic. c) The nurse identifies that the client does not tolerate activity. d) The nurse sets a tolerable pain rating with the client.

b) The nurse assesses urine output following administration of a diuretic.

A nurse is evaluating nursing care and patient outcomes by using a retrospective evaluation. Which action would the nurse perform in this approach? a) The nurse reviews the patient chart while the patient is being cared for. b) The nurse devises a post-discharge questionnaire to evaluate patient satisfaction. c) The nurse directly observes the nursing care being provided. d) The nurse interviews the patient while he or she is receiving the care.

b) The nurse devises a post-discharge questionnaire to evaluate patient satisfaction.

In which of the following situations would the nurse be most justified in implementing trial-and-error problem solving? a) The nurse is attempting to determine which PRN (as needed) analgesic to offer a patient who is in pain. b) The nurse is attempting to landmark an obese patient's apical pulse. c) The nurse is attempting to determine the range of motion of a patient's hip joint following hip surgery. d) The nurse is attempting to determine whether a post-stroke patient has a swallowing deficit.

b) The nurse is attempting to landmark an obese patient's apical pulse. Trial-and-error problem solving can be dangerous to the patient. Testing range of motion by trial-and-error could result in dislocation; trial-and-error drug administration could result in over- or under-medicating; trial-and-error assessment of a potential swallowing deficit could result in aspiration. Each of these situations warrants more systematic problem solving. Trial-and-error landmarking of an anatomically difficult point, such as the apex of an obese patient's heart, does not pose a threat to the patient and a reasonable amount of "hunting" for the apical pulse may be necessary.

The nurse is conducting a peer review of a nursing colleague. Which action by the nurse is an example of peer review? a) The nurse seeks feedback from the nurse manager regarding job performance for the previous year. b) The nurse preceptor provides feedback to the new graduate nurse after 6 weeks of orientation. c) The new graduate nurse consults the policies and procedures of the institution prior to skill implementation. d) The nurse seeks information from the unlicensed assistive personnel (UAP) regarding the family's response to the nurse's education.

b) The nurse preceptor provides feedback to the new graduate nurse after 6 weeks of orientation.

When teaching an adult client how to control stress through relaxation techniques, the nurse should consider what assumption concerning adult learners? a) As clients, adults are the least likely to resist learning because of preconceived ideas about the teaching-learning process. b) The nurse should be able to draw from the previous experience of the client to emphasize the importance of stress reduction. c) The adult learner is not as concerned with the immediate usefulness of the material being taught as he is with the quality of the material. d) As an adult matures, his self-concept becomes more dependent; therefore, this client must be made aware of the importance of reducing stress.

b) The nurse should be able to draw from the previous experience of the client to emphasize the importance of stress reduction.

Practicing careful hand hygiene and using sterile techniques are ways in which nurses meet which basic human need? a) physiologic b) safety and security c) self-esteem d) love and belonging

b) safety and security By carrying out careful hand hygiene and using sterile technique, nurses provide safety from infection.

The nurse is writing learner outcomes for a patient admitted with heart failure. Which learner outcomes statement is correctly written? a) The patient will understand the relationship between sodium intake and fluid retention. b) The patient will describe symptoms of hypokalemia associated with diuretic use. c) The patient will know how to weigh himself daily at home. d) The patient will comprehend the signs and symptoms of heart failure.

b) The patient will describe symptoms of hypokalemia associated with diuretic use. An appropriately constructed learner outcome statement will include verbs from the cognitive, effective, or psychomotor domains. The verb "describe" is a cognitive domain verb and allows the nurse to measure a patient's understanding of information. Verbs such as "understand," "know how to," and comprehend" are not found in these domains, and measurability is difficult to determine.

The nurse determines that an appropriate nursing diagnosis for a postoperative patient is Risk for Infection. Which of the following is an appropriate long-term goal consistent with this identified nursing diagnosis? a) The patient will remain free of infection as evidenced by no purulent drainage from the surgical incision site. b) The patient will remain free of infection after discharge. c) The patient will remain free of infection as evidenced by a normal white blood cell count while hospitalized. d) The patient will remain free of infection as evidenced by a normal body temperature.

b) The patient will remain free of infection after discharge. A long-term goal focuses on discharge, whereas short-term goals are focused on the hospitalization period. An appropriate long-term goal is that the patient will remain free of infection after discharge.

Which of the following statements is true regarding Friedman's theory of family-centered nursing care? a) Illness of one family member strengthens the roles of the sick member in the family structure. b) The role of the family is essential in every level of nursing care. c) The family is composed of independent members who live and function individually. d) The focus on health should be directed at improving the health of the sickest member of the family.

b) The role of the family is essential in every level of nursing care: Friedman and associates identified the importance of family-centered nursing care, based on four rationales. First, the family is composed of interdependent members who affect one another. If some form of illness occurs in one member, all other members become part of the illness. Second, a strong relationship exists between the family and the health status of its members; therefore, the role of the family is essential in every level of nursing care. The third rationale is that the level of health of the family and, in turn, each member can be significantly improved through health-promotion activities. Finally, illness of one family member may suggest the possibility of the same problem in other members; through assessment and intervention, the nurse can assist in improving the health status of all members.

A dialysis nurse is educating a client on caring for the dialysis access that was inserted into the client's right arm. The nurse assesses the client's fears and concerns related to dialysis, the dialysis access, and care of the access. This information is taught over several sessions during the course of the client's hospitalization. Which phase of the working relationship is best described in this scenario? a) The orientation phase b) The working phase c) The termination phase d) The introduction phase

b) The working phase

In order to provide effective nursing care, the nurse should engage in what type of communication with the client and significant others? a) Purposive communication b) Therapeutic communication c) Intrapersonal communication d) Metacommunication

b) Therapeutic communication

What is the purpose of the ANAs Scope and Standards of Practice? a) To describe the ethical responsibility of nurses. b) To define the activities that are special and unique to nursing. c) To establish nursing as an independent and free-standing profession. d) To regulate the practice of nursing.

b) To define the activities that are special and unique to nursing. The ANAs Scope and Standards of Practice define the activities of nurses that are specific and unique to nursing.

The health care team has convened to discuss the care of an end-of-life client who is not able to achieve an acceptable level of comfort. The physician asks for the nurse's perspective of the situation. Which standard for establishing and sustaining healthy work environments does this action represent? a) Appropriate staffing b) True collaboration c) Effective decision making d) Skilled communication

b) True collaboration True collaboration involves skilled communication, mutual respect, shared responsibility, and decision making among nurses, and between nurses and other health team members. Skilled communication requires health team members to communicate in a non-intimidating manner with colleagues, allowing all voices to be heard regarding a matter. Effective decision making ensures nurses are valued and active partners in making policy, directing and evaluating clinical care, and leading organizational operations.

Which of the nurse's following actions is most likely to promote a patient's compliance with her antiretroviral drug regimen for the treatment of human immunodeficiency virus (HIV)? a) Refer to the characteristics of the HIV virus when explaining how antiretrovirals treat HIV. b) Use interactive and learner-appropriate teaching techniques. c) Ensure that teaching materials are evidence-based and explicitly referenced. d) Emphasize the consequences of noncompliance at various points in patient teaching.

b) Use interactive and learner-appropriate teaching techniques. Noncompliance has serious consequences for the health of patients. It can often be prevented by ensuring that patient teaching is interactive and appropriate to the needs and development of each individual learner. Patient education should indeed be evidence-based, but this does not necessarily enhance compliance. Overemphasizing the negative and grounding the education in virology or other forms of science will also not guarantee compliance.

A nurse is documenting care in a source-oriented record. What action by the nurse is most appropriate? a) Place the narrative note chronologically after the respiratory therapist's note. b) Write a narrative note in the designated nursing section. c) Review the laboratory results under the physician section. d) Use a critical pathway to document the physical assessment.

b) Write a narrative note in the designated nursing section. Source oriented records have separate sections for each disciple to document their own information. Therefore, the nurse would not document in the respiratory section or find the lab results under the physician section. Critical pathways are not used to document physical assessments.

Nurse H. graduated several years ago from a 2-year nursing program that he took at a community college near his home city. Recently, Nurse H. has considered moving from providing direct patient care into an administrative role, but he recognizes the need to further his education in order to be considered for such a position. Nurse H. most likely possesses which of the following nursing qualifications? a) baccalaureate b) associate degree c) diploma d) graduate degree

b) associate degree An associate degree in nursing is typically a 2-year program that is offered at a community college or junior college. Such as degree allows a nurse to plan and provide care, but further education is often necessary for administrative positions. A diploma is often provided under the auspices of a hospital, while both baccalaureate and graduate degrees involve a minimum of 4 years of education and qualify a nurse to act in a supervisory role.

A patient with a lump in her breast calls her primary care physician to schedule an appointment for an evaluation. Based upon the patient's actions, what stage of illness is the patient demonstrating? a) experiencing the symptoms b) assuming the sick role c) achieving recovery and rehabilitation d) assuming the dependent role

b) assuming the sick role Seeking out a healthcare provider for diagnosis and treatment is an example of assuming the sick role. Experiencing the symptoms occurs when a person recognizes one or more symptoms. Assuming the dependent role is characterized by the patient's decision to accept the diagnosis and follow the prescribed treatment plan. Achieving recovery and rehabilitation is characterized by the patient giving up the dependent role and resuming normal activities and responsibilities.

A nurse's neighbor tells her, "I have a high temperature, feel awful, and I am not going to work." What stage of illness behavior is the neighbor exhibiting? a) experiencing symptoms b) assuming the sick role c) assuming a dependent role d) achieving recovery and rehabilitation

b) assuming the sick role When people assume the sick role, they define themselves as ill, seek validation of this experience from others, and give up normal activities.

The nurse is aware that development of nursing diagnoses are: a) collaborative in nature and dependent on the medical diagnosis. b) both within the nursing scope of practice and are client focused. c) based on assessment data and the primary care provider's input. d) dictated by the medical diagnoses and change day by day.

b) both within the nursing scope of practice and are client focused.

The nursing student uses evidence-based practice findings in the development of a care plan. This is an example of which type of nursing skill? a) technical skill b) cognitive skill c) ethical/legal skill d) interpersonal skill

b) cognitive skill The student is demonstrating the use of cognitive skills, which is characterized by identifying scientific rationales for the patient's plan of care, selecting nursing interventions that are most likely to yield the desired outcomes, and using critical thinking to solve problems. Technical skills focus on manipulating equipment skillfully to produce a desired outcome. Interpersonal skills are used to establish and maintain a caring relationship. Ethically and legally skilled nurses conduct themselves in a manner consistent with their personal moral code and professional role responsibilities.

The nurse has entered the room of a newly admitted patient who immediately states that she is feeling short of breath. After identifying this complaint as the patient's problem, what steps should the nurse follow in the process of scientific problem solving? a) make a plan for action b) collect assessment data c) perform hypothesis testing d) evaluate e) formulate a hypothesis f) interpret results

b) collect assessment data e) formulate a hypothesis a) make a plan for action c) perform hypothesis testing f) interpret results d) evaluate Scientific problem solving is a systematic, seven-step, problem-solving process that involves (1) problem identification, (2) data collection, (3) hypothesis formulation, (4) plan of action, (5) hypothesis testing, (6) interpretation of results, and (7) evaluation, resulting in conclusion or revision of the study.

The nurse recognizes that discharge planning begins upon admission and the initial step in discharge planning is: a) providing home healthcare referrals b) collecting and organizing data about the patient c) establishing goals with the patient d) teaching the patient self-care activities that are to be conducted in the home setting

b) collecting and organizing data about the patient The initial step in discharge planning is collecting and organizing data about the patient, as this provides information on the patient's healthcare needs. Establishing goals, patient teaching, and providing home healthcare referrals are steps that will follow the collection and organization of data.

Trends in managed care have resulted in an increased focus on the care of patients by nurses in which of the following settings? a) critical care units b) community-based settings c) long-term care settings d) hospital settings

b) community-based settings Healthcare is increasingly provided in community-based settings such as clinics, outpatient settings, and homes. The impetus for this change has largely been the implementation of a system of managed care to control and monitor healthcare services to minimize costs.

A nurse is developing a presentation for a local community group about the knowledgeable healthcare consumer. Which technology would the nurse include as having the greatest effect on this area? a) television b) computers c) cell phones d) global positioning system (GPS) devices

b) computers Consumers are increasingly using their home computers to access healthcare information through the internet.

One method of developing a theory is by first examining a general idea and then considering specific actions or ideas. What is this method called? a) inductive reasoning b) deductive reasoning c) conceptual modeling d) concept development

b) deductive reasoning Moving from a general idea to specific ideas is deductive reasoning.

Janie wants to call an ethics consult to clarify treatment goals for a patient no longer able to speak for himself. She believes his dying is being prolonged painfully. She is troubled when the patient's doctor tells her that she'll be fired if she raises questions about his care or calls the consult. This is a good example of: a) ethical uncertainty b) ethical distress c) ethical dilemma

b) ethical distress Ethical distress results from knowing the right thing to do but finding it almost impossible to execute because of institutional or other constraints (in this case, fear of losing her job). Ethical uncertainty (a) results from feeling troubled by a situation but not knowing if it is an ethical problem. Ethical dilemmas occur when the principles of bioethics justify two or more conflicting courses of action (c).

A nurse has been working with a patient whose vision has been deteriorating as a result of glaucoma. The patient is reluctant to participate in education surrounding adaptations and home modifications that could aid in her adjustment to sensory loss, stating, "What's the point of even fighting the inevitable?" The nurse would recognize the potential for which of the following nursing diagnoses? a) disturbed body image b) hopelessness c) situational low self-esteem d) fear

b) hopelessness The patient's statements of hopelessness and fatalism demonstrate that counseling is likely appropriate. Fear, low self-esteem, and disturbed body image are all possibilities for this patient, but her statement most clearly suggests that she feels hopeless.

Nurse researchers have predicted that a newly created mentorship program will result in decreased absenteeism, increased retention, and decreased attrition among a hospital's nursing staff. Which of the following does the predicted relationship represent? a) methodology b) hypothesis c) abstract d) dependent variable

b) hypothesis A hypothesis is an expected statement of the relationship between variables in a study. In this study, the dependent variables are absenteeism, retention and attrition while the independent variable is the mentorship program. The methodology of a study is the logistical framework that guides the planning and execution of the study and an abstract is a summary of a research study published in a journal.

A nurse is providing instruction to the patient regarding the procedure to change his colostomy bag. During the teaching session, he asks, "What type of foods should I avoid to prevent gas?" The question the patient has asked allows for which of the following? a) a closed-ended answer b) information clarification c) the nurse to give advice d) a yes or no answer

b) information clarification The patient's question allows the nurse to clarify information that is new to the patient or that requires further explanation.

The nursing process ensures that nurses are patient centered rather than task centered. Rather than simply approaching a patient to take vital signs, the nurse thinks "How is Mrs. Barclay today? Are our nursing actions helping her to achieve her goals? How can we better help her?" This demonstrates which characteristic of the nursing process? a) systematic b) interpersonal c) dynamic d) universally applicable in nursing situations

b) interpersonal Each of the other options are characteristics of the nursing process, but the conversation and thinking quoted best illustrates the interpersonal dimension of the nursing process.

A 56-year-old male patients is experiencing withdrawal from alcohol and is placing himself at risk for falls by repeatedly attempting to scale his bed rails. Benzodiazepines have failed to alleviate his agitation and the nurse is considering obtaining an order for physical restraints to ensure his safety. The should recognize that this measure may constitute: a) advocacy b) paternalism c) harm d) deception

b) paternalism Paternalism involves the violation of a patient's autonomy in order to maximize good or minimize harm, a situation that requires careful consideration in light of ethical principles. Deception is unlikely to occur and the risk for harm is likely decreased by the use of restraints. Advocacy is the protection and support of another's rights.

It is important for the nurse to distinguish a client's readiness to learn and ability to learn. An example of a client's ability to learn includes: a) emotional health. b) physical condition. c) culture. d) social and economic stability.

b) physical condition. The ability to learn focuses on physical condition, acuity of senses, cognitive abilities, literacy, level of education, and communication skills. The readiness to learn focuses on a client's willingness to engage in the teaching-learning process. Readiness to learn includes emotional readiness and experiential readiness such as social and economic stability, culture, attitudes toward learning, and past experiences with learning.

The nurse has measured from the tip of the patient's nose to his earlobe and then down to the xiphoid process before inserting a nasogastric (NG) tube and attaching it to low suction. Which of the following components of the nursing process has the nurse demonstrated? a) implementing; evaluation b) planning; implementing c) assessing; diagnosing d) diagnosing; implementing

b) planning; implementing Determining the correct length of the NG tube to insert is an example of the planning that is necessary in order to conduct this nursing action. The actual insertion of the NG tube would constitute an implementation.

A school nurse is teaching a class of junior high students about the effects of smoking. This educational program will meet which of the aims of nursing? a) promoting health b) preventing illness c) restoring health d) facilitating coping with disability or death

b) preventing illness Educational programs can reduce the risk of illness by teaching good health habits.

A professional nurse with a commitment to social justice is most apt to: a) provide honest information to patients and the public b) promote universal access to healthcare c) plan care in partnership with patients d) document care accurately and honestly

b) promote universal access to healthcare The American Association of Colleges of Nursing lists promoting universal access to healthcare as an example of social justice. Providing honest information and documenting care accurately and honestly are examples of integrity, and planning care in partnership with patients is an example of autonomy.

After teaching a class about the various healthcare agencies and services, the instructor determines that the teaching was successful when the class identifies which of the following as a government agency? a) alcoholics anonymous b) public health service c) rural health center d) ambulatory care center

b) public health service The Public Health Service is a government agency.

Which of the following Standards of Professional Performance, as defined by the ANA, entails the nurse considering factors related to safety, effectiveness, cost and the impact on practice in the planning and delivery of nursing services? a) practice evaluation b) resource utilization c) leadership d) quality of practice

b) resource utilization Resource utilization entails the nurse considering factors related to safety, effectiveness, cost, and the impact on practice in the planning and delivery of nursing services. Leadership involves the nurse providing leadership in the profession and professional practice setting. Practice evaluation involves the nurse evaluating one's own practice in relation to professional practice standards and guidelines, relevant statutes, and rules and regulations. Quality of practice enhances the quality and effectiveness of nursing practice.

A group of nursing students are working together on a presentation for their clinical instructor. One student in the group participates by arguing and attempting to block each step of the process of creating this presentation. The student's behavior is causing frustration for the group of students and slowing their progress. Which of the following best describes the role this individual student is playing in relationship to the group dynamics? a) task-oriented b) self-serving c) maintenance d) group-building

b) self-serving The student's behavior is best described as self-serving. Self-serving roles advance the needs of individual members at the group's expense. Task-oriented roles focus on the work to be completed. Group-building or maintenance roles focus on the well-being of the people doing the work.

A nurse is at the end of a busy shift on a medical-surgical unit. The nurse enters a room to empty the client's urinary catheter and the client says, "I feel like you ignored me today." In response to the statement, the nurse should: a) inform the client that the unit was very busy that day. b) sit at the bedside and allow the client to explain the statement. c) ignore the statement and empty the urinary catheter. d) smile at the client and apologize.

b) sit at the bedside and allow the client to explain the statement.

A patient has been admitted to the hospital for treatment of pancreatitis secondary to alcoholism. The patient states that he finds it nearly impossible to quit drinking because of the deep entrenchment of alcohol use in his circle of friends and line of work. As well, he claims that he thought limiting himself to beer and foregoing hard alcohol would prevent health problems. This patient is exhibiting health consequences rooted in which of the following human dimensions? a) physical and emotional b) sociocultural and intellectual c) emotional and environmental d) environmental and spiritual

b) sociocultural and intellectual The fact that the patient is situated in a context that normalizes heavy alcohol use is an example of the sociocultural dimension. His ignorance of the health consequences of drinking beer rather than spirits is a component of the intellectual dimension.

Which phrase best describes the science of nursing? a) the skilled application of knowledge b) the knowledge base for care c) hands-on care, such as giving a bath d) respect for each individual patient

b) the knowledge base for care The science of nursing is the knowledge base for care that is provided. In contrast, the skilled application of that knowledge is the art of nursing.

Chronic illness may be characterized by periods of remission. Remission is best defined as: a) the response of a person to a disease. b) the presence of a disease with the absence of symptoms. c) a pathologic changes in the structure of function of the body or mind. d) the reappearance of symptoms of a disease.

b) the presence of a disease with the absence of symptoms. Remission is defined as the presence of a disease, but the person does not experience the symptoms. Exacerbation is the reappearance of symptoms of a disease. Disease is a pathologic change in the structure of function of the body or mind. Illness is the response of a person to a disease.

What is the purpose of establishing a nursing diagnosis? a) to identify medical problems b) to describe a functional health problem c) to collaborate with the physician d) to meet accreditation criteria

b) to describe a functional health problem

A nurse caring for a client at a health care facility has to maintain a medical record for the client. Which of the following is a use of the medical record? a) to transmit health records between insurance companies b) to investigate the quality of care in the agency c) to release the entire health record for research d) to inform family and others concerned about the client's care

b) to investigate the quality of care in the agency Medical records may occasionally be used to investigate the quality of care in the agency.

A nurse is taking care of a 66-year-old man post knee surgery. She is following a clinical pathway that guides the care of this client after this specific procedure. He is 2 days postoperative and the clinical pathway states that the nurse should advance his diet. The nurse enters the client's room to discuss this order and finds him vomiting in his wastebasket. A change in client care that deviates from the clinical pathway is called: a) deviation. b) variance. c) audit. d) never events.

b) variance. A variance occurs when the client does not proceed along a clinical pathway as planned.

A student nurse is learning how to write a nursing diagnosis for a client. Which actions are accurate guidelines when formulating nursing diagnoses? Select all that apply. a) Include the medical diagnosis in the nursing diagnosis. b) Be sure the problem statement indicates what is unhealthy about the client. c) Write the diagnosis in legally advisable terms. d) Make sure the client problem precedes the etiology. e) Make sure defining characteristics follow the etiology. f) Phrase the nursing diagnosis as a client need rather than alteration.

b, c, d, e The etiology is the cause of the client problem; therefore, the nursing diagnosis precedes the etiology. Standardized NANDA-I nursing diagnoses must be used; the nurse cannot develop her own nursing diagnoses. The problem statement, which is the nursing diagnosis, indicates what is wrong with the client. Defining characteristics support the nursing diagnosis and should follow the etiology to show support for the nursing diagnosis. A medical diagnosis is only made by a physician or primary care provider and should not be included in the nursing diagnosis because the nurse cannot prescribe treatment for a medical diagnosis. The nursing diagnosis is the identification of a client problem, not a need

A nurse is planning care for a male adolescent patient who is admitted to the hospital for treatment of a drug overdose. Which nursing actions are related to the outcome identification and planning step of the nursing process? Select all that apply. a) The nurse formulates nursing diagnoses. b) The nurse identifies expected patient outcomes. c) The nurse selects evidence-based nursing interventions. d) The nurse explains the nursing care plan to the patient. e) The nurse assesses the patient's mental status. f) The nurse evaluates the patient's outcome achievement.

b, c, d.

A nursing diagnosis of "Ineffective Coping" has been chosen for a client after receiving a diagnosis of prostate cancer. What assessments would the nurse consider as evidence for this diagnosis? Select all that apply. a) The client asks for information relating to the cancer diagnosis. b) The client states, "I can't handle all of this." c) The client has difficulty concentrating on the details of treatment options. d) The client requests the minister of his church to visit. e) The client reports an inability to get adequate restful sleep.

b, c, e

A nurse is caring for an older adult patient who presents with labored respirations, productive cough, and fever. What would be appropriate nursing diagnoses for this patient? Select all that apply. a) Bronchial pneumonia b) Impaired gas exchange c) Ineffective airway clearance d) Potential complication: sepsis e) Infection related to pneumonia f) Risk for septic shock

b, c, f Nursing diagnoses are actual or potential health problems that can be prevented or resolved by independent nursing interventions, such as impaired gas exchange, ineffective airway clearance, or risk for septic shock. Bronchial pneumonia and infection are medical diagnoses, and "potential complication: sepsis" is a collaborative problem.

A nurse on a busy surgical unit relies on informal planning to provide appropriate nursing responses to patients in a timely manner. What are examples of this type of planning? Select all that apply. a) A nurse sits down with a patient and prioritizes existing diagnoses. b) A nurse assesses a woman for postpartum depression during routine care. c) A nurse plans interventions for a patient who is diagnosed with epilepsy. d) A busy nurse takes time to speak to a patient who received bad news. e) A nurse reassesses a patient whose PRN pain medication is not working. f) A nurse coordinates the home care of a patient being discharged.

b, d, e

A registered nurse is writing a diagnosis for a 28-year-old male patient who is in traction due to multiple fractures from a motor vehicle accident. Which nursing actions are related to this step in the nursing process? Select all that apply. a) The nurse uses the nursing interview to collect patient data. b) The nurse analyzes data collected in the nursing assessment. c) The nurse develops a care plan for the patient. d) The nurse points out the patient's strengths. e) The nurse assesses the patient's mental status. f) The nurse identifies community resources to help his family cope.

b, d, f In the diagnosing step of the nursing process, the nurse interprets and analyzes data gathered from the nursing assessment, identifies patient strengths, and identifies resources the patient can use to resolve problems. The nurse assesses and collects patient data in the assessment step and develops a care plan in the planning step of the nursing process.

A nurse is planning teaching strategies for patients addicted to alcohol, in the affective domain of learning. What are examples of strategies promoting behaviors in this domain? Select all that apply. a) The nurse prepares a lecture on the harmful long-term effects of alcohol on the body. b) The nurse explores the reasons alcoholics drink and promotes other methods of coping with problems. c) The nurse asks patients for a return demonstration for using relaxation exercises to relieve stress. d) The nurse helps patients to reaffirm their feelings of self-worth and relate this to their addiction problem. e) The nurse uses a pamphlet to discuss the tenants of the Alcoholics Anonymous program to patients. f) The nurse reinforces the mental benefits of gaining self-control over an addiction.

b, d, f. Affective learning includes changes in attitudes, values, and feelings (e.g., the patient expresses renewed self-confidence to be able to give up drinking). Cognitive learning involves the storing and recalling of new knowledge in the brain, such as the learning that occurs during a lecture or by using a pamphlet for teaching. Learning a physical skill involving the integration of mental and muscular activity is called psychomotor learning, which may involve a return demonstration of a skill.

A nurse is about to perform pin site care for a patient who has a halo traction device installed. What is the first nursing action that should be taken prior to performing this care? a) Administer pain medication. b) Reassess the patient. c) Prepare the equipment. d) Explain the procedure to the patient

b. Before implementing any nursing action, the nurse should reassess the patient to determine whether the action is still needed.

When helping a patient turn in bed, the nurse notices that his heels are reddened and plans to place him on precautions for skin breakdown. This is an example of what type of planning? a) Initial planning b) Standardized planning c) Ongoing planning d) Discharge planning

c

A client is scheduled for a CABG procedure. What information should the nurse provide to the client? a) "A complete ablation of the biliary growth will decrease liver inflammation." b) "The CABG procedure will help identify nutritional needs." c) "A coronary artery bypass graft will benefit your heart." d) "The CABG procedure will help increase intestinal motility and prevent constipation."

c) "A coronary artery bypass graft will benefit your heart." Coronary artery bypass graft is abbreviated CABG.

A nursing instructor is discussing a nursing student's Facebook post about a very interesting client situation that happened during clinical. The student states, "I didn't violate client privacy because I didn't use the client's name." What response by the nursing instructor is most appropriate? a) "You may continue to post about client you cared for during clinicals, as long as you do not use the client's name." b) "The information being posted on Facebook is inappropriate. Make sure to discuss information about client's privately with friends and family." c) "Any information that can identify a person is considered a breach of client privacy." d) "All aspects of the clinical experience are confidential and should not be discussed."

c) "Any information that can identify a person is considered a breach of client privacy."

During an admission intake assessment, a nurse uses open-ended questions to gather information. An example of an open-ended question is: a) "Can you tell me the medications you take on a daily basis?" b) "Do you have and advanced directive or a living will?" c) "Can you tell me why your physician sent you here to be admitted?" d) "Are you allergic to any medications?"

c) "Can you tell me why your physician sent you here to be admitted?"

A resident who is called to see a patient in the middle of the night is leaving the unit but then remembers that he forgot to write a new order for a pain medication a nurse had requested for another patient. Tired and already being paged to another unit, he verbally tells the nurse the order and asks the nurse to document it on the physician's order sheet. The nurse's best response is: a) "Thank you for taking care of this!" b) Get a second nurse to listen to the order, and after writing the order on the physician order sheet, have both nurses sign it. c) "I am sorry, but verbal orders can only be given in an emergency situation that prevents us from writing them out. I'll bring the chart and we can do this quickly." d) Try calling another resident for the order or wait until the next shift.

c) "I am sorry, but verbal orders can only be given in an emergency situation that prevents us from writing them out. I'll bring the chart and we can do this quickly."

Which of the following is the best professional response to a patient who tells you that she believes that "white nurses are smarter than nurse of color" and then asks if you agree? a) "You are right!" (The patient/customer is always right!) b) "What I think doesn't matter. What's important is whatever you believe." (Value neutrality) c) "I don't believe being smart is related to race or ethnicity." (Commitment to human dignity)

c) "I don't believe being smart is related to race or ethnicity." (Commitment to human dignity) While it is true that value neutrality commits nurses to care for patients whether or not the nurse and patient hold the same value, it is not true that nurses should sacrifice their moral integrity and compromise their beliefs or values to please a patient.

A student nurse asks why completing an acuity report is important. What is the best response by the nurse? a) "It determines if a client needs to be transferred to a different unit." b) "It's the beginning step in determining the plan of care for the client." c) "It helps determine our staffing requirements." d) "It provides the pharmacy with the newest physician orders."

c) "It helps determine our staffing requirements." The use of an acuity report includes determining the number of staff needed to care for the clients on the unit. The acuity report does not determine the plan of care, or if a client needs to be transferred. Pharmacy is not impacted by the acuity report.

A friend tells you not to even think about carrying your own insurance because "you'll be a magnet for attorneys trying to make a buck." When you seek the advice of the American Nurses Association, you are likely to read which of the following reasons for purchasing a personal professional liability insurance policy? (1) Protection of the nurse's best interests (2) Limitations of employer's coverage (3) Care or advice given outside of work (4) Protection of the institution's best interests a) (1) b) (1) and (2) c) (1), (2), (3) d) All of the above

c) (1) Protection of the nurse's best interests (2) Limitations of employer's coverage (3) Care or advice given outside of work The ANA (1990) lists these for purchasing a personal professional liability insurance policy.

A nurse is documenting patient data in the medical record of a patient admitted to the hospital with a diagnosis of appendicitis. The physician has ordered 10 mg morphine IV every 3 to 4 hours. Which examples of documentation of care for this patient follows recommended guidelines? Select all that apply. a) 6/12/15 0945 Morphine 10 mg administered IV. Patient's response to pain appears to be exaggerated. M. Patrick, RN b) 6/12/15 0945 Morphine 10 mg administered IV. Patient seems to be comfortable. M. Patrick, RN c) 6/12/15 0945 30 minutes following administration of morphine 10 mg IV patient reports pain as 2 on a scale of 1 to 10. M. Patrick, RN d) 6/12/15 0945 Patient reports severe pain in right lower quadrant. M. Patrick, RN e) 6/12/15 0945 Morphine IV 10 mg will be administered to patient every 3 to 4 hours. M. Patrick, RN f) 6/12/15 0945 Patient states she does not want pain medication despite return of pain. After discussing situation, patient agrees to medication administration.

c) 6/12/15 0945 30 minutes following administration of morphine 10 mg IV patient reports pain as 2 on a scale of 1 to 10. M. Patrick, RN d) 6/12/15 0945 Patient reports severe pain in right lower quadrant. M. Patrick, RN f) 6/12/15 0945 Patient states she does not want pain medication despite return of pain. After discussing situation, patient agrees to medication administration. The nurse should enter information in a complete, accurate, concise, current, and factual manner and indicate in each entry the date and both the time the entry was written and the time of pertinent observations and interventions. When charting, the nurse should avoid the use of stereotypes or derogatory terms as well as generalizations such as "seems comfortable today." The nurse should never document an intervention before carrying it out.

Which documentation tool will the nurse use to record the client's vital signs every 4 hours? a) Medication record b) Acuity charting forms c) A flow sheet d) 24-hour fluid balance record

c) A flow sheet A flow sheet is a form used to record specific client variables such as pulse, respiratory rate, blood pressure readings, body temperature, weight, fluid intake and output, bowel movements, and other client characteristics.

Which of the following clinical events is addressed by the provisions of the Health Insurance Portability and Accountability Act (HIPAA)? a) A patient with suicidal ideation is admitted for treatment despite being adamantly opposed to the admission. b) A patient develops a stage II pressure ulcer after several consecutive shifts where the hospital was short of safe staffing levels. c) A patient believes that the healthcare team is withholding information about her diagnosis and wishes to read her medical chart. d) A patient develops a nosocomial infection after sharing a room with another patient who had vancomycin-resistant enterococcus (VRE).

c) A patient believes that the healthcare team is withholding information about her diagnosis and wishes to read her medical chart. HIPAA defines a number of rights that patients possess within the domains of privacy and disclosure. Among these is the right that patients have to see and copy their health record. HIPAA does not address issues of possible negligence or unsafe care, such as the consequences of staffing shortages or lapses in infection control procedures. Patients who are deemed a genuine threat to themselves or to others may legally be admitted for care against their will.

When assessing a client's nonverbal communication, the nurse will assess which characteristic as the most expressive part of the body? a) Hand gestures b) Eye contact c) Facial expressions d) Posture

c) Facial expressions

A nurse during orientation notices that the preceptor gives all subcutaneous injections on a 45-degree angle. When the new nurse asks the preceptor the rationale for the practice the preceptors states, "This is how I do it, and this is how you will do it." The new nurse recognizes this behavior to be: a) nurturing. b) passive. c) assertive. d) aggressive.

d) aggressive.

Which of the following patients is the most appropriate candidate for receiving outpatient care? a) A patient with as history of depression who is currently expressing suicidal ideation b) A man who is receiving treatment for sepsis after his blood cultures came back positive c) A patient whose complaints of irregular bowel movements have necessitated a colonoscopy d) A woman who has previously borne two children who is entering the second stage of labor

c) A patient whose complaints of irregular bowel movements have necessitated a colonoscopy Outpatient services are appropriate for patients who are medically stable but who require diagnostic testing, such as a colonoscopy. Patients in active labor and patients who are actively septic of suicidal require close monitoring and frequent interventions that can only be safely provided on an inpatient basis.

A nursing instructor is discussing differences between nurse-client relationships and social relationships with a group of nursing students. Which statement is a characteristic of a person-centered or helping relationship? a) A person-centered or helping relationship is built on the needs of the helping person. b) A person-centered or helping relationship occurs spontaneously with random individuals. c) A person-centered or helping relationship is characterized by an unequal sharing of information. d) The person being helped in the person-centered or helping relationship is accountable for the outcomes of the relationship.

c) A person-centered or helping relationship is characterized by an unequal sharing of information. A helping relationship is characterized by an unequal sharing of information. The client shares information related to personal health problems, and the nurse shares information in terms of a professional role.

A pregnant client asks the nurse for information on breastfeeding her baby. What type of nursing diagnosis would the nurse formulate? a) A risk nursing diagnosis b) An actual nursing diagnosis c) A wellness diagnosis d) A possible nursing diagnosis

c) A wellness diagnosis

In the development and documentation of a nursing diagnosis, the nurse should follow which of the following guidelines? a) The use of NANDA nursing diagnoses is necessary to communicate the purposes of research and patient care. b) Due to a lack of standard terms, the use of a computerized program to develop nursing diagnoses is discouraged in many hospitals and nursing facilities. c) Accepted terms for nursing diagnoses may vary according to a school, employer, or specialty organization. d) Nurses should use only NANDA-accepted terms to state diagnoses and develop a nursing care plan.

c) Accepted terms for nursing diagnoses may vary according to a school, employer, or specialty organization. Current thinking states that accepted terms for nursing diagnoses may vary according to a school, employer, or specialty organization. It is no longer necessary for nurses to use only NANDA-accepted terms to state diagnoses or to communicate the purposes of research and patient care. Standard terms in nursing are being developed and refined to facilitate computerization, research, and communication for patient care. When a computerized program is used, nurses will need to learn the terms the program uses to represent diagnoses, outcomes, and interventions.

A nurse writes the following outcome for a patient who is trying to stop smoking: "The patient appreciates or values a healthy body sufficiently to stop smoking." This is an example of what type of outcome? a) Cognitive b) Psychomotor c) Affective d) Physical changes

c) Affective Affective outcomes pertain to changes in patient values, beliefs, and attitudes.

A nurse is planning care for an adult client with severe hearing impairment and a new diagnosis of cancer. Which nursing action is most appropriate when establishing the plan of care? a) Talk with the client's children to determine needs. b) Consult the oncology nurse specialist. c) Arrange for an interpreter when discussing treatment. d) Use text-telephone device (TTD) for daily communication.

c) Arrange for an interpreter when discussing treatment.

The nurse is caring for a 70-year-old client with a fractured wrist. Which of the following is the best method to determine if the client has retained the information taught? a) Test the client on the health education and information imparted. b) Observe the change in client's behavior for a month. c) Ask the client to recall after approximately 15 minutes. d) Ask the client to administer the doses of drug himself.

c) Ask the client to recall after approximately 15 minutes.

A patient's morning blood work indicates a significant decline in his hemoglobin and hematocrit, prompting the primary care provider to order a blood transfusion. As a component of the informed consent process, the potential risks and benefits have been explained to the patient. How should the patient's comprehension of this information be determined? a) Supplement the spoken teaching with written materials. b) Ask the patient if he understands the risks and benefits of transfusion. c) Ask the patient to explain the risks and benefits in his own words. d) Provide an opportunity for the patient to ask any questions about the procedure.

c) Ask the patient to explain the risks and benefits in his own words. When determining a patient's comprehension of information, it is insufficient to simply ask if he or she understands. Rather, the patient should be asked to paraphrase or repeat the information that was provided. Supplementing teaching with written materials may enhance the patient's comprehension, but this does not determine whether or not the teaching was understood. Encouraging the patient to ask questions is an important aspect of teaching, but the absence of questions should not be construed as comprehension.

The nurse completed the minimum data set for a newly admitted client to a skilled nursing facility. What action by the nurse is most appropriate? a) Repeat the minimum data set in 2 weeks. b) Provide a comprehensive written report to Medicare. c) Assess the triggers from the data. d) Document the findings on an occurrence report.

c) Assess the triggers from the data. Once the minimum data set is complete, it will identify elements or triggers for issues that the resident either have or are at risk for developing.

A nurse is educating a 4-year-old client about cast care following a tibia-fibula fracture. Which action is not developmentally appropriate to include in the nurse's education? a) Ensuring the client's parents are present. b) Giving stickers as a reward for task completion. c) Blocking 30 minutes of time for skill teaching. d) Using dolls to demonstrate psychomotor skills.

c) Blocking 30 minutes of time for skill teaching.

A nurse is justified in independently identifying and documenting which diagnosis related to impaired elimination? a) Ulcerative Colitis b) Small Bowel Obstruction c) Bowel Incontinence d) Irritable Bowel Syndrome

c) Bowel Incontinence

Which of the following activities is the clearest example of the evaluation step in the nursing process? a) Recognizing that the patient's blood pressure of 172/101 is an abnormal finding. b) Giving the patient a PRN (as needed) dose of captopril (an antihypertensive) in light of this blood pressure reading. c) Checking the patient's blood pressure 30 minutes after administering the captopril. d) Taking a patient's blood pressure on both arms at the beginning of a shift.

c) Checking the patient's blood pressure 30 minutes after administering the captopril. Measuring the patient's blood pressure after performing an intervention such as drug administration determines the extent to which the patient has achieved the outcome desired, which in this case is lowered blood pressure. Initially checking the patient's blood pressure is an example of assessment while recognizing it as an anomaly constitutes diagnosis. Administering the drub is a form of implementation.

During an assessment of a newly admitted client the nurse asks the client many questions. The nurse begins the assessment by asking, "How many times have you been hospitalized this year for your back pain?" This is an example of which type of question? a) Reflective question b) Sequencing question c) Closed question d) Open-ended question

c) Closed question

Which learning domain is the focus for instruction when the nurse educates a new mother about the breast and its role in milk production for feeding the newborn? a) Psychomotor b) Behavioral c) Cognitive d) Affective

c) Cognitive

The parents of a blended family have a 6-month-old baby boy who is due for immunizations. The clinic closest to their home has recently closed, and they feel intimidated by the prospect of going to the large, university hospital near their home. Which of the following factors is the primary influence on this aspect of the family's health? a) Economic factors b) Lifestyle influences c) Community healthcare structure d) Family risk factors

c) Community healthcare structure The size, location, and services of healthcare offerings in a geographical area are components of the community healthcare structure and its influence on health. Family functioning, lifestyle and economic considerations are not primary influences on the family's actions.

A client reports not having a bowel movement for 7 days, followed by a day of small, loose stools. How does the nurse define the health problem? a) Bowel incontinence related to depressive state b) Readiness for Enhanced Nutrition related to constipation c) Constipation related to irregular evacuation patterns d) Diarrhea related to client report of small, loose stools

c) Constipation related to irregular evacuation patterns

Which action is appropriate when evaluating a client's responses to a plan of care? a) Reinforce the plan of care when each expected outcome is achieved. b) Terminate the plan if there are difficulties achieving the goals/outcomes. c) Continue the plan of care if more time is needed to achieve the goals/outcomes. d) Terminate the plan of care upon client discharge.

c) Continue the plan of care if more time is needed to achieve the goals/outcomes.

A student nurse is attempting to improve her communication skills. Which therapeutic communication skill is appropriate? a) Avoid the use of periods of silence. b) Use cliches to enhance a client's understanding of information. c) Control the tone of the voice to avoid hidden messages. d) Be precise and inflexible regarding the intent of the conversation.

c) Control the tone of the voice to avoid hidden messages.

In anticipation of discharge, a nurse is teaching the daughter of an elderly patient how to change the dressing on her mother's venous ulcer. Which of the following teaching strategies is most likely to be effective? a) Provide explicit written and verbal instructions and ask the daughter to explain back to the nurse how she would perform the dressing change. b) Explain the procedure clearly and slowly while providing multiple opportunities for the daughter to ask questions. c) Demonstrate and explain the procedure and then have the daughter perform it. d) Use a multimedia strategy that combines animation with narration.

c) Demonstrate and explain the procedure and then have the daughter perform it. All steps of a procedure such as a dressing change should be demonstrated, practiced, and provided in writing. The patient or caregiver should then perform the procedure or treatment in the presence of the nurse to demonstrate his or her understanding and ability to carry out the procedure. This is more likely to facilitate success than providing a passive multimedia resource, explaining, or providing written instructions alone without reciprocal demonstration.

A nursing student is nervous and concerned about the work she is about to do at the clinical facility. To allay anxiety and be successful in her provision of care, it is most important for her to: a) Determine the established goals of the institution b) Be sure her verbal and nonverbal communication is congruent c) Engage in self-talk to plan her day and decrease her fear d) Speak with her fellow colleagues about how they feel

c) Engage in self-talk to plan her day and decrease her fear By engaging in self-talk, or intrapersonal communication, the nursing student can plan her day and enhance her clinical performance to decrease fear and anxiety.

A nurse who provides care on a postsurgical unit is performing discharge teaching as a component of her effort to ensure continuity of care. Which of the following is the primary goal of continuity of care? a) Controlling costs and maximizing patient outcomes after discharge from hospital b) Increasing patients' knowledge base and health maintenance behaviors c) Ensuring a smooth and safe transition between different healthcare settings d) Minimizing nurses' legal liability during patient transitions between healthcare institutions.

c) Ensuring a smooth and safe transition between different healthcare settings Continuity of care exists primarily to ensure smooth, safe transitions so that patients may maximize recovery and health. Nurses build patients' knowledge bases in order to achieve this goal. Minimizing nurse liability and healthcare costs are not key justifications for continuity of care.

The best description of critical thinking indicators (CTIs) is which of the following? a) Evidence-based descriptions of behaviors that demonstrate the knowledge that promotes critical thinking in clinical practice. b) Evidence-based descriptions of behaviors that demonstrate the knowledge and skills that promote critical thinking in clinical practice. c) Evidence-based descriptions of behaviors that demonstrate the knowledge, characteristics, and skills that promote critical thinking in clinical practice. d) Evidence-based descriptions of behaviors that demonstrate the knowledge, characteristics, standards, and skills that promote critical thinking in clinical practice.

c) Evidence-based descriptions of behaviors that demonstrate the knowledge, characteristics, and skills that promote critical thinking in clinical practice.

A client, whose care plan includes a nursing diagnosis of "Risk for infection related to a disruption of skin integrity secondary to abdominal surgery", is displaying redness, edema, and warmth at the surgical site. What would be the nurse's most appropriate revision of the care plan? a) Revise the nursing diagnosis to include prescribed medication for infection. b) Revise the nursing diagnosis to "Infection as evidenced by redness, edema, and warmth at the surgical site." c) Formulate the collaborative problem "PC: Infection related to disrupted skin integrity." d) Formulate the medical diagnosis "Wound infection related to infectious processes."

c) Formulate the collaborative problem "PC: Infection related to disrupted skin integrity."

A nurse takes a patient's pulse, respiratory rate, blood pressure, and body temperature. On which form would the nurse most likely document the results? a) Progress notes b) Medical records c) Graphic sheets d) Flow sheets

c) Graphic sheets The graphic record is a form used to record specific patient variables such as pulse, respiratory rate, blood pressure readings, body temperature, weight, fluid intake and output, bowel movements, and other patient characteristics.

During a nursing staff meeting, the nurses determine that they will make sure all vital signs are reported and charted within 15 minutes following assessment. This is an example of: a) Group Decision Making b) Group Leadership c) Group Identity d) Group Patterns of Interaction

c) Group Identity Ascertaining that the staff completes a task on time and that all members agree the task is important is a characteristic of group identity.

Which of the following is the acute care setting for people who are too ill to care for themselves at home or who are severely injured or who require surgery? a) Primary care centers b) Ambulatory care centers c) Hospitals d) Daycare centers

c) Hospitals Hospitals are the most appropriate care provider for people who are too ill to care for themselves at home, who are severely injured, or who require surgery or complicated treatments or who are having babies.

A female client has recently been diagnosed with early-stage Alzheimer's disease and her husband has asked the nurse to recommend Web sites that may supplement his learning about her diagnosis. How should the nurse respond to the husband's request? a) Encourage the husband to avoid online resources due to the unregulated nature of the Internet. b) Provide the husband with print-based materials that are clearly referenced and reflect his learning style. c) Identify and recommend some credible Web sites appropriate to his learning needs. d) Direct the husband to online databases such as the Cumulative Index to Nursing and Allied Health Literature (CINAHL)

c) Identify and recommend some credible Web sites appropriate to his learning needs.

A nurse is interviewing an older adult client who has experienced a drastic weight loss following a CVA (cerebrovascular accident). The client states, "I have trouble getting groceries since I can no longer drive, so I don't have much food in the house." Based on this evidence, what would be the most appropriate nursing diagnosis? a) Imbalanced Nutrition: Less than Body Requirements related to drastic weight loss b) Imbalanced nutrition: Less than Body Requirements related to decreased appetite c) Imbalanced nutrition: Less than Body Requirements related to difficulty in procuring food d) Imbalanced Nutrition: Less than Body Requirements related to cerebrovascular accident

c) Imbalanced nutrition: Less than Body Requirements related to difficulty in procuring food The client relates the drastic weight loss to the inability to bring food into the house. The client's statement is the most appropriate etiology for the nursing diagnosis.

A 26-year-old member of the armed forces was severely injured while serving overseas and now lives with the effects of a spinal cord injury. The man's wife is now forced to get a job in order to provide care for both him and their young son, since the man was the family's soled wage earner. In addition, the family realizes that they may have to move into the basement of the wife's parents in order to survive financially. The family is at greatest risk of which of the following nursing diagnoses? a) Powerlessness b) Ineffective Coping c) Interrupted Family Process d) Impaired Parenting

c) Interrupted Family Process These events presents the possibility of numerous nursing diagnoses. Paramount among these, however, is the fact that the functioning of the family itself has been fundamentally changed. This presents the risk of Interrupted Family Process.

Which of the following phrases best describes the philosophy of community-based care? a) It considers the healthcare needs of the community as a whole. b) It centers on individuals and families with acute and chronic illness needs. c) It has a population-based focus, with an emphasis on illness prevention. d) It provides direction for the roles of the nurse in the acute care setting.

c) It has a population-based focus, with an emphasis on illness prevention. Making appropriate referrals for patients as they move from acute care to home care is an essential component of discharge planning for continuity of care.

The nurse receives a verbal order from a physician during an emergency situation. What actions should be taken by the nurse? (Select all that apply.) a) Have the physician review and sign the order during the emergency. b) Record the order on the pharmacy discrepancy sheet. c) Mark the date and time of the order. d) Read back the order. e) Include V.O. with the physician name on the order.

c) Mark the date and time of the order. d) Read back the order. e) Include V.O. with the physician name on the order.

Which of the following national nursing organizations serves as a primary source of research data about nursing education and is the professional organization for nurse educators? a) American Association of Colleges of Nursing (AACN) b) Sigma Theta Tau International c) National League for Nursing (NLN) d) American Nurses Association (ANA)

c) National League for Nursing (NLN) The NLN serves as the primary source of research data about nursing education, conducting annual surveys of school and new RNs. The NLN's objective is to foster the development and improvement of nursing services and nursing education, and it serves as the professional nursing organization for nurse educators. The ANA is the professional organization for RNs in the United States, and it establishes standards of practice, encourages research to advance nursing practice, and represents nursing for legislative actions. Sigma Theta Tau International is the honor society for nursing. AACN is the national voice for baccalaureate and higher degree nursing education programs and provides accreditation for collegiate nursing programs.

A middle-aged couple is considering enrolling in a long-term care insurance plan. What will this insurance cover? a) Inpatient hospital services b) Copayments for primary care services c) Nursing home and home care services d) Pharmaceutical costs

c) Nursing home and home care services Long-term care insurance plans cover a variety of services, such as nursing home care and home care, as well as services that help prevent the institutionalization of older, debilitated and chronically ill people. Adult daycare centers and respite care are also services often covered by such a plan. Pharmaceutical costs and inpatient and outpatient services are covered in the elder years by Medicare.

The nursing instructor is discussing communication with a student. The student identifies that a contract is made with the client during which phase of the nurse-client relationship? a) Termination phase b) Intimate phase c) Orientation phase d) Working phase

c) Orientation phase

The nurse is caring for Mr. M., a 48-year-old man with congestive heart failure. The nurse manager informs the nurse that Mr. M. was enrolled in a clinical trial to assess whether a 10-minute walk, 3 times per day, leads to expedited discharge. Which type of evaluation best describes what the researchers are examining? a) Cost-effectiveness evaluation b) Process evaluation c) Outcome evaluation d) Structure evaluation

c) Outcome evaluation

Prior to the first postoperative visit post gastrectomy, the client will have a weight loss of 10 lbs. This is an example of which type of evaluative statement? a) Cognitive b) Psychomotor c) Physical changes d) Affective

c) Physical changes

A nurse suspects that a patient has a self-care deficit, but needs more data to confirm this diagnosis. What nursing diagnosis would the nurse write for this patient? a) Apparent b) Potential c) Possible d) Actual

c) Possible Possible nursing diagnoses are statements describing a suspected problem for which additional data are needed. Additional data are used to confirm or rule out the suspected problem. An actual nursing diagnosis means that there is data to support a patient's actual health care problem. NANDA-I describes five types of nursing diagnoses: actual, risk, possible, wellness, and syndrome.

The nurse is working with Mr. Milner, a 55-year-old man who recently underwent a coronary artery bypass graft (CABG). He is taking furosemide and metoprolol following his procedure. Together the nurse and Mr. Milner are developing a plan for a heart-healthy diet. During their planning, Mr. Milner states that he really thinks that his diet did not contribute to his heart disease. If he continues to take his medications, he should be fine. According to the Transtheoretical Model of Change, what stage of change is Mr. Milner in related to his diet? a) Contemplation b) Preparation c) Precontemplation d) Maintenance

c) Precontemplation The precontemplation stage is the first stage change, in which the client does not indicate that he will make any changes within the next 6 months. Mr. Milner gives no indication that he will be ready to change his diet within the next 6 months.

A nurse is using the SBAR technique for hand-off communication when transferring a client. What are examples of the use of this process? Select all that apply. a) S: The nurse discusses the client's symptoms with the new nurse in charge. b) R: The nurse explains the rules of the new facility to the client. c) R: The nurse gives recommendations for future care to the new nurse in charge. d) S: The nurse handling the transfer describes the client situation to the new nurse. e) B: The nurse gives the background of the client by explaining the client history. f) A: The nurse presents an assessment of the client to the new nurse.

c) R: The nurse gives recommendations for future care to the new nurse in charge. d) S: The nurse handling the transfer describes the client situation to the new nurse. e) B: The nurse gives the background of the client by explaining the client history. f) A: The nurse presents an assessment of the client to the new nurse.

A patient informs the nurse that he is interested in attending a weight loss support group prior to his discharge from treatment for a myocardial infarction. In preparing a care plan for this patient, an appropriate nursing diagnosis would include which of the following? a) Imbalanced Nutrition: Less Than Body Requirements b) Ineffective Denial c) Readiness for Enhanced Therapeutic Regimen Management d) Health-seeking Behaviors

c) Readiness for Enhanced Therapeutic Regimen Management Based on the patient's desire to lose weight and comply with the therapeutic regimen for myocardial infarction, and appropriate nursing diagnosis for this patient is Readiness for Enhanced Therapeutic Regimen Management. Health-seeking Behaviors, Ineffective Denial, and Imbalanced Nutrition: Less Than Body Requirements are not appropriate nursing diagnoses based upon the patient's desire to improve his nutritional intake and weight.

The nurse has instructed the client in self-catheterization, but the client is unable to perform a return demonstration. What is the nurse's most appropriate plan of action? a) Teach the content again utilizing the same method. b) Revise the plan to include the inclusion of a support group. c) Reassess the appropriateness of the method of instruction. d) Report the client's inability to learn to the case manager.

c) Reassess the appropriateness of the method of instruction.

The nurse is providing care for a patient who has a tracheotomy and whose pulse oximeter has recently alarmed, showing the patient's oxygen saturation to be 77%. The nurse has repositioned the patient and applied supplementary oxygen, interventions that have raised the patient's oxygen levels to 80% and somewhat decreased his work of breathing. The patient is not in immediate distress, and his level of consciousness remains high. The nurse should page which of the following practitioners? a) Physical therapist b) Physician c) Respiratory therapist d) Occupational therapist

c) Respiratory therapist A respiratory therapist is an expert in lung function and oxygenation whose expertise is needed in the care of this patient. Because the patient is not experiencing severe distress or respiratory arrest, the nurse is justified in foregoing contact with the physician in the short term. A physical therapist or occupational therapist is not likely able to provide needed interventions at this time.

Which action should the nurse take when client data indicate that the stated goals have not been achieved? a) Implement a standardized plan of care. b) Collect more data for the database. c) Review each preceding step of the nursing process. d) Change the nursing orders.

c) Review each preceding step of the nursing process.

The nursing is caring for a client who requests to see a copy of his or her medical records. What action by the nurse is most appropriate? a) Access the medical record at the bedside and show the client how to navigate the electronic medical record. b) Explain that only a paper copy of the medical record can be viewed by the client. c) Review the hospital's process for allowing clients to view their medical records. d) Discuss how the hospital can be fined for allowing clients to view their medical records.

c) Review the hospital's process for allowing clients to view their medical records. The nurse needs to be aware of the policies regarding clients reviewing medical records.

A patient is experiencing difficulty in adjusting to a new prosthesis despite conscientious patient teaching by numerous members of the healthcare team. How should the team respond to the patient's lack of learning to this point? a) Scale back the scope and detail of patient teaching. b) Explore alternatives to prosthesis. c) Revise the teaching plan that has guided education. d) Refer the patient to outside sources of information.

c) Revise the teaching plan that has guided education. If evaluation of patient teaching indicates that patient learning has not met outcomes, it is appropriate to revise the teaching plan. This does not necessarily entail reducing the detail or referring the patient to outside information sources. Exploring alternatives to prosthesis does not address the patient's learning needs.

The nurse assists a postoperative patient with ambulation. The nurse recognizes that assisting the patient when performing this skill meets which of Maslow's basic human needs? a) Self-actualization. b) Self-esteem. c) Safety and security. d) Love and belonging

c) Safety and Security: Nurses carry out a wide variety of activities to meet patients' physical safety needs, such as moving and ambulating patients. Assisting the patient to ambulate ensures that the patient will not experience a fall.

Which of the following statements is true of the nursing process? a) It is more appropriate in medical-surgical settings than community healthcare. b) It is a valid alternative to using intuition to respond to nursing situations. c) Scientific problem solving can occur within the nursing process. d) Trial-and-error problem solving is incongruent with the nursing process.

c) Scientific problem solving can occur within the nursing process. Problem solving and the nursing process are not competing or mutually exclusive processes. Rather, both scientific problem solving and trial-and-error may take place within the nursing process. One of the strengths of the nursing process is that it is applicable to all nursing contexts.

What type of authority regulates the practice of nursing? a) International standards and codes b) Federal guidelines and regulations c) State nurse practice acts d) Institutional poilicies

c) State nurse practice acts Nurse practice acts are established in each stated to regulate the practice of nursing.

Which of the following guidelines should a nurse use when choosing a position in relation to a patient during a verbal interaction? a) Ask the patient where he or she would like you to position yourself and move accordingly. b) Choose a position that is no closer than 2 feet, but no farther than 4 feet. c) Take note of the patient's cues when choosing a position and act on these cues. d) Assess the patient's culture during the initial meeting or assessment.

c) Take note of the patient's cues when choosing a position and act on these cues. Preferences regarding space and territoriality vary greatly. A useful strategy to foster good communication is to note, and act on, patient cues. While preferences are often culturally rooted, knowing a patient's culture does not provide all the data a nurse needs in order to accommodate variables around positioning. Rigid parameters are likely to simplistic and explicitly asking the patient may make him or her feel uncomfortable.

A nurse is teaching a 50-year-old male patient how to care for his new ostomy appliance. Which teaching aid would be most appropriate to confirm that the patient has learned the information? a) Ask Me 3 b) Newest Vital Sign c) Teach-back tool d) TEACH acronym

c) Teach-back tool The teach-back tool is a method of assessing literacy and confirming that the learner understands health information received from a health professional. The Ask Me 3 is a brief tool intended to promote understanding and improve communication between patients and their providers. The Newest Vital Sign (NVS) is a reliable screening tool to assess low health literacy, developed to improve communications between patients and providers. The TEACH acronym is used to maximize the effectiveness of patient teaching by tuning into the patient, editing patient information, acting on every teaching moment, clarifying often, and honoring the patient as a partner in the process.

Which model is most useful in examining the cause of disease in an individual and is based upon external factors? a) The High-Level Wellness Model b) The Health Belief Model c) The Agent-Host-Environment Model d) The Health-Illness Continuum

c) The Agent-Host-Environment Model The Agent-Host-Environment Model is useful for examining the cause of disease in an individual. The agent, host, and environment interact in ways that create risk factors. The Health-Illness Continuum is a way to measure a person's level of health. The High-Level Wellness Model is characterized by functioning to one's maximum potential while maintaining balance and purposeful direction in the environment. The Health Belief Model is used to describe health behaviors.

A friend asks you about the new Bill of Rights for nurses. What can you tell her that accurately reflects the concerns of the drafters of these rights? a) The Bill of Rights was drafted by nurses who care more about themselves than they do about patients. b) The Bill of Rights was drafted by union nurses who are always looking for a reason to strike. c) The Bill of Rights was drafted to empower nurses and to improve conditions in the workplace.

c) The Bill of Rights was drafted to empower nurses and to improve conditions in the workplace. The American Nurses Association's Bill of Rights for Registered Nurses was a result of advocacy on behalf of nurses, to aid in improving workplaces and to ensure that nurses would have what they needed to provide safe, quality patient care. It is not true that these nurses cared more for themselves than patients (a) nor that these were nurses looking for reasons to strike (b).

Which organization audits charts regularly? a) Sigma Theta Tau International b) American Nurses Association c) The Joint Commission d) National League for Nursing

c) The Joint Commission

Which of the following aspects of nursing would be most likely defined by legislation at a state level? a) The process that nurses must follow when handling and administering medications. b) The criteria that patients must meet in order to qualify for Medicare or Medicaid. c) The differences in the scope of practice between registered nurses (RNs) and licensed practical nurses (LPNs). d) The criteria that a nurse must consider when delegating tasks to unlicensed care providers.

c) The differences in the scope of practice between registered nurses (RNs) and licensed practical nurses (LPNs). The scope of practice defines the parameters within which nurses provide care and is established by state legislation, most commonly in the form of a Nurse Practice Act. The criteria and due process for delegation in the clinical setting is addressed by a stated board of nursing. Qualification criteria for programs such as Medicare and Medicaid are established by federal legislation while the process for safe and appropriate medication administration is defined and monitored by a stated board of nursing.

Which of the following is the most important indicator of quality nursing care? a) The nurse follows the policies and procedures of the institution. b) The nurse is organized and efficient in client care. c) The nurse considers the individual needs of clients. d) The nurse takes measures to ensure accurate medication administration.

c) The nurse considers the individual needs of clients.

A nurse evaluates clients prior to discharge from a hospital setting. Which action is the most important act of evaluation performed by the nurse? a) The nurse evaluates the types of health care services available to the client. b) The nurse evaluates the competence of nurse practitioners. c) The nurse evaluates the client's goal/outcome achievement. d) The nurse evaluates the plan of care.

c) The nurse evaluates the client's goal/outcome achievement.

The nurse is caring for a client who has been diagnosed with a sexually transmitted infection (STI). The nurse plans to address the nursing diagnosis of Risk Prone Behavior. What assumption has the nurse made? a) The nurse has assumed that the client does not understand the complications of sexually transmitted infections. b) The nurse has assumed that having a sexually transmitted infection means the client is unaware of the risks of unprotected sex. c) The nurse has assumed that having a sexually transmitted infection means the client is sexually promiscuous. d) The nurse has assumed that the client needs education to decrease the likelihood of repeated infection.

c) The nurse has assumed that having a sexually transmitted infection means the client is sexually promiscuous. Risk Prone Behavior identifies habits of the client that are dangerous. Being sexually promiscuous would be a dangerous behavior. Risk prone behavior does not mean that the client is not knowledgeable or needs further instructions about complications.

The nurse is finding it difficult to plan and implement care for a client and decides to have a nursing care conference. What action would the nurse take to facilitate this process? a) The nurse sends or directs someone to take action in a specific nursing care problem. b) The nurse, along with other nurses, visits clients with similar problems individually at each client's bedside in order to plan nursing care. c) The nurse meets with nurses or other health care professionals to discuss some aspect of client care. d) The nurse consults with someone in order to exchange ideas or seek information, advice, or instructions.

c) The nurse meets with nurses or other health care professionals to discuss some aspect of client care. A nursing care conference is a meeting of nurses to discuss some aspect of a client's care.

During a home health care visit, the nurse identifies a nursing diagnosis of Caregiver Role Strain for a parent who is caring for a child dependent on a ventilator. What subjective assessment data would support the nurse's diagnosis? a) The parent states, "A member of my church gives me a break twice a week." b) The parent states, "I make sure that I get regular exercise." c) The parent states, "I cannot allow anyone else to help because they won't do it right." d) The parent states, "I attend support group meetings when I am able to go."

c) The parent states, "I cannot allow anyone else to help because they won't do it right."

Family assessment of a father, mother and four children has suggested the presence of several risk factors. Which of the following aspects of the family's structure and function would be considered a psychosocial risk factor? a) The family's electricity has been cut off at various times due to nonpayment. b) The mother has a history of heavy alcohol use. c) The parents have a tumultuous relationship with frequent separations in the past. d) The family lives in a small apartment in a poor neighborhood with high crime rates.

c) The parents have a tumultuous relationship with frequent separations in the past. Conflict is an example of a psychosocial risk factor. Chemical dependency is considered a lifestyle risk factor while a lack of adequate housing is an environmental risk factor.

Which of the following best describes acute illness? a) The leading health problem in the world. b) A medical condition that is life threatening and requires surgery. c) The rapid onset of symptoms lasting a relatively short time. d) A condition that causes a permanent change and a long period of care.

c) The rapid onset of symptoms lasting a relatively short time. Acute illness is generally described as a rapid onset of symptoms lasting a relatively short time. Acute illnesses are not always life threatening and may not even require medical care or surgery. A chronic illness causes a permanent change and requires a long period of care or support. Chronic illnesses are the leading health problem in the world.

The Patient Care Partnership form, given to patients on admission to the hospital, includes several specific rights during hospitalization. Which of the following is included in this form? a) The right to wear one's own bed clothing. b) The right to use a personal cell phone. c) The right to have help when leaving the hospital. d) The right to take medications brought from home.

c) The right to have help when leaving the hospital. One item on the Patient Care Partnership form is the right to have help when leaving the hospital.

Of the following statements, which one is true of self-actualization? a) Humans are born with fully developed self-actualization. b) Self-actualization needs are met by having confidence and independence. c) The self-actualization process continues throughout life. d) Loneliness and isolation occur when self-actualization needs are unmet.

c) The self-actualization process continues throughout life. Self-actualization, or reaching one's full potential, is a process that continues through life.

The nurse caring for a morbidly obese client formulates the possible nursing diagnosis, "Imbalanced Nutrition: More than Body Requirements related to excessive food intake as evidenced by morbid obesity." In order to assure the accuracy of the diagnosis, which further step must the nurse take? a) Interview the client to asses the client's motivation to lose weight. b) Determine what weight loss programs the client has utilized in the past. c) Validate with the client that excessive food intake is the cause of the client's obesity. d) Research the client's medical history to determine the client's usual weight.

c) Validate with the client that excessive food intake is the cause of the client's obesity.

A nurse has chosen to characterize a new initiative as "wellness promotion" rather than "health promotion." Which of the following statements best describes the difference between the concept of wellness and the concept of health? a) Wellness is determined by age-related expectations, whereas health is an achievable state at any point in the lifespan. b) Wellness is dependent on the resolution of acute and chronic illnesses, whereas health can exist at any stage or condition. c) Wellness is an active state, whereas health is a more passive state dependent on the absence of disease. d) Health is a state that can be promoted and protected by nursing practice, whereas wellness is solely dependent on the patient.

c) Wellness is an active state, whereas health is a more passive state dependent on the absence of disease. Good health is a passive state wherein the person is not ill. Wellness is a more active state, regardless of one's level of health. Wellness is not contingent on the resolution of disease or illness and it supersedes age. Both health and wellness can be influenced by nursing practice.

A client is admitted to the health center with chronic diarrhea. When should the nurse begin imparting health teaching about the benefits of proper diet to the client so that the risk of diarrhea is minimized? a) When providing treatment b) When discharging the client c) When admitting the client

c) When admitting the client

Nurse advocates often are conflicted about respecting a patient's right to be self-determining, while at the same time wanting to do everything in their power to promote the patient's best interests. Which is the best general guideline for situations like these? a) Patient rules! "It's my life!" b) Nurse rules! "It may be your life, but in this instance you don't know enough to make the right choice!" c) When in conflict, weigh the benefits and risks of following each option and then choose wisely.

c) When in conflict, weigh the benefits and risks of following each option and then choose wisely. Neither respecting and supporting patient preferences (a) nor ignoring patient preferences to achieve a medical benefit (b) routinely trump all other considerations. When a nurse cannot do both simultaneously, she must carefully weigh the benefits and risks of each option and the choose wisely.

Care provided to a client following surgery and until discharge represents which phase of the nurse-client relationship? a) Orientation phase b) Termination phase c) Working phase d) Evaluation phase

c) Working phase

What historic event in the 20th century led to a increased emphasis on nursing and broadened the role of nurses? a) religious reform b) Crimean War c) World War II d) Vietnam War

c) World War II During World War II, large numbers of women worked outside the home. There was an increased emphasis on education and a knowledge explosion in medicine and technology, broadening the roles of nurses.

What information provides the nurse with accuracy when developing a nursing diagnosis? a) specific nursing interventions b) a set of lab values c) a set of clinical cues d) abnormal diagnostic tests

c) a set of clinical cues Each piece of client information is considered a clinical cue; a set of clinical cues forms a cluster that is present if the diagnosis is accurate.

Establishing the criteria for the education and licensure of nurses is a component of: a) evidence-based practice. b) the US Department of Health and Human Services' Healthy People 2010 document. c) a state's nurse practice act. d) the ANA Standards of Practice.

c) a state's nurse practice act. While nurse practice acts vary from state to state, they typically specify the criteria for the education and licensure of nurses. The ANA Standards of Practice and Healthy People 2010 do not address such legal and licensure issues. The philosophy of evidence-based practice does not encompass specific criteria for education and licensing nurses.

Which process evaluates and recognizes educational programs as having met certain standards? a) licensure b) credentialing c) accreditation d) certification

c) accreditation Accreditation is the process by which an educational program is evaluated and recognized as having met certain standards. Credentialing refers to ways in which professional competence is ensured and maintained. Licensure is the process by which a state determines that a candidate meets certain minimum requirements to practice in the profession and grants the license to do so. Certification is the process by which a person who has met certain criteria established by a nongovernmental association is granted recognition in a specified practice area.

A patient admitted to the telemetry floor informs the nurse that he has an advance directive and does not wish to be resuscitated if his heart stops beating. The patient takes a copy of the advance directive from his wallet and hands it to the nurse. The nurse documents on the electronic chart that the patient has an advance directive, makes a copy of the advance directive, and immediately informs the patient's physician. In this situation, what is the nursing role demonstrated by the nurse? a) researcher b) leader c) advocate d) counselor

c) advocate The nursing role that focuses on advocacy involves the protection of human or legal rights and the securing of care for all patients based on the belief that patients have the right to make informed decisions about their own health and lives. In this situation, the nurse recognizes the patient's right to make end-of-life decisions and advocates for the patient by documenting that the patient has an advance directive and also by communicating this information to the patient's physician.

A nurse researcher is studying the effects of exercise and sleep on blood pressure. The researcher identifies blood pressure as which type of variable? a) exploratory b) correlational c) dependent d) independent

c) dependent The dependent variable is the variable being studied and is determined by manipulating conditions (the independent variables).

A nurse has completed 4 hours of his 8-hour shift on a medical-surgical unit when he receives a phone call from the nursing supervisor. The nursing supervisor informs the nurse that he needs to give a report to the other two nurses on the medical-surgical unit and immediately report to the telemetry unit to assist with staff needs on that unit. The nurse informs the supervisor that he has been busy with his patient assignment and feels this will overwhelm the nurses on the medical-surgical unit. The supervisor informs the nurse that the need is greater on the telemetry unit. This is an example of which type of ethical problem? a) advocacy in a market-driven environment b) deception c) allocation of scarce nursing resources d) conflicts concerning new technology

c) allocation of scarce nursing resources This is a clear example of ethical issues surrounding the allocation of scarce nursing resources and involves distributing nursing care.

A nurse researcher has proposed a study that will examine the effects of a preoperative teaching module on patients' levels of anxiety. In order to carry out the study, the researcher needs to identify a test that can reliably quantify each participant's anxiety. Which of the following tools will accomplish this task? a) a test b) a framework c) an instrument d) a scale

c) an instrument An instrument is a research tool that can be used to collect, record, and quantify data. Typically, instruments are surveys, tests, or questionnaires that have been shown to be reliable and valid in capturing the phenomenon in question.

When preparing a health promotion program for patients in an adult daycare center, what is the first step the nurse must take? a) develop learning outcomes b) develop a teaching plan c) assess the patients' learning needs and learning readiness d) diagnose the patients' learning needs

c) assess the patients' learning needs and learning readiness The first step of the teaching-learning process is to assess the patient's learning needs and learning readiness.

A registered nurse enters a patient's room and observes the nursing assistant pushing a patient down on the bed. The patient starts crying and informs the nursing assistant that he needs to go to the bathroom. The nursing assistant holds the patient down and tells him he was just in the bathroom. The nurse observing this incident is aware that the nursing assistant's action is an example of which of the following? a) fraud b) defamation of character c) battery d) assault

c) battery The nursing assistant is engaging in battery, which is an assault that is carried out and includes willful, angry, and violent or negligent touching of another person's body or clothes or anything attached to or held by that other person. Assault is a threat or an attempt to make bodily contact with another person without that person's consent. Fraud is willful and purposeful misrepresentation that could cause, or has caused, loss or harm to a person or property. Defamation of character is an intentional tort in which one party makes derogatory remarks about another that diminishes the other party's reputation.

John and Mary, each parents of one child, are both divorced. When they marry, the family structure that is formed will be described as which of the following? a) Nuclear family b) Extended family c) Blended family d) Cohabiting family

c) blended family A blended family is formed when parents bring unrelated children from previous relationships together to form a new family.

Having recently completed a specialty nursing program in neonatal care, a nurse is now preparing to leave her current position on a medical unit and begin providing care in the hospital's neonatal intensive care unit (NICU). The nurse has completed which of the following processes of credentialing? a) accreditation b) licensure c) certification d) validation

c) certification The process of certification involves the attainment and validation of specialized nursing knowledge and skills. Certification is often necessary in order to ensure that the nursing care that is provided in specialized and high-acuity settings is safe and appropriate. Accreditation is the process by which an educational program, rather than an individual nurse, identifies that standards are being met. The process of licensure involves the determination that a nurse meets minimum requirements to practice, but not necessarily the specialized knowledge that is necessary for some care settings.Validation is not a specific aspect of the process of credentialing.

An oncology nurse with 15 years of experience, certification in the area of oncology nursing, and a master's degree is considered to be an expert in her area of practice and works on an oncology unit in a large teaching hospital. Based upon this description, which of the following career roles best describes this nurse's role based on her qualifications and experience? a) nurse entrepreneur b) nurse educator c) clinical nurse specialist d) nurse practitioner

c) clinical nurse specialist A clinical nurse specialist is a nurse with an advanced degree, education, or experience who is considered to be an expert in a specialized area of nursing. The clinical nurse specialist carries out direct patient care; consultation; teaching of patients, families, and staff; and research. A nurse practitioner has an advanced degree and works in a variety of settings to deliver primary care. A nurse educator usually has an advanced degree and teaches in the educational or clinical setting. A nurse entrepreneur may manage a clinic or health-related business.

The nursing diagnosis taxonomy provides nursing with: a) discharge planning. b) evaluative care. c) common language. d) legal information.

c) common language. Professions require a sound scientific base; the nursing process is nursing's scientific base. To achieve this scientific foundation, nursing requires a taxonomy, or classification system, to provide a structure for nursing practice.

An obstetrical nurse wishes to identify whether patients' perceptions of a high level of support from their partner is associated with a decreased length of the second stage of labor. Which type of quantitative research is most appropriate for this research question? a) descriptive research b) experimental research c) correlational research d) quasi-experimental research

c) correlational research Correlational quantitative research is used to examine relationships between two or more variables. In this case, the variables are perceptions of partner support and length of stage two labor. There is no manipulation of the variables as there would be in an experimental or quasi-experimental study and the focus on the relationship between the two variables goes beyond simple description of events.

When conducting research, information is collected. This would be identified as which of the following? a) subject b) analysis c) data d) abstract

c) data Data are observable and verifiable information collected to describe, explain, or predict events.

A hospice nurse is meeting with the parents of a terminally ill child. The nurse listens to the concerns and fears that the parents are verbalizing as they prepare to allow the child to die peacefully at home. What Critical Thinking Indicator characterizes the behavior of the hospice nurse? a) flexible b) self-disciplined c) empathetic d) creative

c) empathetic The hospice nurse is demonstrating empathetic behavior. Empathy is described as listening well and showing the ability to imagine other's feelings and difficulties. Being flexible is defined as changing approaches as needed to get the best results. Self-disciplined requires the nurse to stay on task and manage time to focus on priorities. Creative behaviors offer alternative solutions and approaches. While the nurse will need to demonstrate flexibility, creativity, and self-discipline to assist the parents in preparing for the child's death, the nurse is demonstrating empathetic behavior by listening to the parents.

The nursing student is nervous and concerned about the work she is about to do at the clinical facility. To ally anxiety and be successful in her provision of care, it is important for her to: a) determine the established goals of the institution b) be sure her verbal and nonverbal communication is congruent c) engage in self-talk to plan her day and decrease her fear d) speak with her fellow colleagues about how they feel

c) engage in self-talk to plan her day and decrease her fear By engaging in self-talk, or intrapersonal communication, the nursing student can plan her day and enhance her clinical performance to decrease fear and anxiety.

Risk factors for illness are divided into six categories. Working with carcinogenic chemicals is an example of which type of risk factor for illness? a) health habits risk factor b) physiologic risk factor c) environmental risk factor d) lifestyle risk factor

c) environmental risk factor Working and living environments may contribute to disease. Working with cancer-causing chemicals is an example of an environmental risk factor for illness.

A nurse working on a critical care unit was informed by a patient with multiple sclerosis that she did not wish to be resuscitated in the event of cardiac arrest. The patient is no longer able to express her wishes, and the family has informed the physician that they want the patient to be resuscitated. Aware of the patient's wishes, the nurse is involved in a situation that may experience: a) confidentiality b) deception c) ethical distress d) paternalism

c) ethical distress The nurse is involved in a situation that involves ethical distress. Ethical distress occurs when the nurse knows the right thing to do but either personal or institutional factors make it difficult to follow the correct course of action. Paternalism is acting for patients without their consent to secure good or prevent harm. Deceptions and confidentiality can result in ethical problems for nurses when there is a conflict between the patient's and nurses' values and interests. In this scenario, the nurse is aware of the patient's wishes but the conflict lies with the family and thus the nurse will experience ethical distress.

When assessing a patient's nonverbal communication, the nurse will assess which of the following as the most expressive part of the body? a) posture b) eye contact c) facial expression d) hand gestures

c) facial expressions The face is the most expressive part of the body. Eye contact, the lack of eye contact, posture, gesture, and silence are other methods of nonverbal communication.

A 61-year-old man is distraught because he has just learned that his most recent computed tomography (CT) scan shows that his colon cancer has metastasized to his lungs. Which of the following nursing aims should the nurse prioritize in the immediate care of this patient? a) preventing illness b) promoting health c) facilitating coping d) restoring health

c) facilitating coping This patient's care in the coming weeks or months will likely encompass all of the four foundational roles of the nurse. However, because the patient has just recently received bad news and is emotionally distraught, helping the patient cope is an appropriate priority in his immediate care.

A nurse is asked to teach a group of young adults a healthy lifestyle. One young man says, "I stopped eating fast foods to lose weight." What model of health or illness explains this behavior? a) health-illness continuum b) agent-host-environment model c) health promotion model d) health belief model

c) health promotion model The health promotion model illustrates how health related behaviors and beliefs promote health.

A staff development nurse is asking a group of new staff nurses to read and be prepared to discuss a qualitative study that focuses on nursing events of the past in an attempt to increase understanding of the nursing profession today. What method of qualitative research is used in this article? a) grounded theory b) ethnography c) historical d) phenomenology

c) historical This article uses historical methodology, which examines events of the past to increase understanding of the nursing profession today. Phenomenology is used to describe experiences as they are lived by the subjects being studied. Grounded theory is the discovery of how people describe their own reality and how their beliefs are related to their actions in a social scene. Ethnography is used to examine issues of a culture that are of interest to nursing.

Which type of healthcare facility employs the largest percentage of RNs? a) Ambulatory care b) Long-term care c) Hospitals d) Physician offices

c) hospitals Despite a downward trend, the largest percent of RNs are employed by hospitals.

Based on the Health Insurance Portability and Accountability Act (HIPAA), a patient admitted to a healthcare facility must be provided with a written explanation of: a) the names and addresses of responsible providers b) a Patient's Bill of Rights c) how health information will be used and disclosed d) how often family members or friends may visit or call

c) how health information will be used and disclosed HIPAA ensures privacy of information use and disclosure.

When describing the term concept to a group of students, which word would the instructor most likely use? a) fact b) science c) idea d) truth

c) idea A concept, like an idea, is an abstract impression of reality.

When the nurse instructs a client about breast self-examination, the primary purpose of the client education is: a) promotion of self-efficacy and body image. b) promotion of coping. c) illness prevention. d) restoration of function.

c) illness prevention.

The results of a patient's most recent computed tomography (CT) scan indicate that his renal tumor has most likely metastasized to his liver and lungs. The healthcare team has scheduled a family meeting in order to discuss these findings with the patient and his family. Because a vacant meeting room is unavailable at short notice, the family meeting will take place shortly in the patient's room, which is shared by two other patients. This may constitute a direct violation of which of the following? a) the hospital's policies and procedures b) the hospital's risk management program c) the Health Insurance Portability and Accountability Act (HIPAA) d) the state's Nurse Practice Act

c) the Health Insurance Portability and Accountability Act (HIPAA) HIPAA guarantees patients' rights to privacy, and discussion of this patient's health status in a venue where others may overhear the proceedings is likely a contravention of HIPAA. Nurse Practice Acts focus primarily on issues related to scope of practice and professional discipline rather than the ethics of privacy. The hospital's policies and procedures may not directly address invasion of privacy and a risk management program exists primarily to reduce malpractice claims rather than ensure privacy.

The nurse recognizes that the goals established for a patient's discharge are more likely to be accomplished when: a) the multidisciplinary team develops the goals b) the nurse develops the goals c) the patient assists in developing the goals d)the physician develops the goals

c) the patient assists in developing the goals If the patient is involved in establishing the goals, it is more likely that the expected outcomes of the discharge plan will be met. The patient may fail to follow the plan if the goals are not mutually agreed on or are not based on a complete assessment of the patient's needs.

Which of the following parties is responsible for obtaining informed consent from a patient? a) the patient's family or significant other b) the patient's nurse c) the person performing the procedure, study, or treatment d) the nursing supervisor

c) the person performing the procedure, study, or treatment Obtaining informed consent is the responsibility of the person who will be performing the diagnostic or treatment procedure or the research study. The nurse's roles are to confirm that a signed consent form is present in the patient's chart and to answer any patient questions about the consent.

A dialysis nurse is teaching a patient to care for the dialysis access that was inserted in the patient's right arm. The nurse assesses the patient's fears and concerns related to dialysis, the dialysis access, and care of the access. This information is taught over several sessions during the course of the patient's hospitalization. What phase of the working relationship is best described in this scenario? a) the introduction phase b) the orientation phase c) the working phase d) the termination phase

c) the working phase There are three phases of a helping relationship: the orientation phase, the working phase, and the termination phase. The introduction phase is not a valid phase, yet the nurse will introduce herself during the orientation phase. The scenario defines characteristics of the working phase, during which the nurse and patient work together to meet the patient's physical and psychosocial needs. During the orientation phase, the tone and guidelines for the relationship are established. The termination phase occurs when the conclusion of the initial agreement is acknowledged.

The nursing process is based upon the process of problem solving. What type of problem solving is the nurse using if she attempts to obtain a blood pressure on the patient's right arm, the left arm, the left leg, and then finally the right leg where a blood pressure is finally obtained. a) intuitive thinking b) critical thinking c) trial-and-error problem solving d) scientific problem solving

c) trial-and-error problem solving The nurse is using trial-and-error problem solving. This type of problem solving involves testing any number of solutions until one is found that works for that particular problem. In this situation, the nurse attempts to obtain a blood pressure reading on three extremities before finally achieving success on the right leg, and this required the nurse to test a number of locations.

The nurse is caring for a patient who speaks Chinese, and the nurse does not speak Chinese. An appropriate approach for communication with this patient includes: a) writing messages for the patient and offering him a dictionary for translation b) avoiding the use of gesture or play-acting c) using a caring voice and repeating messages frequently d) speaking directly and loudly to the patient

c) using a caring voice and repeating messages frequently Approaches to use when a patient speaks a different language include speaking slowly and distinctly, avoiding loud voices. Using a caring voice, keeping messages simple, and repeating messages frequently. The use of a language dictionary by the nurse is appropriate, but writing messages and asking the patient to translate is not an appropriate approach. Gestures, pictures, and play-acting help the patient understand.

During an interaction with a patient diagnosed with epilepsy, a nurse notes that the patient is silent after she communicates the plan of care. What would be appropriate nurse responses in this situation? Select all that apply. a) Fill the silence with lighter conversation directed at the patient. b) Use the time to perform the care that is needed uninterrupted. c) Discuss the silence with the patient to ascertain its meaning. d) Allow the patient time to think and explore inner thoughts. e) Determine if the patient's culture requires pauses between conversation. f) Arrange for a counselor to help the patient cope with emotional issues.

c, d, e c) Discuss the silence with the patient to ascertain its meaning. d) Allow the patient time to think and explore inner thoughts. e) Determine if the patient's culture requires pauses between conversation.

A nurse is identifying outcomes for a patient who has a leg ulcer related to diabetes. An example of an affective outcome for this patient is a) Within 1 day after teaching, the patient will list three benefits of continuing to apply moist compresses to leg ulcer after discharge. b) By 6/12/15, the patient will correctly demonstrate application of wet-to-dry dressing on leg ulcer. c) By 6/19/15, the patient's ulcer will begin to show signs of healing (e.g., size shrinks from 3″ to 2.5″). d) By 6/12/15, the patient will verbalize valuing health sufficiently to practice new health behaviors to prevent recurrence of leg ulcer.

d

A 76-year-old patient states, "I have been experiencing complications of diabetes." The nurse needs to direct the patient to gain more information. What is the most appropriate comment or question to elicit additional information? a) "Do you take two injections of insulin to decrease the complications?" b) "Most physicians recommend diet and exercise to regulate blood sugar." c) "Most complications of diabetes are related to neuropathy." d) "What specific complications have you experienced?"

d) "What specific complications have you experienced?"

A nurse is teaching patients of all ages in a hospital setting. Which examples demonstrate teaching that is appropriately based on the patient's developmental level? Select all that apply. a) The nurse plans long teaching sessions to discuss diet modifications for an older adult diagnosed with type 2 diabetes. b) The nurse recognizes that a female adolescent diagnosed with anorexia is still dependent on her parents and includes them in all teaching sessions. c) The nurse designs an exercise program for a sedentary older adult male patient based on the activities he prefers. d) The nurse includes an 8-year-old patient in the teaching plan for managing cystic fibrosis. e) The nurse demonstrates how to use an inhaler to an 11-year-old male patient and includes his mother in the session to reinforce the teaching. f) The nurse continues a teaching session on STIs for a sexually active male adolescent despite his protest that "I've heard enough already!"

c, d, e. Successful teaching plans for older adults incorporate extra time, short teaching sessions, accommodation for sensory deficits, and reduction of environmental distractions. Older adults also benefit from instruction that relates new information to familiar activities or information. School-aged children are capable of logical reasoning and should be included in the teaching-learning process whenever possible; they are also open to new learning experiences but need learning to be reinforced by either a parent or health care provider as they become more involved with their friends and school activities. Teaching strategies designed for an adolescent patient should recognize the adolescent's need for independence, as well as the need to establish a trusting relationship that demonstrates respect for the adolescent's opinions.

A nurse performs nurse-initiated nursing actions when caring for patients in a skilled nursing facility. Which are examples of these types of interventions? Select all that apply. a) A nurse administers 500 mg of ciprofloxacin to a patient with pneumonia. b) A nurse consults with a psychiatrist for a patient who abuses pain killers. c) A nurse checks the skin of bedridden patients for skin breakdown. d) A nurse orders a kosher meal for an orthodox Jewish patient. e) A nurse records the I&O of a patient as prescribed by his physician. f) A nurse prepares a patient for minor surgery according to facility protocol.

c, d, f. Nurse-initiated interventions, or independent nursing actions, involve carrying out nurse-prescribed interventions resulting from their assessment of patient needs written on the nursing plan of care, as well as any other actions that nurses initiate without the direction or supervision of another health care professional.

A nurse develops a detailed plan of care for a 16-year-old female who is a new single mother of a premature infant. The plan includes collaborative care measures and home health care visits. When presented with the plan, the patient states, "We will be fine on our own. I don't need any more care." What would be the nurse's best response? a) "You know your personal situation better than I do, so I will respect your wishes." b) "If you don't accept these services, your baby's health will suffer." c) "Let's take a look at the plan again and see if we can adjust it to fit your needs." d) "I'm going to assign your case to a social worker who can explain the services better."

c. When a patient does not follow the plan of care despite your best efforts, it is time to reassess strategy.

A nurse is caring for a patient who is diagnosed with congestive heart failure. Which statement below is not an example of a well-stated nursing intervention? a) Offer patient 60 mL water or juice (prefers orange or cranberry juice) every 2 hours while awake for a total minimum PO intake of 500 mL. b) Teach patient the necessity of carefully monitoring fluid intake and output; remind patient each shift to mark off fluid intake on record at bedside. c) Walk with patient to bathroom for toileting every 2 hours (on even hours) while patient is awake. d) Manage patient's pain.

d

A nurse is transfusing multiple units of packed red blood cells (PRBCs). After the second unit is transfused, the nurse auscultates bilateral crackles at the bases of the client's lungs and the client reports dyspnea. The nurse telephones the health care provider and provides an SBAR report. Which statement represents the final step in this type of communication? a) "I am calling because the client receiving blood has developed dyspnea and had crackles." b) "It seems like this client has fluid volume overload." c) "This client has a medical history of heart failure." d) "I think the client would benefit from intravenous furosemide."

d) "I think the client would benefit from intravenous furosemide." Situation, Background, Assessment, and Recommendations provides a consistent method for hand-off communication that is clear, structured, and easy to use.

A nurse is asking a colleague about a situation. Which statement demonstrates assertive communication? a) "Why are you treating me this way?" b) "What is your problem with me?" c) "You always act like this." d) "I think there is a better way to handle this."

d) "I think there is a better way to handle this."

A nurse is counseling a 19-year-old male athlete who had his right leg amputated below the knee following a motorcycle accident. During the rehabilitation process, the patient refuses to eat or get up to ambulate on his own. He says to the nurse, "What's the point. My life is over now and I'll never be the football player I dreamed of becoming." What is the nurse counselor's best response to this patient? a) "You're young and have your whole life ahead of you. You should focus on your rehabilitation and make something of your life." b) "I understand how you must feel. I wanted to be a famous singer, but I wasn't born with the talent to be successful at it." c) "You should concentrate on other sports that you could play even with a prosthesis." d) "I understand this is difficult for you. Would you like to talk about it now or would you prefer me to make a referral to someone else?"

d) "I understand this is difficult for you. Would you like to talk about it now or would you prefer me to make a referral to someone else?" This answer communicates respect and sensitivity to the patient's needs and offers an opportunity to discuss his feelings with the nurse or another health care professional. The other answers do not allow the patient to express his feelings and receive the counseling he needs.

The client reports to the clinic as ordered by her primary care provider for counseling on weight loss to improve overall health. The client received printed information in the mail to review before the session, and states having read through it before the appointment. Which client statement alerts the nurse to a need for clarification and further education? a) "I can lower my blood pressure by losing weight." b) "I can monitor my caloric intake by measuring portions." c) "Osteoarthritis in my knees may be because of my weight." d) "I will be doing well if I lose between 5 and 10 pounds a week."

d) "I will be doing well if I lose between 5 and 10 pounds a week."

A nurse is discharging a patient from the hospital following a heart stent procedure. The patient asks to see and copy his medical record. What is the nurse's best response? a) "I'm sorry, but patients are not allowed to copy their medical records." b) "I can make a copy of your record for you right now." c) "You can read your record while you are still a patient, but copying records is not permitted according to HIPAA rules." d) "I will need to check with our records department to get you a copy."

d) "I will need to check with our records department to get you a copy."

A nurse enters a patient's room and examines the patient's IV fluids and cardiac monitor. The patient states, "Well, I haven't seen you before. Who are you?" What is the nurse's best response? a) "I'm just the IV therapist checking your IV." b) "I've been transferred to this division and will be caring for you." c) "I'm sorry, my name is John Smith and I am your nurse." d) "My name is John Smith, I am your nurse and I'll be caring for you until 11 p.m."

d) "My name is John Smith, I am your nurse and I'll be caring for you until 11 p.m."

The nurse enters the patient's room ad examines the patient's IV fluids and cardiac monitor. The patient states, "Well, I haven't seen you before. Who are you?" Which of the following is the nurse's best response? a) "I'm just the IV therapist checking your IV." b) "I've been transferred to this division and will be caring for you." c) "I'm sorry, my name is John Smith. I'm responsible for your IV." d) "My name is John Smith. I'll be caring for you until 11pm."

d) "My name is John Smith. I'll be caring for you until 11pm." The nurse should identify himself, be sure the patient knows what will be happening, and the time period he will be with his patient.

The nurse is explaining charting by exception (CBE) to a client who is curious about documentation. Which statement by the nurse is most accurate? a) "CBE is the best way to protect against lawsuits." b) "CBE is a relatively new format of documentation in electronic health records." c) "The benefit of CBE is it demonstrates whether high quality care is given." d) "The benefit of CBE is less time needed on computer charting."

d) "The benefit of CBE is less time needed on computer charting."

A nurse has developed strong rapport with the wife of a patient who has been receiving rehabilitation following a debilitating stroke. Thee wife has just been informed that her husband will be unlikely to return home and will require care that can only be provided in a facility with constant nursing care. The patient's wife tells the nurse, "I can't believe it's come to this." How should the nurse best respond? a) "Do you understand that everyone here has your husband's best interests at heart?" b) "Why do you think that the care team has made this recommendation?" c) "What would help you accept that this is best for both of you?" d) "This must be very difficult for you to hear. How do you feel right now?"

d) "This must be very difficult for you to hear. How do you feel right now?" Asking the wife to elaborate on her feelings may be therapeutic as well as insightful. Asking about the reasons underlying the care team's recommendation is less likely to be of benefit and attempting to redirect the wife to a positive outlook at this early point is insensitive.

A client is meeting with the nurse to discuss options for smoking cessation. Which statement by the nurse is most appropriate for this client? a) "I think you should just go cold turkey." b) "Reduce your stress and it will be easy." c) "Did you know that smoking can cause lung cancer?" d) "What do you see as your biggest hurdle to stopping?"

d) "What do you see as your biggest hurdle to stopping?"

A 76-year-old patient states, "I have been experiencing complications of diabetes." The nurse needs to direct the patient to gain more information. What is the most appropriate comment or question to elicit additional information? a) "Do you take two injections of insulin to decrease the complications?" b) "Most physicians recommend diet and exercise to regulate blood sugar." c) "Most complications of diabetes are related to neuropathy." d) "What specific complications have you experienced?"

d) "What specific complications have you experienced?" Requesting specific information regarding complications of diabetes will elicit specific information to guide the nurse in further interview questions and specific assessment techniques.

During rounds, a charge nurse hears the patient care technician yelling loudly to a patient regarding a transfer from the bed to chair. When entering the room, what is the nurse's best response? a) "You need to speak to the patient quietly. You are disturbing the patient." b)"Let me help you with your transfer technique." c) "When you are finished, be sure to apologize for your rough demeanor." d) "When your patient is safe and comfortable, meet me at the desk."

d) "When your patient is safe and comfortable, meet me at the desk."

You are the charge nurse responsible for the evening shift. During rounds, you hear the patient care technician yelling loudly to a patient regarding his transfer from the bed to chair. When entering the room, which of the following is your best response? a) "You need to speak to the patient quietly. You are disturbing the patient." b) "Let me help you with your transfer technique." c) "When you are finished, be sure to apologize for your rough demeanor." d) "When your patient is safe and comfortable, meet me at the desk."

d) "When your patient is safe and comfortable, meet me at the desk." The charge nurse should direct the patient care technician to determine the patient's safety. Then she or he should address any concerns regarding the patient care technician's communication techniques privately. It is important that the nurse directs the patient care technician on aspects of therapeutic communication.

A nurse helps a patient who has cystic fibrosis prepare a standalone personal health record. Which statement by the nurse best explains this type of information? a) "You can link your record to a specific health care organization's electronic health record system." b) "Your entire health care team may access and securely share your vital medical information electronically." c) "Your health care provider is obligated to read your personal health record and share it with your insurance provider." d) "You can fill in information from your own records and store it on your computer or the Internet."

d) "You can fill in information from your own records and store it on your computer or the Internet." With a standalone personal health record, clients fill in information from their own records, and the information is stored on clients' computers or the Internet.

The nurse is visiting a client who was released from inpatient rehabilitation 6 weeks ago after a 5-month recovery from a motor vehicle accident that left him immobile. As the nurse enters the home, the client braces his hands on the arms of his chair to rise and uses crutches to walk across the room. What is the best response by the nurse? a) "Are you supposed to be out of the wheelchair?" b) "Let me document that you can walk." c) "Those physical therapists work wonders. d) "You have made an amazing recovery."

d) "You have made an amazing recovery."

Which of the following circumstances likely requires the most documentation and communication on the part of the nurse? a) A patient is returning to her assisted-living facility following her colonoscopy earlier that day. b) A patient is being discharged home following a laparoscopic appendectomy 2 days earlier. c) A patient is being transferred from one medical unit of the hospital to another in order to accommodate a patient on isolation precautions. d) A geriatric patient is being transferred from a subacute medical unit to a new long-term care facility following his recovery from pneumonia.

d) A geriatric patient is being transferred from a subacute medical unit to a new long-term care facility following his recovery from pneumonia. Transfer from the hospital setting to a long-term care facility is likely to require significant documentation and communication from the nurses facilitating the transfer. This may include copying the chart or summarizing a large amount of relevant data. Transfers within a hospital typically require somewhat less documentation and communication while discharges home or to an existing facility may not require a formal report of any type.

What is the best broad definition of a family? a) A father, a mother, and children. b) A group whose members are biologically related. c) A unit that includes aunts, uncles, and cousins. d) A group of people who live together.

d) A group of people who live together. Although all the responses may be true, the best definition is a group of people who live together.

In which of the following situations would the SBAR technique of communication be most appropriate? a) A nurse is explaining the process of bone marrow biopsy to a patient who is scheduled for the procedure. b) A nurse is teaching a patient about the benefits of smoking cessation and the risks of continuing to smoke. c) A nurse is facilitating a family meeting in order to coordinate a patient's discharge planning. d) A nurse is calling a physician to report a patient's new onset of chest pain.

d) A nurse is calling a physician to report a patient's new onset of chest pain. There are numerous applications of the SBAR technique of communication, including nurse-physician communication surrounding acute patient developments. The technique is not normally applied in patient education or in communication between the healthcare team and patients' families.

Which of the following patients is the most appropriate candidate for hospice services? a) A patient with Guillain-Barre undergoing plasmapheresis b) A patient on hemodialysis who works as a schoolteacher c) A patient with newly diagnosed breast cancer undergoing chemotherapy and radiation therapy d) A patient with lung cancer and a 3-4 month life expectancy

d) A patient with lung cancer and a 3-4 month life expectancy The patient with lung cancer and a 3-4 month life expectancy is a candidate for hospice care. Hospice care provides support and services to dying patients and their families. The patient with breast cancer is undergoing treatment for the cancer and is not a candidate for hospice. An active hemodialysis patient and a patient seeking treatment for Guillain-Barre are also not candidates for hospice care.

For a client with self-care deficit, the long-term goal is that the client will be able to dress himself by the end of the 6-week therapy. For best results, when should the nurse evaluate the client's progress toward this goal? a) only when the client shows some progress b) when the client is discharged c) at the end of the 6-week therapy d) as soon as possible

d) as soon as possible

An employee health nurse is assisting a stressed working mother with value clarification. Which of the following best defines value clarification? a) A systematic inquiry into principles of right and wrong conduct, of virtue and vice, and of good and evil, as they relate to conduct. b) A belief about the worth of something, about what matters, that acts as a standard to guide one's behavior. c) An organization of values in which each is ranked along a continuum of importance, often leading to a personal code of conduct. d) A process by which people come to understand their own values and value systems.

d) A process by which people come to understand their own values and value systems. Value clarification is a belief about the worth of something, about what matters, that acts as a standard to guide one's behavior. A value system is an organization of values in which each is ranked along a continuum of importance, often leading to a personal code of conduct. Ethics is a systematic inquiry into principles of right and wrong conduct, of virtue and vice, and of good and evil, as they relate to conduct.

An adolescent confides in the school nurse that she is arguing daily with her mother and she often wonders if her mother loves her. The school nurse recognizes that the student faces which of the following risk factors for altered family health? a) A biologic risk factor. b) A lifestyle risk factor. c) A developmental risk factor. d) A psychosocial risk factor

d) A psychosocial risk factor: Conflicts between family members are considered psychosocial risk factors.

Mr. Grewal's health maintenance organization (HMO) dictates a fixed, prospective reimbursement to the hospital that will perform his pending hip replacement surgery. This structure of payment may contribute to which of the following? a) Decreased need for home healthcare b) Increased intensity of rehabilitation while in hospital c) Continuity of care d) A shortened length of stay in hospital

d) A shortened length of stay in hospital Prospective reimbursement is a practice that has been identified as a contributor to decreased lengths of hospital stays and a consequent increase in the need for home healthcare. This phenomenon increases the patient's need for continuity of care, but it does not promote the practice in and of itself. Prospective reimbursement does not necessarily result in more intense rehabilitation while the patient is in the hospital.

Which of the following is an emerging trend in healthcare delivery? a) Reduction in the use of technology at the bedside b) Resolution of the nursing shortage c) Simplification of patient care d) Active involvement of consumers

d) Active involvement of consumers Trends in healthcare delivery include the active involvement of consumers, continuing nursing shortage, increased complexity of patient care, and a technology explosion. Other trends include changing demographics, increasing diversity, globalization of economy and society, increasing costs of healthcare, and the effects of health policy and regulation.

Which of the following is one of the developmental tasks of the older adult family? a) Maintain a supportive home base b) Prepare for retirement c) Cope with loss of energy and privacy d) Adjust to loss of spouse

d) Adjust to loss of spouse A developmental task of the older adult family is adjusting to the loss of a spouse.

Which of the following activities related to respiratory health is an example of tertiary health promotion and illness prevention? a) Assisting with lung function testing of a patient to help determine a diagnosis. b) Teaching a patient that "light" cigarettes do not prevent lung disease. c) Advocating politically for more explicit warning labels on cigarette packages. d) Administering a nebulized bronchodilator to a patient who is short of breath.

d) Administering a nebulized bronchodilator to a patient who is short of breath. The use of medications is characteristic of tertiary health promotion and illness prevention. Testing and screening are examples of secondary health promotion and illness prevention, while patient education and political advocacy are associated with primary prevention.

Which of the following factors is the primary criterion for eligibility in the federally funded Medicare program? a) Diagnosis with more than one chronic health condition b) Low income c) Lack of enrollment in a group health plan d) Age greater than 65

d) Age greater than 65 Medicare is available to those over age 65 and to those under 65 who are disabled or who require dialysis. Many recipients have multiple chronic health problems, a low income, or are not enrolled in a group health plan; however, these factors do not determine eligibility.

A fellow student asks you about your legal liability when you do your clinical practice. Which of the following are true? (1) Student nurses are responsible for their own acts of negligence if these result in patient injury. (2) Student nurses are held to the same standard of care that would be used to evaluate the actions of a registered nurse. (3) A hospital may also be held liable for the negligence of a student nurse enrolled in a hospital-controlled program because the student is considered an employee of the hospital. (4) Nursing instructors may share a student's responsibility for damages in the event of patient injury if the instructor failed to provide reasonable and prudent clinical supervision. a) (1) and (3) b) (2) and (4) c) (1), (2), (3) d) All of the above

d) All of the answers are true.

Which of the following organizations is the best source of information when a nurse wishes to determine whether an action is within the scope of nursing practice? a) National League for Nursing (NLN) b) International Council of Nurses (ICN) c) American Association of Colleges in Nursing (AACN) d) American Nurses Association (ANA)

d) American Nurses Association (ANA) The ANA produces the 2003 Nursing: Scope and Standards of Practice, which defines the activities specific and unique to nursing. The AACN addresses educational standards while the NLN promotes and fosters various aspects of nursing. The ICN provides a venue aspects of nursing organizations to collaborate but does not define standards and scope of practice.

A patient has an order for an analgesic medication to be given PRN. When would the nurse administer this medication? a) Every three hours b) Every four hours c) Daily d) As needed

d) As needed

A nurse administering medications accidentally gives a double dose of blood pressure medications. After ensuring the safety of the client, the nurse would document the error in which documents? a) Occurrence report and critical pathway b) Critical pathway and care plan c) Care plan and client's record d) Client's record and occurrence report

d) Client's record and occurrence report An occurrence report should be completed when a planned intervention is not implemented as ordered. The incident, with actions taken by the nurse, should also be included in the client's record.

A nurse has drafted an SBAR communication before contacting the primary care provider of a patient whose condition has worsened. How should the nurse best conclude this communication? a) Provide the most likely diagnosis of the problem. b) Provide the patient's most recent vital signs. c) Ask the care provider if he or she is familiar with this patient. d) Ask the care provider to come and assess the patient.

d) Ask the care provider to come and assess the patient. The final phase of an SBAR communication involves making a recommendation. In the case of a patient whose condition is worsening, this may entail recommending that the primary care come to assess the patient. Asking the care provider if he or she is familiar with the patient should be done early in the communication, while providing assessment data and possible diagnoses are addressed in the background and assessment sections of the tool.

The care plan for a client who has been frequently admitted to the hospital for exacerbation of COPD (chronic obstructive pulmonary disease) has a nursing diagnosis of "Noncompliance related to lack of knowledge as evidenced by frequent admissions to the hospital." What is the most appropriate method for the nurse to use to validate the nursing diagnosis? a) Assess the severity of the client's illness. b) Assess the client's access to health care. c) Assess the client's financial resources. d) Assess the client's knowledge of COPD.

d) Assess the client's knowledge of COPD.

The nurse is working with a 26-year-old smoker who is a client in an outpatient center. The client states that she is very committed to quitting smoking to improve her health as well as to provide a good example for her young son. The nurse knows that participating in a smoking cessation support group is a key component to smoking cessation. During discussion, the nurse asks the client "on a scale of 0 to 10, how likely are you to attend a support group?" Which strategy of motivational interviewing is the nurse using with the client? a) Elicit-provide-elicit b) Evoking change talk c) Rating d) Assessing importance

d) Assessing importance Using the 0 to 10 scale is a key aspect of assessing importance with motivational interviewing. It helps the nurse to understand the client's feelings toward the recommended activity and can help start a conversation about why the client chose that rating number—and what the nurse could do to increase the number.

For which of the following research activities is a phenomenological research method most appropriate? a) Understanding how patients cope with a new diagnosis of Alzheimer's disease. b) Explaining how the particular characteristics of the Vietnam War affected the roles of nurses. c) Describing the health maintenance activities that are practiced by homeless intravenous drug users. d) Attempting to understand non-English speaking immigrants' experiences of being hospital patients.

d) Attempting to understand non-English speaking immigrants' experiences of being hospital patients. Phenomenology is the type of qualitative research that attempts to understand individuals' experiences and the meanings they assign to these. Research that is undertaken within a specific culture, such as that of homeless intravenous drug users, is characteristic of ethnography. Examining the significance of past events, such as war, is associated with historical qualitative research. Processes such as coping are the most common focuses of grounded theory.

A homeless client in the public health clinic has a strong body odor and is wearing clothes that are visibly soiled. What nursing diagnosis would be most appropriate for the nurse to identify? a) Impaired Impulse Control related to poor socioeconomic conditions as evidenced by visibly soiled clothing b) Inadequate Hygiene related to homelessness as evidenced by client's stink c) Homelessness Syndrome related to lack of housing as evidenced by visibly soiled clothing d) Bathing Self-care Deficit related to lack of access to bathing facilities as evidenced by a strong body odor

d) Bathing Self-care Deficit related to lack of access to bathing facilities as evidenced by a strong body odor

If you harm a patient by administering a medication (wrong drug, wrong dose, etc.) ordered by a physician, which of the following is true? a) You are not responsible, since you were merely following the doctor's orders. b) Only you are responsible, since you actually administered the medication. c) Only the physician is responsible, since he or she actually ordered the drug. d) Both you and the physician are responsible for your respective actions.

d) Both you and the physician are responsible for your respective actions. Nurses are legally responsible for carrying out the orders of the physician in charge of a patient unless an order would lead a reasonable person to anticipate injury if it were carried out. If the nurse should have anticipated injury and did not, both the prescribing physician and the administering nurse are responsible for the harms to which they contributed.

A nurse was informed that a family member was involved in a car accident and transported to the emergency department in the same facility. What action by the nurse best demonstrates understanding of client privacy? a) Accessing the electronic medical record of the family member to find out extent of injury b) Asking the emergency department nurse for information on the family member c) Finding the emergency medical technicians that transported the family members about the injuries d) Calling the client information desk to find out the room number of the family member

d) Calling the client information desk to find out the room number of the family member

Which agency focuses on the epidemiology, prevention, control and treatment of communicable diseases? a) Indian Health Services b) National Institutes of Health c) Veterans Administration d) Centers for Disease Control and Prevention

d) Centers for Disease Control and Prevention The Centers for Disease Control and Prevention (CDC) in Atlanta focuses on the epidemiology, prevention, control and treatment of communicable diseases. The National Institutes of Health engages in funding and conducting various health research activities. The Veterans Administration provides healthcare to veterans. Indian Health Services is the principal source of support of Native American healthcare.

A hospital is changing the format for documentation in an attempt to decrease the amount of time the nurses are spending on charting. The new type of charting will require that the nurses document the significant findings as a narrative note, in a shorthand method using well-defined standards of practice. Which of the following best defines this type of charting? a) Variance charting b) FOCUS charting c) Problem, Intervention, Evaluation (PIE) charting d) Charting by exception (CBE)

d) Charting by exception (CBE) Charting by exception (CBE) is a shorthand documentation method that makes use of well-defined standards of practice; only significant findings or "exceptions" to these standards are documented in the narrative notes.

During the course of assessing the family structure and behaviors of a pediatric patient's family, the nurse has identified a number of highly significant risk factors. Which of the following actions should the nurse prioritize when addressing these risk factors? a) Validate the family's unique way of being. b) Introduce the family to a family that possesses fewer risk factors. c) Enlist the help of community and social support. d) Engage in appropriate health promotion activities.

d) Engage in appropriate health promotion activities. The role of the nurse in reducing risk factors involves activities that promote health for all family members at any level of development. This consideration supersedes the importance of validating the family's present way of being or enlisting the help of others. Introducing the family to a "model" family is ethically and logistically questionable.

When maintaining medical records for a client, the nurse knows that a medical record also serves as a legal document of evidence. What should the nurse do to ensure legal defensible charting? a) Leave spaces between entries and signature. b) Use abbreviations wherever possible. c) Record all facts and subjective interpretations. d) Ensure that the client's name appears on all pages.

d) Ensure that the client's name appears on all pages. The nurse should ensure that the client's name appears on all pages to ensure legally defensible charting.

A hospital patient has an aggressive fungal infection in his right eye that has necessitated evisceration (removal of the eye). Consequently, the patient requires twice-daily packing and dressing changes to his orbit. Which of the nurse's following actions in the care of this patient most clearly demonstrates interpersonal skills? a) Understanding the anatomy and physiology of the affected parts of the patient's body. b) Maintaining aseptic technique when performing the dressing change. c) Documenting the condition of the patient's orbit and the procedure of the dressing change in an accurate and timely manner. d) Ensuring the patient's privacy during dressing changes and providing an explanation during the procedure.

d) Ensuring the patient's privacy during dressing changes and providing an explanation during the procedure. A central aspect of interpersonal skills is maintaining privacy and dignity, as well as keeping patients informed during their care. Documentation is an outcome of legal/ethical skills while knowledge of anatomy and physiology demonstrates cognitive skill. The maintenance of asepsis involves technical skill.

A nurse is formulating a clinical question in PICO format. Which of the following is represented by the letter P? a) Comparison to another similar treatment. b) Clearly defined, focused literature review. c) Specific identification of the desired outcome. d) Explicit descriptions of population of interest.

d) Explicit descriptions of population of interest. The P in the PICO format represents an explicit description of the patient population of interest.

Priority setting is based on the information obtained during reassessment. Priority setting is used to rank nursing diagnoses. Each of the following contributes to priority setting except which of the following? a) The client's condition b) Feedback from the family c) Time and resources d) Finances of the client

d) Finances of the client

Which nurse in history is credited with establishing nursing education? a) Clara Barton b) Lillian Wald c) Lavinia Dock d) Florence Nightingale

d) Florence Nightingale Florence Nightingale established nursing education.

A nurse is caring for a 45-year-old male client who lost function in both of his legs due to an automobile accident. Which of the following should the nurse do first to personalize the learning? a) Prepare the training plan for the client. b) Develop confidence in the client. c) Analyze the client's behavior. d) Gather pertinent information from the client.

d) Gather pertinent information from the client. To personalize the learning, the nurse must first gather pertinent information from the client. Analyzing the client's behavior, preparing the training plan for the client, and developing confidence in the client are the next steps to personalize the learning.

A nurse is looking for trends in a postoperative patient's vital signs. Which documents would the nurse consult first? a) Admission sheet b) Admission nursing assessment c) Activity flow sheet d) Graphic record

d) Graphic record While one recording of vital signs should appear on the admission nursing assessment, the best place to find sequential recordings that show a pattern or trend is the graphic record. The admission sheet does not include vital sign documentation, and neither does the activity flow sheet.

When providing health promotion classes, a nurse uses concepts from models of health. What do both the health-illness continuum and the high-level wellness models demonstrate? a) illness as a fixed point in time. b) The importance of family. c) Wellness as a passive state. d) Health as a constantly changing state.

d) Health as a constantly changing state. Both these models view health as a dynamic (constantly changing state).

A nurse educator uses models of health and illness when teaching. Which model of health and illness places high-level health and death on opposite ends of a graduated scale? a) Agent-Host-Environment Model b) Health Belief Model c) Health Promotion Model d) Health-Illness Continuum

d) Health-Illness Continuum The Health-Illness Continuum views health as a constantly changing state, with high-level wellness and death being on opposite ends of a graduated scale. The Agent-Host-Environment Model is useful in examining the causes of disease in an individual. The Health Belief Model describes health behaviors. The Health Promotion Model incorporates individual characteristics and experiences and behavior-specific knowledge and beliefs to motivate health-promoting behavior.

Which of the following best defines nursing diagnoses? a) Identification of client problems that require collaboration with other health care professionals b) Identification of signs and symptoms that identify diseases c) Identification of actual client problems, not including potential problems d) Identification of client problems that nurses can treat independently

d) Identification of client problems that nurses can treat independently

A physician is in a hurry to leave the unit and tells the nurse to give a morphine 2 mg IV every 4 hours as needed for pain. What action by the nurse is appropriate? a) Call the pharmacy to have the ordered entered in the electronic record. b) Read back the order and write the order in the client's record. c) Add the new order to the medication administration record. d) Inform the physician that a written order is needed.

d) Inform the physician that a written order is needed. Verbal orders should only be accepted during an emergency. No other action is correct other than asking the physician to write the order.

Which statement best describes community-based care? a) It requires coordination by a physician or healthcare corporation. b) It is population-based and focuses on the health of the community. c) It focuses on the care of an individual in the hospital setting. d) It centers on individual and family healthcare needs.

d) It centers on individual and family healthcare needs. Community-based care is centered on individual and family healthcare needs and is provided to people who live within a defined geographic area. In contrast, community health and public health nursing are population-based and focus on the health of the community. Community-based care considers the continuity of care as an individual moves from one level or setting to another, and the nurse is often the primary person responsible for communicating the patient's needs, teaching self-care, and providing the care.

What nursing theorist focuses on rehabilitation as a major outcome of nursing care? a) Jean Watson b) Martha Rogers c) Sr. Callista Roy d) Lydia Hall

d) Lydia Hall The central theme of Lydia's Hall nursing theory has a focus on rehabilitation. The major outcome of nursing care under her theory focuses on rehabilitation and feelings of self-actualization by the patient. Martha Rogers emphasized the science and art of nursing, with the unitary human being central to the discipline of nursing. Jean Watson was concerned with promoting and restoring health, preventing illness, and caring for the sick. Sr. Callista Roy's theme focused on human beings as biopsychosocial beings existing within an environment.

A home healthcare nurse is performing a home visit to a 58-year-old man and his 56-year-old wife who receives home chemotherapy as part of her treatment regimen for breast cancer. The nurse will recognize that this family is likely to be engaged in which of the following development tasks? a) Adjusting to retirement b) Moving from the family home c) Adjusting to the loss of a spouse d) Maintaining ties with older and younger generations

d) Maintaining ties with older and younger generations This couple is likely to have children who are middle-aged adults, in which case the task of maintaining ties with older and younger generations is important. The couple is less likely to be retired or moving from their home and it would be presumptuous to assume the loss of a spouse.

Which of the following nursing degrees prepares a nurse for advanced practice as a clinical specialist or nurse practitioner? a) LPN b) ADN c) BSN d) Master's

d) Master's A master's degree prepares advanced practice nurses.

A student health nurse is counseling a female college student who wants to lose 20 pounds. The nurse develops a plan to increase the student's activity level and decrease the consumption of the wrong types of foods and excess calories. The nurse plans to evaluate the student's weight loss monthly. When the student arrives for her first "weigh-in," the nurse discovers that instead of the projected weight loss of 5 pounds, the student has lost only 1 pound. Which is the best nursing response? a) Congratulate the student and continue the plan of care. b) Terminate the plan of care since it is not working. c) Try giving the student more time to reach the targeted outcome. d) Modify the plan of care after discussing possible reasons for the student's partial success.

d) Modify the plan of care after discussing possible reasons for the student's partial success.

A nurse is counseling a 60-year-old female patient who refuses to look at or care for a new colostomy. She tells the nurse, "I don't care what I look like anymore, I don't even feel like washing my hair, let alone changing this bag." The nurse diagnoses Altered Health Maintenance. This is an example of what type of problem? a) Collaborative problem b) Interdisciplinary problem c) Medical problem d) Nursing problem

d) Nursing problem Nursing Problem, because it describes a problem that can be treated by nurses within the scope of independent nursing practice. Collaborative and interdisciplinary problems require a teamwork approach with other health care professionals to resolve the problem. A medical problem is a traumatic or disease condition validated by medical diagnostic studies.

At what point during a hospital stay should discharge planning be initiated? a) After surgery and successful recovery. b) After the patient is less anxious. c) Immediately before discharge. d) On admission to the acute care setting.

d) On admission to the acute care setting. Effective discharge planning begins on admission.

A client has been diagnosed with a recent myocardial infarction. What collaborative problem would be the priority for the nurse to address? a) PC: Fear related to new diagnosis of myocardial infarction b) PC: Activity intolerance related to decreased oxygenation capacity c) PC: Disturbed body image related to decreased activity tolerance d) PC: Decreased cardiac output related to cardiac tissue damage

d) PC: Decreased cardiac output related to cardiac tissue damage All these collaborative problems may be indicated for a client with a recent myocardial infarction; however, priority must be given to life threatening issues. Decreased cardiac output is life threatening so it must be the priority concern.

A group of students are reviewing information about hospitals in preparation for a quiz the next day. Which of the following, if identified by the students as associated with a hospital, demonstrates the need for additional study? a) Provision of acute care b) Inpatient services c) Outpatient services d) Parish nursing activities

d) Parish nursing activities Hospitals are acute care providers, providing services to both inpatients and outpatients.

Following the birth of his first child and after reading about the long-term effects of nicotine, John decides to stop smoking. This behavior change is most likely based on John's perceptions of all but one of the following. Which one is not true? a) His susceptibility to lung cancer. b) How serious lung cancer would be. c) What benefits his stopping smoking will have. d) Personal choice and economic factors.

d) Personal choice and economic factors. Responses a, b and c are components of the health belief model.

The emergency room has a strict protocol regarding IM (intramuscular) injection technique. A nurse working in the emergency room has learned of a new technique to decrease pain with IM injections and would like to use it. What is the most appropriate way for the nurse to implement the technique? a) Ask the ER physician to order IM injections with the new technique. b) Research the protocols at other area emergency rooms. c) Begin using the technique to determine if it is effective. d) Petition to change the protocol based on the new evidence.

d) Petition to change the protocol based on the new evidence.

A group of nurses are planning to investigate the effectiveness of turning immobilized stroke patients more frequently in order to prevent skin breakdown. The team has begun by formulating a PICO question. Which of the following will the "O" in the teams PICO question refer to? a) The currently-used turning schedule b) Turning patients more frequently c) Patients who have experienced a stroke d) Preventing skin breakdown

d) Preventing skin breakdown Within the PICO question framework, the "O" denotes the outcome of interest. In this case, the desired outcome is the prevention of skin breakdown. Stroke patients are the "P", or population of interest, while turning patients more frequently is the "I", or intervention. The current turning schedule is the "C", referring to the comparison of interest.

A nurse documents the following patient data in the patient record according to the SOAP format: Patient complains of unrelieved pain; patient is seen clutching his side and grimacing; patient pain medication does not appear to be effective; Call in to primary care provider to increase dosage of pain medication or change prescription. This is an example of what charting method? a) PIE charting method b) Source-oriented method c) Focus charting method d) Problem-oriented method

d) Problem-oriented method The problem-oriented method is organized around a client's problems rather than around sources of information. With this method, all health care professionals record information on the same forms. The advantages of this type of record are that the entire health care team works together in identifying a master list of patient problems and contributes collaboratively to the plan of care.

The Joint Commission is conducting an accreditation visit at the hospital. What is the focus of the evaluation being conducted? a) Magnet status b) Quality improvement c) Peer review d) Quality assurance

d) Quality assurance

A nurse just reported to the oncoming shift that she had failed to do an ordered dressing change. She reported to the nurse manager that this was the second time this week she had not had time to do the dressing change. The nurse manager recognized that the nurse normally was very punctual and was known to provide good care for her clients however the unit census had been very high on this particular week. The nurse manager knows that quality care must be provided and reports this occurrence as what type of quality approach? a) Quality by inspection b) Quality by design c) Quality by promotion d) Quality by opportunity

d) Quality by opportunity Quality by opportunity is focused on finding opportunity to improve quality through teamwork and sharing in a nonthreatening environment. Mistakes are seen as a breakdown in the system rather than a lack of motivation or competence of the nurse.

Which of the following skills is foundational for a nursing student's participation in the evidence-based practice process? a) Become involved in evidence-based practice research. b) Publish an article on evidence-based practice findings. c) Critically analyze research articles and the findings. d) Read and understand a research article.

d) Read and understand a research article. The use of evidence-based research requires critical analysis and extensive, systematic reviews of research articles and findings. However, a beginning nursing student has not acquired the knowledge to publish new research. The first stop for a student is to be able to read and understand a research article.

A postmastectomy patient is due to come up to postsurgical care unit from postanesthetic recovery. Which of the following nursing actions should the nurse prioritize when attempting to establish an effective relationship with her? a) Assess the patient's knowledge of her activity limitations. b) Explain and answer questions about the Health Insurance Protability and Accountability Act (HIPAA). c) Address the patient's potential learning needs. d) Recognize and address the patient's anxiety.

d) Recognize and address the patient's anxiety. An early priority when admitting a patient to a unit and establishing a relationship is to recognize and take steps to reduce the patient's anxiety. Assessing and addressing the patient's learning needs are important goals but should be addressed after the patient has been settled on the unit. HIPAA should have been explained to the patient earlier in her admission.

A patients with a diagnosis of colorectal cancer has been presented with her treatment options but wishes to defer any decisions to her uncle, who acts in the role of a family patriarch within the patient's culture. Which of the following is the patient's right to self-determination best protected by? a) Holding a family meeting and encouraging the patient to speak on her own behalf. b) Revisiting the decision when the uncle is not present at the bedside. c) Teaching the patient about her right to autonomy. d) Respecting the patient's desire to have the uncle make choices on her behalf.

d) Respecting the patient's desire to have the uncle make choices on her behalf. The right to self-determination (autonomy) means that it should never be forced on anyone. The patient has the autonomous right to defer her decision making to another individual if she freely chooses to do so.

The wife of an elderly man with Alzheimer's disease contacts a community health nurse because she is planning to visit her sister in a neighboring state and needs assistance in arranging care for her husband while she is away. Which type of care is the wife seeking? a) Hospice services b) Parish nursing services c) Rehabilitation care d) Respite care

d) Respite care The wife of the patient is seeking respite care. Respite care is a type of care provided for caregivers of homebound ill, disabled or elderly patients. The main purpose is to give the primary caregiver some time away from the responsibilities of day-to-day care. Hospice services provide care to dying persons and their family members. Rehabilitation care specializes in services for patients requiring physical or emotional rehabilitation and for treatment of chemical dependency. Parish nursing is often volunteer-based and situated within the church.

Which of the following errors has the nurse made in formulating the following nursing diagnosis: Prolonged Immobility related to impaired skin integrity AEB one-inch diameter open area on right buttocks surrounded by a one-inch margin of redness; wound surface clean and beefy red; no drainage or foul odor detected. a) Omitted the defining characteristics of the client health problem b) Writing the diagnosis in terms of a need rather than a client response c) Identified environmental factors rather than client factors as the problem d) Reversed the health problem and the etiology

d) Reversed the health problem and the etiology

Mr. Nguyen was admitted to the hospital 2 weeks ago following an ischemic stroke. Following the early introduction of stroke rehabilitation, he has seen significant improvements in both his medical status and activities of daily living (ADLs). This morning, however, his nurse notes that Mr. Nguyen has been coughing since he ate his minced and pureed breakfast and auscultation of his chest reveals the presence of coarse crackles. Which of the following practitioners should the nurse liaise with in order to obtain a swallowing assessment? a) Respiratory therapist b) Physical therapist c) Physician d) Speech therapist

d) Speech therapist The diagnosis and treatment of dysphagia (swallowing problems) is within the purview of speech therapists. The physician should be made aware and respiratory therapy may be involved with assessing and promoting the patient's oxygenation, but swallowing assessment is a task most often performed by a speech therapist.

The charge nurse is reviewing SOAP format documentation with a newly hired nurse. What information should the charge nurse discuss? a) The plan includes interventions, evaluation, and response. b) Abnormal laboratory values are common items that are documented. c) Objective data is what the client states about the problem. d) Subjective data should be included when documenting.

d) Subjective data should be included when documenting.

A client is reluctant to learn to do finger sticks for her home INR monitoring. What is the best statement by the nurse? a) Most people are afraid of sticking themselves. b) Are you worried about the pain? c) Why don't you want to do this? d) Tell me what you know about these tests.

d) Tell me what you know about these tests.

Injuries related to lifting or transferring patients occur in the healthcare setting and may be considered a work-related injury. Which law was intended to reduce work-related injuries and illnesses? a) The Health Care Quality Improvement Act of 1986 b) Americans with Disabilities Act of 1990 c) Title VII of the Civil Rights Act of 1964 d) The Occupational Safety and Health Act of 1970

d) The Occupational Safety and Health Act of 1970 The Occupational Safety and Health Act of 1970 set legal standards in the United States in an effort to ensure safe and healthful working conditions for men and women. The Health Care Quality Improvement Act of 1986 was enacted to encourage healthcare practitioners to identify and discipline practitioners who engage in unprofessional conduct and to restrict the ability of incompetent practitioners to move from state to state without disclosure of the practitioner's previous performance. Title VII of the Civil Rights Act of 1964 protects employees from discrimination. The Americans with Disabilities Act of 1990 prohibits discrimination against disabled people and requires covered entities to reasonably accommodate individuals who are protected by the Act.

A registered nurse has had her license suspended after being convicted of being impaired at work. What governing body has the authority to revoke or suspend a nurse's license? a) The National League for Nursing b) The Supreme Court c) The employing healthcare institution d) The State Board of Nurse Examiners

d) The State Board of Nurse Examiners The State Board of Nurse Examiners in the United States may revoke or suspend a nurse's license or registration for drug or alcohol abuse, which is the most common reason.

A Chinese client who was previously treated at the health care facility for an open wound has been admitted again because the wound has become gangrenous. It has been identified that the client failed to understand proper wound care. What is the probable reason for the client failing to understand the instruction? a) The client has a short attention span. b) The client is a passive learner. c) The client is not interested. d) The client belongs to a different culture.

d) The client belongs to a different culture. The probable reason for the client failing to follow proper care for the wound could be that the client belongs to a different culture. As a result, he failed to understand the language in which the nurse imparted the instructions. If the client is a passive learner or has short attention, he would have retained at least part of the instructions. The client being uninterested is also not a reason for his not understanding the instructions.

A male client has been recently diagnosed with diabetes after receiving emergency treatment for a hyperglycemic episode. Which of the client's actions indicates that he has achieved a cognitive outcome in the management of his new health problem? a) The client's blood sugars have been maintained within acceptable range in the days prior to discharge. b) The client expresses a desire to change the way that he eats and the amount of exercise he performs. c) The client can demonstrate the correct technique for using his new glucometer. d) The client is able to explain when and why he needs to check his blood sugar.

d) The client is able to explain when and why he needs to check his blood sugar.

A client has been diagnosed with PVD. What area of the body should the nurse focus the assessment? a) The abdominal area b) Heart rate and rhythm c) Lung sounds d) The lower extremities

d) The lower extremities Peripheral vascular disease (PVD) mostly affects the lower extremities.

A nurse is collecting evaluative data for a patient who is finished receiving chemotherapy for an osteosarcoma. Which nursing action represents this step of the nursing process? a) The nurse collects data to identify health problems. b) The nurse collects data to identify patient strengths. c) The nurse collects data to justify terminating the plan of care. d) The nurse collects data to measure outcome achievement.

d) The nurse collects data to measure outcome achievement.

Which of the following is the most important indicator of quality nursing care? a) The nurse follows the policies and procedures of the institution. b) The nurse takes measures to ensure accurate medication administration. c) The nurse is organized and efficient in client care. d) The nurse considers the individual needs of clients.

d) The nurse considers the individual needs of clients. The personal side of nurse is the most important indicator of quality nursing care. Considering the individual needs of the clients demonstrates the nurse's belief in the importance of the client. Being organized and efficient, following policies and procedures, and ensuring accurate medication administration are important parts of nursing care but are mainly task oriented.

The nurse enters the room of a patient with cancer. He is crying and states, "I feel so alone." Of the following statements, which is the most therapeutic? a) The nurse stands at the patient's bedside and states, "I understand how you feel. My mother said the same thing when she was ill." b) The nurse places a hand on the patient's arm and states, "You feel so alone." c) The nurse stands in the patient's room arm and asks, "Why do you feel so alone? Your wife has been here every day." d) The nurse holds the patient's hand and asks, "What makes you feel so alone?"

d) The nurse holds the patient's hand and asks, "What makes you feel so alone?" The use of Therapeutic Touch conveys acceptance, and the implementation of an open-ended question allows the patient time to verbalize freely.

The nurse has entered a patient's room after receiving morning report, rapidly assessed the patient's airway, breathing, and circulation and greeted the patient by saying, "good morning." The patient has made no reciprocal response to the nurse. How should the nurse best respond to the patient's silence? a) The nurse should ask the patient if he feels afraid or angry. b) The nurse should document the patient's withdrawal and diminished mood in the nurse's notes. c) The nurse should apologize for bothering the patient, perform necessary assessments efficiently and leave the room. d) The nurse should ask appropriate questions to understand the reasons for the patient's silence.

d) The nurse should ask appropriate questions to understand the reasons for the patient's silence. Silence can have many meanings, and the nurse should attempt to identify the meaning of the patient's silence in a tactful manner. Directly asking if the patient is angry or fearful is likely presumptuous and my harm rapport. The nurse should not make assumptions around the patient's mood nor should the nurse cease to engage with the patient.

With which of the following groups should the nurse apply the COPE (Creativity; Optimism; Planning; Expert Information) model to facilitate learning? a) Infection control nurses who conduct hand washing audits in a hospital. b) A group of junior high school students who are being taught about nutrition. c) Nurses who provide home healthcare in an inner-city setting. d) The parents of a teenager who has just been diagnoses with type 2 diabetes.

d) The parents of a teenager who has just been diagnoses with type 2 diabetes. The COPE model is specifically aimed at helping family members become effective problem solvers and support a nurse's teaching efforts. Consequently, the parents of a child who has a new diagnosis are appropriate subjects for this model.

A nurse is teaching a patient with a new diagnosis of diabetes. Which example demonstrates cognitive learning by the patient? a) The patient expresses a desire to improve his nutritional intake and lose weight. b) The patient demonstrates proper technique for injecting insulin. c) The patient prepares the skin for the administration of an insulin injection. d) The patient describes signs and symptoms of hypoglycemia.

d) The patient describes signs and symptoms of hypoglycemia. The patient's ability to describe the signs and symptoms of hypoglycemia demonstrates cognitive learning (the storing and recalling of new knowledge in the brain). Demonstrating a skill, such as insulin injection, is an example of psychomotor learning. Effective learning includes changes in attitudes, values, and feelings (e.g., desire to lose weight).

The nurse is preparing to begin the discharge planning process with a patient whose pulmonary embolism has recently resolved. Which of the following factors should the nurse prioritize during this process? a) The nurse's knowledge base and experience level b) The patient's potential for recurrence c) The NANDA diagnoses relevant to the patient's condition d) The patient's identified needs and goals

d) The patient's identified needs and goals The central focus of patient teaching and the larger discharge planning process should be the identified healthcare needs of the patient and the goals that he or she identifies or acknowledges. The nurse's skills and knowledge, the patient's potential for recurrence, and the relevant NANDA Nursing Diagnoses are all elements that may inform the discharge planning process, but they are superseded by the patient's goals and expressed needs.

What criterion must be met for home care visits to be reimbursed? a) The nurse must visit the patient daily and complete daily documentation of the visit. b) The patient's home must have adequate room for storage of supplies and work space for the healthcare personnel. c) The patient's family must be present for each home care visit and offer to assist. d) The physician must write an order for all services, and the patient must meet eligibility criteria.

d) The physician must write an order for all services, and the patient must meet eligibility criteria. For home healthcare visits to be reimbursed, the physician must write an order for all services and the patient must meet eligibility criteria.

A 3-year-old child is being admitted to a medical division for vomiting, diarrhea, and dehydration. During the admission interview, the nurse should implement which communication techniques to elicit the most information from the parents? a) The use of reflective questions b) The use of closed questions c) The use of assertive questions d) The use of clarifying questions

d) The use of clarifying questions The use of the clarifying question or comment allows the nurse to gain an understanding of a patient's comment. When used properly, this technique can avert possible misconceptions that could lead to an inappropriate nursing diagnosis. The reflective question technique involves repeating what the person has said or describing the person's feelings. Open-ended questions encourage free verbalization and expression of what the parents believe to be true. Assertive behavior is the ability to stand up for oneself and others using open, honest, and direct communication.

A 3-year-old child is being admitted to the medical division for vomiting, diarrhea, and dehydration. During the admission interview, the nurse should implement which of the following communication techniques to elicit the most information from the parents? a) The use of statements that indicate the patient will be all right. b) The use of questions that contain the word how. c) The use of a leading question and those involving yes or no. d) The use of questions that direct comments to clarify.

d) The use of questions that direct comments to clarify. Direct comments that clarify will assist the nurse in obtaining adequate information.

A nurse is working with a 15-year-old client with sickle cell anemia. He was started on a new pain management plan today, and the nurse is evaluating the effectiveness of the plan. Which is not appropriate to include in the nursing care? a) ensuring the client that the conversation is confidential except under extreme circumstances b) answering questions openly and honestly c) including a note about who was taught this new information in the client's chart d) asking only the client's parents to be present at the education session

d) asking only the client's parents to be present at the education session

A parish nurse is preparing to provide a health promotion class to a group of adults in the parish. In preparing to meet the learning needs of this group, the nurse recognizes which of the following as a characteristic of an adult learner? a) The material presented should focus on future application. b) Previous experiences have little impact on learning. c) Peer group acceptance is a critical issue for this age group. d) Their readiness to learn is often related to a developmental task or social role.

d) Their readiness to learn is often related to a developmental task or social role. An adult's readiness to learn is often related to a developmental task or social role. The previous experience of the adult is a rich resource for learning. Most adults' orientation to learning is that material should be useful immediately. Peer group acceptance is a critical issue for the adolescent group.

Which of the following phrases describes one of the purposes of the ANAs Nursing's Social Policy Statement? a) To describe the nurse as a dependent caregiver. b) To provide standards for nursing educational programs. c) To regulate nursing research. d) To describe nursing's values and social responsibility.

d) To describe nursing's values and social responsibility. The Nursing's Social Policy Statement describes the values and social responsibility of nursing.

When collecting data on a client, the nurse implements which nonverbal communication form as one of the most effective to express feelings? a) Eye contact b) Gait c) Body posture d) Touch

d) Touch

A nurse administered oral pain medication 1 hour ago. Which documentation by the nurse best reflects the effectiveness of the pain medication? a) States pain is not relieved, talking with family on phone. b) Rates pain 8/10, states nauseated for last 30 minutes. c) Rates pain higher on pain scale, notified physician. d) Vital signs within normal limits, sleeping.

d) Vital signs within normal limits, sleeping.

Which documentation by the nurse best supports the PIE charting system? a) Blood pressure 88/42 mm Hg, 500 mL IV fluids given, no statements of nausea b) States nauseated, vomiting 250 mL undigested food, hypoactive bowel sounds, antiemetic given c) Vomiting 250 mL undigested food, states abdominal pain, blood pressure 114/68 mm Hg d) Vomiting 250 mL undigested food, antiemetic given, no further vomiting

d) Vomiting 250 mL undigested food, antiemetic given, no further vomiting PIE charting includes the Problem, Intervention, and Evaluation. The only entry that follows PIE charting is vomiting 250 mL undigested food (Problem), antiemetic given (Intervention), no further vomiting (Evaluation).

A nurse is using the health belief model to assess a patient. Using this model, what should the nurse begin to understand? a) Which clinical and financial resources the patient requires to improve his lifestyle. b) What motivates the patient to learn new behaviors. c) The effects the health delivery system has on the patient's health patterns. d) Whether the patient is willing to take actions to support health.

d) Whether the patient is willing to take actions to support health. The health belief model is designed to explain why persons are willing to take action to support their health.

Nurses are occasionally asked to witness a testator's (person who makes the will) signing of his or her will. Which of the following guidelines is true regarding a nurse's role is witnessing a testator's signature? a) Witnesses do not need to observe the signing of the will and can sign it at a later time. b) A beneficiary to a will is allowed to act as a witness. c) A single witness is sufficient for a will. d) Witnesses to a signature do not need to read the will.

d) Witnesses to a signature do not need to read the will. Witnesses to the signature on a will do not need to read it, but they should be sure the document being signed is a will and not some other document. Witnesses should watch the testator sign the will, and they should sign in the presence of each other. A beneficiary to a will is not allowed to act as a witness in most states. Two or three witnesses are most commonly required on a will.

Private insurance most often is called a third-party payer, indicating which of the following? a) You pay no monthly premium, paying for all healthcare costs yourself. b) You belong to a preferred provider organization. c) You are at risk if your healthcare provider is not a part of the plan. d) You pay a monthly premium, and the insurance company pays the bills.

d) You pay a monthly premium, and the insurance company pays the bills. You (and your employer, if appropriate) pay monthly premiums, and the insurance company pays all or most of the cost of care.

Upon entering the hospital system, the nurse discusses the patient's rights and responsibilities that he is entitled to in the institution. The information the nurse discusses is commonly referred to as: a) The Code of Ethics for Nurses b) The Bill of Rights for Registered Nurses c) Standards of Clinical Nursing Practice d) A Patient's Bill of Rights

d) a Patient's Bill of Rights The American Hospital Association developed A Patient's Bill of Rights (revised in 2003 as The Patient Care Partnership). The Bill of Rights includes the rights and responsibilities of the patient while receiving care in the hospital, and range from "the right to considerate and respectful care" to "the right to be informed of hospital policies and practices that relate to patient care, treatment and responsibilities".

A large university hospital has commissioned a multidisciplinary group to review client records following discharge in order to evaluate client outcomes and the character and quality of nursing care that clients receive. This evaluative program is: a) an accreditation inspection. b) a structure evaluation. c) a process evaluation. d) a nursing audit.

d) a nursing audit. A nursing audit is a method of evaluating nursing care that involves reviewing client records to assess the outcomes of nursing care, or the process by which these outcomes were achieved. Structure evaluation addresses the environment in which care is provided. A process evaluation addresses performance expectations during the various stages of the nursing process. A nursing audit may be performed during an accreditation inspection, but this is not the only time that it takes place.

Mrs. Miller is a 60-year-old woman status post a hip replacement. She has had multiple complications following surgery including a skin infection and a blood clot. As a result, she has been a client on the unit for 6 weeks. The nurse has just returned from vacation and this is her first day caring for Mrs. Miller. A colleagues looks at the nurse and describes Mrs. Miller as "quite difficult to deal with." The nurse knows that all of the following can contribute to difficult behaviors except: a) multiple family members in the room. b) fatigue. c) language barrier. d) a quiet room.

d) a quiet room.

Which of the following nursing interventions is an example of tertiary preventative care? a) teaching stress reduction classes at a wellness center b) administration of immunizations to a 6-month-old child c) blood pressure screenings at a senior center d) assisting with speech therapy for a patient with a traumatic brain injury

d) assisting with speech therapy for a patient with a traumatic brain injury Tertiary prevention begins after the illness and is used to help rehabilitate patients. Speech therapy is an example of tertiary preventative care. The administration of immunizations and teaching stress reduction classes are examples of primary preventative care. Blood pressure screening is an example of secondary preventative care.

A homeless patient has been brought to the emergency department (ED) by ambulance after being found unresponsive outside a mall. The patient is known to the ED staff as having bipolar disorder, and assessment reveals likely cellulitis on his left ankle. He is febrile with a productive cough, and the care team suspects pneumonia. A sputum culture for tuberculosis has been obtained and sent to the laboratory. Which of the following aspects of the patient's medical condition would be considered a chronic condition? a) pneumonia b) tuberculosis c) cellulitis d) bipolar disorder

d) bipolar disorder Bipolar disorder is a longstanding diagnosis that requires the lifelong education and treatment associated with chronic conditions. Pneumonia, tuberculosis, and cellulitis are all acute, infectious diseases that may be treated with antibiotic regimens of varying length.

Before starting the education process, the nurse should determine the preferred learning style, age and developmental level, capacity to learn, motivation level, readiness to learn, and learning needs of the client. How does this help the nurse in the client's health education? a) by assisting the client's learning b) by reducing chances of any miscommunication c) by fulfilling the client's requirements d) by implementing effective teaching

d) by implementing effective teaching

Which of the following theories of ethics most highly prioritizes the nurse's relationship with patients and the nurse's character in the practice of ethical nursing? a) deontology b) utilitarianism c) principle-based ethics d) care-based ethics

d) care-based ethics Central to the care-based approach to ethics is the nurse's relationships with patients and the nurse's "being", or character and identity. Deontology, utilitarianism, and principle-based ethics each prioritize goals and principles that exist beyond the particularities of the nurse-patient relationship.

While in a discharge planning meeting, the nurse discusses a patient's goals for discharge. The nurse informs the group of the resources that will be needed to achieve these goals in the home environment. What two types of competency is the nurse demonstrating in this scenario? a) interpersonal and ethical/legal b) technical and ethical/legal c) cognitive and technical d) cognitive and interpersonal

d) cognitive and interpersonal The nurse is relying on cognitive and interpersonal competencies. Cognitively skilled nurses grasp what is necessary to achieve valued goals. Interpersonally skilled nurses work collaboratively with the healthcare team and provide the team with knowledge about the patient's valued goals and expectations. This situation does not require the technical skills of the nurse, as technically skilled nurses manipulate equipment skillfully to produce desired outcomes. The ethical and legal skills of the nurse are not paramount in this situation.

The nurse is caring for a patient in a critical care unit. The patient's cardiac monitor alarms, and the nurse recognizes the rhythm as atrial flutter. What two skills did the nurse use to interpret this cardiac rhythm? a) interpersonal and technical skills b) interpersonal and ethical skills c) cognitive and ethical skills d) cognitive and technical skills

d) cognitive and technical skills The nurse used his or her cognitive and technical skills to interpret this cardiac rhythm. Cognitive and technical skills equip nurses to manage the clinical problems stemming from the patient's changing health or illness state. Interpersonal and ethical skills are essential for concerns related to the patient's broader well-being.

The nurse has paged a patient's primary care physician because the patient's potassium level this morning is 2.6 mEq/L, a value that the nurse recognizes as a significant threat to the patient's cardiac health. What has the nurse enacted? a) technical skills b) interpersonal skills c) legal/ethical skills d) cognitive skills

d) cognitive skills The nurse's cognitive knowledge of anatomy, physiology, and pathophysiology underlies the understanding of the patient's potassium level. Technical skills involve the ability to perform kinesthetic tasks while interpersonal skills address patients' psychosocial needs. Legal/ethical skills provide the basis for providing care that is fair and right.

A client shares with the nurse how much she appreciates understanding the physiology of her breastfeeding. She states, "I felt very comfortable with what you explained to me and I feel I will be successful at breastfeeding." In affective learning, this represents: a) creating rational thought and learning. b) creating an educational opportunity for the future. c) creating specific learning sessions for new information. d) creating an atmosphere for discussion of feelings.

d) creating an atmosphere for discussion of feelings.

When a state attorney decides to charge a nurse with manslaughter for allegedly administering a lethal medication order, this is an example of what type of law? a) public law b) private law c) civil law d) criminal law

d) criminal law Criminal law concerns state and federal criminal statutes, which define criminal actions such as murder, manslaughter, criminal negligence, theft, and illegal possession of drugs.

The terms "criteria" and "standards" are often used interchangeably, but they actually have distinct definitions. "Measurable qualities, attributes, or characteristics that identify knowledge or health status" are known as: a) evidence-based practice. b) evaluation. c) standards. d) criteria.

d) criteria.

What step of the nursing process is the nurse performing when analyzing patient data to identify patient strengths and health problems that independent nursing interventions can prevent or resolve? a) assessing b) implementing c) evaluating d) diagnosing

d) diagnosing Analysis of patient data to identify patient strengths and health problems that independent nursing interventions can prevent or resolve is a function of the diagnosis step of the nursing process. Assessing involves collection, validation, and communication of patient data. Implementation is carrying out the plan of care. Evaluating is measuring the extent to which the patient has achieved the outcomes specified in the plan of care.

Which of the following theories emphasizes the relationships between the whole and the parts and describes how parts function and behave? a) developmental theory b) adaptation theory c) nursing theory d) general systems theory

d) general systems theory General systems theory describes how to break whole things into parts and then to learn how the parts work together in "systems". Nursing theory attempts to describe, explain, predict, and control desired outcomes of nursing care practices. Adaptation theory defines adaptation as the adjustment of living matter to other living things and to environmental conditions. Developmental theory outlines the process of growth and development of humans as orderly and predictable.

Noncompliance with a therapeutic regimen can be a significant problem for elderly people. Which of the following is one of the common reasons for noncompliance in the elderly? a) lack of time b) religious practice c) childlike behavior d) inadequate financial resources

d) inadequate financial resources Noncompliance is often associated with inadequate finances, along with factors suck as patient confusion, disappointment, misunderstanding, or fear.

The American Association of Colleges of Nursing identified five values that epitomize the caring professional nurse. Which of these is best described as acting in accordance with an appropriate code of ethics and accepted standards of practice? a) altruism b) autonomy c) human dignity d) integrity e) social justice

d) integrity The American Association of Colleges of Nursing defines integrity as acting in accordance with an appropriate code of ethics and accepted standards of practice.

A student nurse has reconstituted a dose of ampicillin by adding sterile water to the vial that contained the powdered form of the drug. The monograph attached to the vial specifies that the nurse should wait 30 minutes before administering the reconstituted drug. The student has asked her preceptor for the rationale behind this waiting period, but the preceptor is unsure of the rationale. Together, the student and the preceptor have approached a pharmacist on the unit for an answer. The preceptor has demonstrated which of the following attitudes? a) intuitiveness b) being fair-minded c) discipline d) intellectual humility

d) intellectual humility A nurse who is willing to acknowledge that he or she does not know something, and who is willing to openly seek out answers, demonstrates intellectual humility. Intuitiveness and discipline are not central in this scenario while fair-mindedness is associated with being open to differing points of view.

The nurse and the physical therapist discuss the therapy schedule and goals for a patient on a rehabilitation unit. What type of communication is occurring between the nurse and the therapist? a) organizational b) small-group c) intrapersonal d) interpersonal

d) interpersonal The nurse and physical therapist are engaging in interpersonal communication, which occurs between two or more people with the goal to exchange messages. Intrapersonal communication, or self-talk, is the communication that happens within the individual. Small-group communication occurs when nurses interact with two or more individuals. Organizational communication occurs when individuals and groups within an organization communicate to achieve established goals.

A client has come into the clinic for a postoperative visit. The client states that the postoperative pain continues to be 6 on a 0 to 10 rating scale. The nurse evaluates the client and the current plan of care. Based on the information provided by the client, the nurse should: a) terminate the plan of care. b) call the pharmacy to determine if the client is taking pain medication. c) continue with the current plan of care. d) modify the plan of care.

d) modify the plan of care.

A nurse is discussing the benefits of smoking cessation with a patient. The nurse informs the patient that smoking cessation will reduce his risk for cancer, improve his respiratory status, and enhance the quality of his life. The nurse also shared her story of smoking cessation, provides information on other individuals who have successfully quit, and encourages the patient to attend a support group for smoking cessation. The patient discusses his feelings on smoking cessation and verbalizes a desire to quit smoking. What type of counseling did the nurse provide to this patient? a) long-term counseling b) situational counseling c) developmental counseling d) motivational counseling

d) motivational counseling Motivational counseling involves discussing feelings and incentives with the patient. Long-term counseling extends over a period of time. Developmental counseling occurs when a patient is going through a developmental stage or passage. Situational counseling occurs when a patient faces an event or situational crisis.

Mr. Edmondson has always prided himself in maintaining good health and is consequently shocked at his recent diagnosis of type 2 diabetes. The nurse asked Mr. Edmondson, "How do you think your diabetes is going to affect your lifestyle?" The nurse has utilized which of the following interviewing techniques? a) validating question b) reflective question c) closed question d) open-ended question

d) open-ended question The nurse's question allows for a wide range of responses and encourages free verbalization, characteristics of a useful open-ended question. Validating questions allow the nurse to confirm what was previously said, while closed questions necessitate a "yes" or "no" answer. A reflective question or comment repeats what the patient has recently said.

Of all the physiologic needs, which one is the most essential? a) food b) water c) elimination d) oxygen

d) oxygen Oxygen is the most essential of all needs because all body cells require oxygen for survival.

The nurse is preparing a care plan for a 68-year-old African American man who was recently diagnosed with hypertension. Age, race, gender, and genetic inheritance are examples of what human dimension? a) environmental b) sociocultural c) emotional d) physical

d) physical The physical dimension includes genetic inheritance, age, developmental level, race, and gender. These components strongly influence a person's health status and health practices. The emotional dimension focuses on how the mind affects body function and responds to body conditions. The environmental dimension includes influences such as housing, sanitation, climate, and pollution of food, air, and water. Sociocultural dimensions are health practices and beliefs strongly influenced by economic status, lifestyle, family, and culture.

A school nurse is discussing bike and outdoor safety measures with a group of Boy Scouts. What type of health teaching and counseling is the nurse providing to this group of children? a) restoring health b) facilitating coping c) promoting health d) preventing illness

d) preventing illness Preventing illness includes first aid, safety, immunizations, screening, and identification and management of risk factors. Promoting health focuses on developmental and maturation issues, hygiene, nutrition, exercise, mental health, and spiritual health. Restoring health focuses on developing self-care practices that promote recovery. Facilitating coping assists the patient in learning to cope with permanent health alterations.

The process of nursing diagnosis carries legal implications for nurses. Which of the following legal responsibilities exists for a nurse who has documented a nursing diagnosis related to a client's kidney failure? a) independently managing the client's kidney failure b) coordinating the treatment of the client's kidney failure c) choosing interventions to resolve the client's kidney failure d) reporting signs and symptoms related to the client's kidney failure

d) reporting signs and symptoms related to the client's kidney failure In producing a nursing diagnosis, a nurse creates accountability for detecting and reporting the signs and symptoms of a medical diagnosis. The nurse is not legally responsible for independently managing or coordinating the client's treatment. Choosing and performing interventions to resolve the condition is primarily within the purview of the physician.

During the admission assessment of 40-year-old female patient with a suspected mandibular fracture, the patient discloses to the nurse that her injury came as a result of her husband hitting her. Which of the following actions should the nurse prioritize when responding to this disclosure? a) Informing the patient of her right to keep this information private b) performing an assessment to confirm the patient's statement c) ensuring the patient's statement is confirmed by another nurse d) reporting the abuse to the appropriate authorities

d) reporting the abuse to the appropriate authorities Nurses have a legal and ethical obligation to report cases of abuse. It would be inappropriate and likely unethical to require a third party witness to the statement or to withhold action pending assessment results. The nurse's obligation to report abuse legally supersedes the patient's right to privacy.

Five-year-old Bobby has dietary modifications related to his diabetes. His parents want him to value good nutritional habits and they decide to deprive him of a favorite TV program when he becomes angry after they deny him foods not no his diet. This is an example of what mode of value transmission? a) Modeling b) Moralizing c) Laissez-faire d) Rewarding and punishing e) Responsible choice

d) rewarding and punishing When rewarding and punishing are used to transmit values, children are rewarded for demonstrating values held by parents and punished for demonstrating unacceptable values.

Which healthcare provider is a major source of health assessment and health education for children? a) hospital emergency department b) community center c) nurse practitioner d) school nurse

d) school nurse School nurses do provide much of the health assessment and health information for the nation's children.

When you ask an older student why it is necessary to change the patient's bed every day, he says, "I guess we have always done it that way." This answer is an example of which of the following? a) unsubstantiated knowledge b) scientific knowledge c) authoritative knowledge d) traditional knowledge

d) traditional knowledge Traditional knowledge is the part of nursing practice passed down from generation to generation, often without research data to support it.

A nurse in a physician's office has noted on several occasions that one of the physicians frequently obtains controlled drug prescription forms for prescription writing. The physician reports that his wife has chronic back pain and requires pain medication. One day the nurse enters the physician's office and sees him take a pill out of a bottle, and he mentions he suffers from migraines and it really helps when he takes his wife's pain medication. What type of nurse-physician ethical situation is illustrated in this scenario? a) conflicts regarding the scope of the nurse's role b) disagreements about the proposed medical regimen c) claims of loyalty d) unprofessional, incompetent, unethical or illegal physician practice

d) unprofessional, incompetent, unethical, or illegal physician practice The physician is demonstrating unprofessional, incompetent, unethical or illegal physician practice.

The nurse is addressing primary prevention with a group of college students. Which of the following is an example of a primary prevention measure? a) annual pap smears for the female students b) testicular examinations for the male students c) HIV screening d) use of condoms and safer sex practices

d) use of condoms and safer sex practices Safer sex practices, which includes the use of condoms, is an example of primary prevention measures. HIV screening, annual pap smears, and testicular examinations are secondary prevention measures.

An obstetric nurse has reported to the nurse manager that he observed a colleague taking narcotic drugs from the medication cart ad not administering the drugs to the patient. The nurse manager dismisses this information and does not investigate further and asks the nurse to overlook the violation. Understanding the seriousness of this situation, the nurse notifies local authorities. The nurse;s notification of the local authorities is an example of which of the following? a) developing technical skills b) developing accountability c) intuitive thinking d) whistle-blowing

d) whistle-blowing Whistle-blowing refers to employees who report their employer's violation of the law to appropriate law enforcement agencies outside the employer's facilities. Many individual nurses and nursing organizations are working to secure state and federal legislation to protect whistle-blowers.

A nurse is using the ISBARR physician reporting system to report the deteriorating mental status of Mr. Sanchez, a male patient who has been prescribed morphine via a patient-controlled analgesia pump (PCA) for pain related to pancreatic cancer. Place the following nursing statements related to this call in the order in which they should be performed. a) "I am calling about Mr. Sanchez in Room 202 who is receiving morphine via a PCA pump for pancreatic cancer." b) "Mr. Sanchez has been difficult to arouse and his mental status has changed over the past 12 hours since using the pump." c) "You want me to discontinue the PCA pump until you see him tonight at patient rounds." d) "I am Rosa Clark, an RN working on the second floor of South Street Hospital." e) "Mr. Sanchez was admitted two days ago following a diagnosis of pancreatic cancer." f) "I think the dosage of morphine in Mr. Sanchez's PCA pump needs to be lowered."

d, a, e, b, f, c. The order for ISBARR is: identity/introduction, situation, background, assessment, recommendation, and read-back.

A student nurse is organizing clinical responsibilities for an 84-year-old female patient who is diabetic and is being treated for foot ulcers. The patient tells the student, "I need to have my hair washed before I can do anything else today; I'm ashamed of the way I look." The patient's needs include diagnostic testing, dressing changes, meal planning and counseling, and assistance with hygiene. How would the nurse best prioritize this patient's care? a) Explain to the patient that there is not enough time to wash her hair today because of her busy schedule. b) Schedule the testing and meal planning first and complete hygiene as time permits. c) Perform the dressing changes first, schedule the testing and counseling, and complete hygiene last. d) Arrange to wash the patient's hair first, perform hygiene, and schedule diagnostic testing and counseling.

d. As long as time constraints permit, the most important priorities when scheduling nursing care are priorities identified by the patient as being most important.

A home health nurse who performs a careful safety assessment of the home of a frail elderly patient to prevent harm to the patient is acting in accord with which of the principles of bioethics? a) autonomy b) beneficence c) justice d) fidelity e) nonmaleficence

e) nonmaleficence Nonmaleficence is defined as the obligation to prevent harm. Autonomy is respect for another's right to make decisions, beneficence obligates us to benefit the patient, justice obligates us to act fairly, and fidelity obligates us to keep our promises.


Conjuntos de estudio relacionados

Welding Carbon and Alloy Steels 2nd Year

View Set

Orange County Sheriff's Department | PHONETIC ALPHABET

View Set

Electricity and Electronics Exam #1

View Set